You are on page 1of 134



        


!#"$!&%'()* + 
($,
() $-./0
12 2 3
465798;:=<9>;?
@6A9B;C;D=E9D

12 2 3

FHGIKJL)MNPOQ)QR
OQ)QRTSVUXWVFZY

[ \^] N`_HababcdXNHe fg'hc(L)iG


jlk

m gPn

opKqr s0tKuHrwv*x(pKq
r x'y{z)p |}v*|Kz)r~HqH
l

F(;{M}GdXNiK{#(LNGI \ c(M}G0
l

O ; L(IKc(L(i_0M}GdXNiG
c(M'IhN`_M}GdXNi
G hNHNI
llll

gK

[ hNTL}GN`c)H_Hab0Lab_I*c(iG \ G [ _HababcdXNHeK

0 ** H wK0 0 H 0 H bH*

& # 0 H0 H  *

0
}l

( K0K `H^w{0^H0 { H wP0



9
0
* * w H0* 0w* 0 




  
9

 !




 "#$%&
00 * *b }0 P  )( 0X ' 0+*T (
*H,( - 0{ 0 w0* { *0. 0Z { {0* ).
0 */$T  .H+
00 `*0*& 0 1( &0 ^
H  1( T0 02
* ' P0

 0* 0T H {*00
0* P 0'P 0 0
w0*T0 3 /P00'  H4(0 1({H
T0  00H  }b 0 P0* 6 5 6 (0 ^ * bHK*0*
*Hw 8 7
9
:;=<>@?AB 
C &
` D
'XE
*. G
F B=H=I $J BKL
N
PRS HBTUVW N
\^_ VW`a
MOJ B  /X9Q
X <Y [Z ]
*'Q
'{Kc
b Q
de ?fBKI N
ijk 3 /T9m
ln "
g$h ^_ VW`a 
o  >pqr =$%&ts

0
}l

'TT0TKH
H

0 0 P^1xH H0H 00 0H y 0' ^3x


0H z{ *^*|(0  
8 } 3 ~

0

}l

*( uwv

uwv

]
z{

V


8 } 3



}l

* 00*$0} }0 H00^}T H0TTH 01/1 T 0  ^0



K* 00  H20 bH0 *0 0H * 00*P
9 
?f  I
?
1/1 mf  

` J
 [

)

0{001xbH H
H w 0
Hw*  0*

# 1 313 

^ 
H* H  0


0Hw* }0y
 m 
11 $

0



11

313 6


9

T>

0
}l

bH 0TH

$%&

}l

0 00*

"$%&

0^* 1(

0

0
}l

3D,|1N3c116|1118Q,111c81|1+w=12
,-w y821y

Q+1

 
    

! "$#

% '& )(+*-,/. 10
2,/.
3*-0
4, *. 5& +*.687 9,:0;687
0<.=6
:4/*:,/.
> (?*-,/.A@B?0DCE,/.FGHI KJMLN*-0DO " 4,P*.-QN*.6R7TSU,:0D6R7
V 0<.=6 W  4/*:,/.

1 1

f11+

YX Z

w

|1

= )

Q+1

! "$#

\[

N1D11 8c1

]_^ `bd a5c d ?e

/f hg 1i

1 6 y

11

je )X )%

3 )

c1

9e
+k 9e
ml
Qno pqrs ]_^ `bd a5c d !_tu@vfwS?gxSyi2z{X}|~%}GAsP{XW
%#W "s~P"  G 
3,y,w2 D16

,w2+

&

88

| |1

8Qc

+61 ,

|1G w1y3

Q+1

! "$#
2

e
fe
g
ye 5\5
je
e
je
m[
?f
7
H fT g< ~ tu@ V 5\5 
      u GAC "HfG
 
5! 7"$#

D

|1D

1 E16w8m1

168E

+16w8

111+|1

|1D+E36N2,11 1 3

6 c|1m|1

16w8E3

|1E38) , ,wE1 w

++

7 A& (+*-,/. *-,6T7 /R )4/*:,/.A6 X\ h


4/*- ,6) 4/,:.=6 75X\ h /,:
7
> (+*:,/.A@B1*-,67TS /R V 4/*:,/.6 X\UwCOFGHI KJML
4/*- ,6) V 4/,:.=6 75X\N,:M
5! 7"$#

Q1

1

Q 1 1

|1

1+w|1

++

Y 
_*M6 PX 9,6 3X 26 X 3 e
.
P
-(+*:,/. 1
e
.
+
7
> OFPN*6 _X SK,6 _X :6 X _  tu@ 
  .8CHI KJMLC~(?*-,/.AGA:n.RIGA M
F  GA
5! 7"$#

|1+111

8 E

,

1N


m 8

G1

1 w|1 ]-|3w+8 |+1

 1D1 c16

 1

31



 A  

3

|1E13 Dw3

1

E 1 )

|1| 2c1 |1

++

++

!57"$#


 
7  75X k 9e

75X
\7  57 X\l
7

 )S7RS S75X Gr
s   su WS
7TS S-75X75Xsu@G   
5! 7"$#

11

1, ,w 1

|1

w,

Q1

1y|1

 ,8|1c1

wE 3 f

++

A(+*-,/.

)7 !
(+*-,/. 2 " $#
%*- &,'" (.)#
e
9e
ye
4**- 54*,'" )4*.+#
 \X\ h
3(K,"-#
7
*-,/.}C/./01/2/30/4 7T5 =C(?*-,/.76AGHI  V MS5"S+#98 O
FG:1I KJML;*-MS5,'" ;.+#;/ <./=/1/2/3{!?>I <@/A:BC<D
E #GF:HJI G7>2*4 *-MS*4 ,'"4*.+#<
./K\X\B(K,"-#GA

5! 7"$#



 w+e

[ G

1 )Q1|1

1w

3+6|

|1

8Q1cc3

w6

L

1

1 1

1

11

|1EE1

/M

G

Q1

- D

 3G

|3

1

++

 , 3fw]|1

&M

,-w yE1

NS PO X V X QR
RPN W TQ OUQ

y3]|1

++

V
G  s[MG r 
S NPRPO N X W X TQ QOUR Q 7

LMSGM<Y/Z

!57"$#

A& 8(+*-,/. +*:, 6 Y K,/.6 =.K*;6 ?


W(+*-,/.
4 *-,x6W4 ,/.=6W4.K* 4 *-,
7
> (+*:,/.A@B1*-,x6 Y}SU,/.=6 TS.K*6 WUw C(+*:,/.76GHI KJML 4m*-,
6}4 ,/.=6W4.K* 4 *:, 
5! 7"$#

= 1

y1

=3

1 1

w|1

++

\^l ]_a`cbdPe m5npiko j fg_hf&dPe m e


\^l ]_a`cbdPe m5npiko j fg_hf&dPe m

Y% A&
Q

75X

zy{ ~ } | wyx

2 11++8111

e

+31G1D

372e

11,1

+e
1

9e

?f

N1 6

1+1,1

++

1|1+3 8 )Q8+

3

&(P&
/

!57"$#

1y|1E3 )2E


zy{ ~ } | wyx @ f G7//D/   tu@  
 s['T:
[:7T 

,q ?r G 313sqtr
 tu@uq S5r/ s~ vqtr}
1 w

13


 s

++

!57"$#

 
         
!"  #%$  
&' (*)++,.-0/1 243  2

657 4 89$ 5 (

:  (<;=>
(
?
@>ACB"DEF

)A

LA

MNPO

VA

WX

@ HA
`

HACIJ

KACEI

Q ACIJR

SATF I

,A

@>+AZYYR

@ @>A\[[R

@ )A

@>KA

bb
c

@>LAdD
F

@ Q A

@>,A

i
jk
lmn

@>VApoqF

)+AE

a _

@>SAIJ

GG

D>D UG

] ^

ePfg
h f

$ 5 (
?

D>rtsvuqw xzy|{w~}yxvy }=6x  C



r9y
uqy w x"x
yy


> r .
 w=6=uzxq

rx

>xw} 4x

r9*

y9  ry

= 

y 6y{wy7y>xz

y
xx

w zxJ
=x.u
=

 g

 y x
.q
 ww4 


     

  
         
#  & $  ' %  (  # )    '  # ) !" 
* 1 2 +",-* 1 2 ./,3 0 54 6
/7 9 : 8
? ; @=
B A A E C A AF D A A
LT MON K U B P K E U C QSK F RVD NGN
[;]\ ^ J_/= ? ; @=

{7y xzyyxz


xu

1 


6  r

I rty xzuxw

 wy x} zuq6

E*G



xy'y

r =
 y9xw zxYr 7

x#x
=y x x'{
y x} zx yx 
>

q 

 ux*xy ywxz>x{
y x}% zx
yx r xww xx =y x

x

>7{x

<; >=
G7
H
I; >= ? ; J=
WZXZ Y

x4 w~}={yywy x} y

7Y *D
[ FR r

x

6 ww zx

{yw

 wuq

G Y 7


y

7
b

Fr

Ga

7 6x<4
ww9u'uq>zxy  y y'R *y 6x<'
ww9u'uq>zxy' 7 #x* y
   }4FRERG R 9YRr *> yq'G'{yw' >r

zu{{x< x.x  uq>zxy6 7 x<r9*

Gc

u'6 }q1 }
 > 

yxz

'

xv  x%xy1

/d

D=xzv[ry xx xy6 u'D=xzEx


1
1FY{yw

D.xFYr 4 x> FRy uw 7 ux  D>D


II FF"D
F IY D
J F E D
[ IF I[ F D

e4
or p pks pGq
~;  k 

xz
y1'{7y6 xz9  xzx vD
R1
6.x=y
 
6IG FG.x y| 6 ww zx

f glmhil h hiljkn j j
tIu/vxwGy/z|{ };
=

{y6'7 xz "  xz x


6E
6 .x y
 r*'>zx
'{7yw >r =
 .x


{yw' >9yxy9  
6x4
z=
*y

O=

d4x
{
7
{6 >y
==} uxy
9
x
 ux y| xzw}r

E r

xxx

y7
 

x=

yyyqw.} [r zxww xx

y xz%x uqxy

e\

wy

=uw > D
RRr

x

wy6yxzy

y

 >6
z=
EI r 4 x6x

9xxy

G^
/GJ H
/GJG H
J - | m 4



q

\ N ^
^ \

x yw

r =


YR

x

 9


xx

{
 ww w

I >vr4*

y>

  @7
xz

y
dy

x' uqxzy== xx

r 7

y
 wx

{
 ww w

}y w6yxz

ww uxzyxxx4
z=
9yuyuq r

w xI
y

6

>

x

y

y9xzxE*x~
z=y

y }y w

Grtx

y"x uq
wxz

y9xw>>zxI r 4.x4x
*


Yr

wyzx4x wuq 6

G "  O 
J7
-

Jc

@ 

  @ O k / H7
G |" "


xz

y=xz

 
xzy

r xww xx

\ ^

 
          
(*) - +03 &#  $'2 " ,4 1  . #  $%2 " /  ! "

8 9 75 6 < = :7;
 wuq=q

=

q

 >

 
r

> ? @BA7CDEL CF@BA7GDIM HJ J K

*u

ydx

zx

IJRr*yd6 y=uqCy

#xxz
y w

r>*

 x4
z=
9y*IJRr

=

>

^` ] a _ bdcfeg h7e` i a _ jlknmporqsutvswq e xy zI~{ |}{ b


Y P Z\T[ UWY QSV R Z X R
jSgro e7hopm feh e7hro o g uBsutne o e7j*e o eB *j ` i a _ B !ms opsute
sgopee  g 7je7  ^ 7 g q

o
B

m
*
m
u
s
v

r
e
v
t
7


k

e

e



g
p
m
f


e

w
s
m

\I\  b
b !m*s opsutvesgope^e veo q eg opeo eSgkjfe eg q sg e*!m wb! e7g swme7k7
o 
     
  b oswmg g q e7h*e7m*sg7g qh 7gemog q g u ojlk opswem bope
7 m* o o s mm*wsouwemmlo 7g  opsg s o mS *vsjops g fs o o e *e7opemposgopee
kg hops gm*e7jStve7qg qo eSmpkj osg Bsgs oBro e e7 jSeo*shSm*e*semopeg!bcfeg h7e B    
s mm*wsouf *e7opeo g "!$)+#&* % %(' bcfeg h7efedop 0 1 ,.- / g q 0 1 ,.-.2 g qeo
< 8 5.6 7"9 ? A=C 5.:6 7 > D B@ =AC 5.: 6 3 7 > D B@ =AC 5.: 6 4 7 >D B ? :"; g q N J G.H.ILK Q SOU G.H.I I P V TR SOU G.I H.E I VP TR SUO G.I H.F I VP T Q I"M
*eme7hosute7ubpodB rm o o     B g q g u B XW. Y  Z g q !b e
g!mpfeswmoknm [.\.]d [.\ ^_d [.\ `_d [.\.ad [.\ b
e c cLa ] b
h
!bgfqkh7eihj olkm* o omhnopr qsklb egutvqwhkxWytzn{hj|W
tvqs kmh m g q'm*yqsk} Wqwh~W !b mvhnjywkrqwjrfe te

z ! l sh7 sute mwhn.W !b L . uvsg h7!m*sg e


w sghnrqffe te
g q n
j

e7g he L. z b
k
!bopeo o ( .z . . g q jS *7ge *j op' !be*u e77h7 gkjfe
*jop o os m *e7wosute7u *swjerop Bs b e b g oqsutvs m*suwe 7g q Z" sutem *s m*erop
!mm*suvwet7 kne no egkje*op Bb7 egkjfe mpknh!mm*suvw e tv7ukems m
" "( b
 s od*ej7sg!m oh7kvgoo !m*egkvjfem * j&fo l sh7*eg od*ewosute7u *sjeop
g q f swh7  sutve *swme o g e !mm*suvwe tv7ukem gkjSe* op mb em*e'*e *e7hswme7 gkjfem
q sutvs m*suwel fe g e7e7qg oh7*e vko o e Bhop Zm  *e77qevh7ee7qm "b e*e*e
mpkh7%gkjfem* g 7je7SY Z e77h%f swh7'sutem*s me o g e !mm*suwe
gkje*op o e f*e .Ym  Y *eme7hosutve7bponB wmo o!o eg!mpfeswm "( & b
O
Jr

xzxx

yyyyqw

}

b egkjfe f m m*e7we7hopsg o f tvem o em*7jeh7kv swm l swero o


m*e7we7hopsg o l tvemS q sBe*e7golh kvms mzbw eoknm tve          l sh
m*sjSws swemop
   b  sg h7e                   jSvswjs m*sg sBm
e7ksutv7we7goo jSvsjs msg ! b m # *, + 2($ 2" -*35. 4(6(+ 7 / %'&(&') o ejSvswjfkj!!mm*suvwetv7uke
! s m 0(0 b swmh7 *em g qmo 8: 9 ; 1 ;



g
q

v


s
f
j

k

j


m
*
m
u
s
v

e
v
t
7


k
e

ve7sg <(<C B =5>(?(@ D >(>(A b


b m m lgsgo eBskv*e7eo F E g q H G ve
o e h7s *hkjShe7go*em IhnKJ g qLInrqMJ
*eme7hosute7uON\ve oe jSswq Bsgo inKJ
g Qq Pve o e jSswq B sgo lhnlRb  sg he
gwe o h7ego*e\s m o lshe gwe o
h7s *hkjSe*e7gh7e7fee To SrVh UXW nW ( Yg q
o e*eBweZ Sz[n UV\ q W .( Ybcfeg h7e

UZ

P
o jMk[l ] p `^ _ba 'm n ced(f(g a m'n (g h'am i p d(a m
HG
F E TN H G
rtq s w uKv
t [
[
K
t 

q
J
UV

 g qm*rhn *eh wsge7b


opeo o
g q *e'h wsgeb!kvpo e*jS *e7lfe tv e {K| ~ }zxz} y g q
m*sjSwe h7jk opos gsutem
bXope wm* o o %m*
!*m o o
b

!b s op Bs eqJ}  l e*e feh7g
h7 gmpopkh olhs *h7wefs op h7ego*eZJg q*7q s knm[Jh
l swh7s m o em7jemTJmnbtve sgo go e
h7s *hkjSe*e7gh7e o e h7s *h7we s mE!mm*suv e
!ms ops g% qbg'oshkwBl e7g opemo e
e o*e7jSetv7ukem7g qvfeeoo lhs *h7wemfs op
h7ego*em " g q J.Wy *eme7hos te7 7gq
fs o\*7q s knm 7g q *eme7hosutve7 7g q o e
*tvsq efo efvkvg q  B o eS*e7q*e s g!b!go e
skv*e7o e m7q e7q*es gSs mo e m*eo *e7q sgo mb

opedo ohJ n W (!mm*e7hoJhn m*e7

( 5'``5'((b5` e e e

5` (' 5 (


 
 

 

      
             

& '

 

"
$

 (
$

 (

$
$

 (
$

+* (5

 ( )
!

5
0

6 1

,-

6 3
,

b` bX( 5 ( (e( 5e (<;

55` ( ( (`'` 5'(

6 2
,

`5``'('(('(
X(5 ( 5(``5`(b

6 7
,

8 :9

@ E

AB@ = F A =
C = F @ =
D = F C =
H
H
H
H I
C
C >
C =
C ?
G

((P9:RQ( (<;

O ;

S
V [
]

b` b

8 :9

L` N

'

L N

(`

e
j

\T V

S
\U X

Y
^

ad o h
ln
dgmn
f

a
o

\S W

WBV
^

'
c k

bf o d
lp
dim
c

JKJML` N

^ _

X
a

JKJML N

o r

` } 5~X(<;

t y z u v s { w y s z t y { u v s | x y

((P9:RQ( '

 
:

JKJML Q N

 
:

JKJML :N

O ;

b` b

8 :9

L :N

'

LQ N

(; ('; b

(; b

Z[( : 8  (( e ((5( ( ; 9 9:


  (( P9 5  ` J ` <( M  P9 5 L N ' `b

( Vb ; 9 9: X5(` R5R':(be(5('`b( ( `' K< `  z( 
` J `
Z(
; `( (5  ' `' (; 9 9:  (5 5(  5' `}'e
( ;  9 P 9: (5 ((5   P 9 5 5 ' ` O ' ` (; 9 P9:Q((`
5(  5 ' ( K( ;  9 9:R( (`(T(5   P 9 5 5 ' ` e ( (5
(`(( ((` '(M(( e` ( R ' (e    P 9 5 L   ` N ((` 
( ' L 5( :N ( T((` '(   Qb  (  ' ((b
*
(  < `   R  P 9 5 ((  9 R ; `  ( 5(55e(  Qb (O(( '  O'
  ( ` +  Qb  5(   e( M( (5' 5K      



ZX( 
T  5( 5R 5  `b
`b' 9 b 5b' b  ')( '"!*,$+ #&% '
X5( .
-/
0213-4 5 (` 68: 7 < 79 5(>=
0@?
7
7;
= 5 L (`e' (A=
0 2?B= N X (
QR
(< ; G CEDH I FDH (b S R U J QT U KELM&NO&P V + V(
F
` <  `t( W 0 5(@
E '< ; t(`
`b X5( . -4 50 K((``  5` 5

YZ\[ ]^&_`acb
[
X l
km
Y

l efY








]^&_d`&a
X
k
Zgh]^&_gaicb
n
k

l efYj
o Y

 9 `  M(5' 5KK((  EP9 (`5(y-/


E

prqsut&vw)x

(b``('``  (;

 i*

zz
}

{)|}
}

~~ ~
~|

~|

X5(`

e( b

 5('5( 

X :9b` b (L```Z5 '




* ' > ( (55e(':Re(O( ` e( (`(5 5R5'5
9 `` 
5(  ` R 5(`( ( ` bV  5Z Q* ( `(  '(('[



5( ,

j
&




( `e5 5(-/

5((`(`.-/
z( K5`

 i*


"
,


$

\f
\f
"
,

X5(Ke( bO = 
@?B= ?u= 54
2?
E 5( +* (5 ` (;

=
>@?

9:

c


\\u


\

5((`(`        



 ('~  (`( r -4
[-4 5(r-4 54
` '` ; ` (<;
&  * '  +    ,  '    (
)  * &  +    ,  !&  "#  (
'  * )  + $  ,  %) $  (
 (((P  X(b`  (K5( '  (' '
. /10 2 9 = 0 3 : = 04; 5#67 8 < > 0 2 - = 0 3 - = 04 -
@AB R W
C D E
F GC H ? S V C I ? T V CJL? U K W C D E
e' M( M1N B R W OP
X
@ R C H S V C I T V CJ U C H V C I V CJ F Q J

>DC
<
E
>BA

=
;

8 _ ` YZ[]\^ a * (`t(R(; ` ( 9 ``' b c 8  5 (9  (b5(


`5 5b'bX5  ' '(:9  ( f l d r g m e s hi l e r j monplqr n e t k
* ( 9   (`''R'( ' `('e( vwxzy ~  y{vwx ~ |}vwx u ~ "
   { " " #  "L ""
"" 1  1 1 " 
]  1#  "  " %
 "  " ""# "o  o  "  "

 
 #
 #



 

"       "   %    !" 
 
" " $# 1 % " & % ) ( ' ) * ' " - , +  "
/ 4 . 5 0 4 6813
7 2  "  " "  

GIH KJML

 o " "]

O N

"QP

V8SU
V T R

" o L

W
XZY[
k W b f i XZY
g c a j bb
d
e fe W\

e
f h XZY[
k WZ]DY^W_`h YWXZY
h
l
>
O N
L

o L  L ml L   " " o }]  


" " "on " o  o #L  " o " " H "
" " " o L o  l " o "
 " p # " " p] rqZs x w t v u y8s x w v u n1 zl p " 

}~ Z
Z Z{ | Z
Z } ~  
Z  } ~

  }
 ^Z

}

}
{ Z

`D

" "  "## "  >BG  " o


  > " A  # G  [ 
DID " o "" L  o M    n o ? p
J o % L    " " #n L ? "
 " " >@H"H"> p  o " L " 

International Mathematical Olympiad


Hong Kong Preliminary Selection Contest 2007

2007
26th May 2007
2007 5 26

Time allowed: 3 hours


3

Instructions to Candidates:

1. Answer ALL questions.

2. Put your answers on the answer sheet.

3. The use of calculators is NOT allowed.

Section A (1 mark each)


1
1.

Someone forms an integer by writing the integers from 1 to 82 in ascending order, i.e.
1234567891011808182. Find the sum of the digits of this integer.
1 82 1234567891011808182

2.

Find the smallest positive integer n for which the last three digits of 2007n (in decimal notation) are
837.
n 2007n 837

3.

ABCD is a parallelogram with D obtuse. M, N are the feet of perpendiculars from D to AB and BC
respectively. If DB = DC = 50 and DA = 60, find DM + DN .
ABCD D M N D AB BC DB = DC =
50 DA = 60 DM + DN

4.

One day, a truck driver drove through a tunnel and measured the time taken between the moment at
which the truck started entering the tunnel and the moment at which the truck left the tunnel
completely. The next day a container was added and the length of the truck was increased from 6 m
to 12 m. The driver reduced the speed by 20% and measured the time again. He found that the time
taken was increased by half. Find the length of the tunnel (in metres).

6 12 20%

5.

ABC has area 1. E and F are points on AB and AC respectively such that EF // BC. If AEF and
EBC have equal area, find the area of EFC.
ABC 1EF AB AC EF // BC AEF EBC
EFC

6.

Let [x] denote the greatest integer not exceeding x. If p, q, r are positive, find the minimum value of:
[x] x pqr

p + q q + r r + p
r + p + q

7.

3
999
Let n = 999
14K
24
3 . How many 9s are there in the decimal representation of n ?

2007 digits
3
n = 999
999
14K
24
3 n 9
2007

8.

Let x, y be nonnegative integers such that x + 2 y is a multiple of 5, x + y is a multiple of 3 and


2 x + y 99 . Find the minimum possible value of 7 x + 5 y .
xy x + 2 y 5 x + y 3 2 x + y 99
7 x + 5 y
2

9.

The three-digit number abc consists of three non-zero digits. The sum of the other five three-digit
numbers formed by rearranging a, b, c is 2017. Find abc .
abc abc
2017 abc

10. Find the sum of the greatest odd factor of each of the numbers 2007, 2008, , 4012.
200720084012
11. Let A1 , A2 , , A11 be 11 points on a straight line in order, where A1 A11 = 56 . Given that Ai Ai + 2 12
for i = 1, 2, , 9 and Aj Aj +3 17 for j = 1, 2, , 8, find A2 A7 .
A1 A2 A11 11 A1 A11 = 56 i = 1, 2, , 9
Ai Ai + 2 12 j = 1, 2, , 8 Aj Aj +3 17 A2 A7
12. In ABC, AB = 5 , BC = 1 and AC = 2. I is the incentre of ABC and the circumcircle of IBC cuts
AB at P. Find BP.
ABC AB = 5 BC = 1AC = 2I ABC IBC AB P
BP
13. Let x1 , x2 , x3 , x4 , x5 be nonnegative real numbers whose sum is 300. Let M be the maximum of
the four numbers x1 + x2 , x2 + x3 , x3 + x4 and x4 + x5 . Find the least possible value of M.
x1 x2 x3 x4 x5 300 M x1 + x2 x2 + x3
x3 + x4 x4 + x5 M
14. ABCD is a square with side length 9. Let P be a point on AB such that AP : PB = 7 : 2 . Using C as
centre and CB as radius, a quarter circle is drawn inside the square. The tangent from P meets the
circle at E and AD at Q. The segments CE and DB meet at K, while AK and PQ meet at M. Find the
length of AM.
ABCD 9 P AB AP : PB = 7 : 2 C
CB P E AD Q
CE DB K AK PQ M AM
15. ABCD is a rectangle with AB = 2 and BC = 1. A point P is randomly selected on CD. Find the
probability that APB is the largest among the three interior angles of PAB.
ABCD AB = 2BC = 1 CD P APB PAB

16. Let a, b, c be positive integers such that ab + bc ca = 0 and a c = 101 . Find b.


abc ab + bc ca = 0 a c = 101 b
17. A bag contains 15 balls, marked with the 15 numbers 20 , 21 , 22 , , 214 respectively. Each ball is
either red or blue, and there is at least one ball of each colour. Let a be the sum of the numbers on all
red balls, b be the sum of the numbers on all blue balls and d be the H.C.F. of a and b. Find the
greatest possible value of d.
15 20 21 22 214 15
a b
d a b d
3

18. Find the sum of all real roots to the equation 3 tan 2 x + 8 tan x + 3 = 0 in the range 0 < x < 2 .
3 tan 2 x + 8 tan x + 3 = 0 0 < x < 2
19. For 0 x 1 and positive integer n, let f 0 ( x) = |1 2 x | and f n ( x) = f 0 ( f n 1 ( x)) . How many
solutions are there to the equation f10 ( x) = x in the range 0 x 1 ?
0 x 1 n f 0 ( x) = |1 2 x | f n ( x) = f 0 ( f n 1 ( x)) f10 ( x) = x
0 x 1
20. Determine the number of ordered pairs (x, y) of positive integers satisfying the following equation:
(x, y)

x y + y x 2007 x 2007 y + 2007 xy = 2007


Section B (2 marks each)
2
21. How many nine-digit positive integers consist of nine pairwise distinct digits and are divisible by
4950?
4950
22. Let S be the set of points whose coordinates x, y and z are integers that satisfy 0 x 2 , 0 y 3
and 0 z 4 . Two distinct points are randomly chosen from S. Find the probability that the
midpoint of the two chosen points also belongs to S?
S xyz 0 x 2 0 y 3 0 z 4 S
S

23. If x is positive, find the minimum value of


x

x 4 + x 2 + 2 x + 1 + x 4 2 x3 + 5 x 2 4 x + 1
.
x

x 4 + x 2 + 2 x + 1 + x 4 2 x3 + 5 x 2 4 x + 1

1 1 1
1
, , , ,
respectively. Each ball
2 3 4
1000
is either red or blue, and the number of red balls is a positive even number. Let S denote the product
of the numbers on all red balls. Find the sum of all possible values of S.
1 1 1
1
999
999
2 3 4
1000
S S

24. A bag contains 999 balls, marked with the 999 numbers

25. Find the minimum value of | sin x + cos x + tan x + cot x + sec x + csc x | for any real number x.

x | sin x + cos x + tan x + cot x + sec x + csc x |


END OF PAPER
4

International Mathematical Olympiad


Preliminary Selection Contest 2007 Hong Kong

Outline of Solutions

Answers:
1.

5.

9.

667

52

425

2.

691

3.

88

4.

24

6.

7.

4013

8.

366

10. 4024036
85
22

13. 100

14.

17. 4681

18. 5

21. 576

22.

23
177

11. 29

12.

15.

16. 2550

3 1

52

19. 2048

20. 6

23.

24. (Cancelled)

10

25. 2 2 1

Solutions:

1.

We dont have to care about zeros as far as sum of digits is concerned, so we simply count the
number of occurrences of the non-zero digits. The digit 3 occurs 18 times (10 as tens digits in
30, 31, , 39 and 8 as unit digits in 3, 13, , 73). The same is true for the digits 4, 5. 6
and 7. A little modification shows that the digits 1 and 2 each occurs 19 times. Finally the
digit 8 occurs 11 times (8, 18, 28, , 78, 80, 81, 82) and the digit 9 occurs 8 times (9, 19,
29, , 79). Hence the answer is
(3 + 4 + 5 + 6 + 7) 18 + (1 + 2) 19 + 8 11 + 9 8 = 667 .

2.

Since 2007 7 (mod 1000), we need n for which 7 n 837 (mod 1000). Noting that
7 143 = 1001 1 (mod 1000), we multiply both sides of 7 n 837 (mod 1000) by 143 to get
7 n(143) 837(143) (mod 1000), or n 691 (mod 1000). Hence the answer is 691.

Alternative Solution. To find the smallest n for which 7 n 837 (mod 1000), we look for the
smallest multiple of 7 in the sequence 837, 1837, 2837, It turns out that 4837 is the first
term divisible by 7, and hence the answer is 4837 7 = 691 .

3.

Note that N is the mid-point of BC, so NB = NC

= 30. By Pythagoras Theorem, we have

DN = 502 302 = 40 .

50

60

Computing the area of ABCD in two ways, we


get

50

60 40 = 50 DM ,

which gives DM = 48. It follows that


DM + DN = 48 + 40 = 88 .

4.

We resort to the formula Distance = Speed Time. Here distance refers to the total length
of the tunnel and the truck. When the speed is reduced by 20% and the time taken is increased
by half, the distance must be increased by (1 20%) (1 + 50%) 1 = 20% . Hence, if we let the
6+ x 5
answer be x m, then we have
= , which gives x = 24 .
12 + x 6

5.

Using EF and BC as bases, let the heights of AEF


and EBC to be k times and 1 k times that of ABC

respectively. Let [XYZ] denote the area of XYZ.


Since AEF ~ ABC, we have [ AEF ] = k 2 ; since

EBC shares the same base as ABC, we have


[ EBC ] = 1 k . Hence we get k 2 = 1 k , and thus

k=

1 + 5
as k > 0 . It follows that
2

[ EFC ] = 1 k 2 (1 k ) = 5 2 .

F
C

6.

Note that for positive real number x, we have [ x] > x 1 and x +

1
2 . Hence
x

p + q q + r r + p p + q q + r r + p
r + p + q > r 1 + p 1 + q 1

p q q r r p
= + + + + + 3
q p r q p r
2+2+23
=3

p + q q + r r + p
Since
+
+
is an integer, it must be at least 4. Such minimum is
r p q
attainable, for example when p = 6 , q = 8 and r = 9 .

7.

Note that n = 102008 1 . Hence


n3 = (102008 1)3 = 106014 3 104016 + 3 102008 1 = 999
K3
99 7 000
K3
00 2999
K3
99
1
424
1
424
1
424
2007 digits

2007 digits

2008 digits

and so the answer is 2007 + 2008 = 4015 .

8.

Let x + 2 y = 5a and x + y = 3b . Solving for x, y in terms of a and b, we have x = 6b 5a and


y = 5a 3b . Also, 2 x + y = 9b 5a and 7 x + 5 y = 27b 10a . Hence the problem becomes

minimising 27b 10a over non-negative integers a, b satisfying 6b 5a 3b and


9b 5a 99 .

Clearly standard linear programming techniques will solve the problem. We present a purely
algebraic solution below. We first establish lower bounds on a and b: 9b 5a + 99 3b + 99
gives b 17 , while 5a 3b 3(17) gives a 11 . Next 9b 5a + 99 5(11) + 99 gives b 18 .
We consider two cases.

If b = 18 , then we have 5a 9b 99 = 9(18) 99 = 63 , which gives a 12 and hence


27b 10a 27(18) 10(12) = 366 . We check that (a, b) = (12,18) satisfies all conditions

with 27b 10a = 366 .

If b > 18 , then 27b 10a = 9b + 2(9b 5a ) 9(19) + 2(99) = 369 .

Combining the two cases, we see that the answer is 366.

9.

We have abc + 2017 = abc + acb + bac + bca + cab + cba = 222(a + b + c) . Since abc is
between 111 and 999, 222(a + b + c) is between 2128 and 3016, so that a + b + c must be 10,
11, 12 or 13.
If a + b + c = 13 , then abc = 222 13 2017 = 869 , leading to the contradiction 13 = 8 + 6 + 9 .
Similar contradictions arise if a + b + c equals 10 or 12. Finally, a + b + c = 11 , then

abc = 222 11 2017 = 425 and we check that 4 + 2 + 5 is indeed equal to 11. Hence the
answer is 425.

10. For a positive integer n, let p (n) denote its greatest odd divisor. Then n = 2k p(n) for some
nonnegative integer k. Hence if p (r ) = p ( s ) for r s , then one of r and s is at least twice the
other. Because no number from 2007, 2008, , 4012 is at least twice another, p (2007) ,
p (2008) , , p (4012) are 2006 distinct odd positive integers. Note further that each of them

must be one of the 2006 odd numbers 1, 3, 5, , 4011. It follows that they are precisely 1, 3,
5, , 4011 up to permutation, and so the answer is
1 + 3 + 5 + L + 4011 = 2006 2 = 4024036 .

11. We have 56 = A1 A11 = A1 A2 + A2 A5 + A5 A8 + A8 A11 A1 A2 + 17 + 17 + 17 , so that A1 A2 5 . On the


other hand we have A1 A2 = A1 A4 A2 A4 17 12 = 5 , so we must have A1 A2 = 5 . Similarly
A10 A11 = 5 .
Next we have A2 A7 = A1 A4 + A4 A7 A1 A2 17 + 17 5 = 29 on one hand, and on the other hand
we have A2 A7 = A1 A11 A1 A2 A7 A10 A10 A11 56 5 17 5 = 29 . It follows that A2 A7 = 29 .

Remark. The scenario in the question is indeed possible. One example is Ai Ai +1 = 5 , 7, 5, 5, 7,


5, 5, 7, 5, 5 for i = 1, 2, , 10.

12. Let A, B, C denote the corresponding interior angles


of ABC. Noting that B, P, I, C are concyclic, we

have
APC = 180 CPB = 180 CIB =
ACP = 180 A APC =

B C
+
2 2

B C
+
2 2

I
A

B
P

It follows that AP = AC = 2 and hence BP = 5 2 .

13. We have 3M ( x1 + x2 ) + ( x2 + x3 ) + ( x4 + x5 ) = 300 + x2 300 , so M 100 . This minimum is


attainable when x1 = x3 = x5 = 100 and x2 = x4 = 0 . Hence the answer is 100.
4

14. By tangent properties, we have PE = PB = 2 and we


may let QD = QE = x . Then QA = 9 x and PQ = x + 2 .

Applying Pythagoras Theorem in APQ, we have


63
(9 x) 2 + 7 2 = ( x + 2)2 , which gives x = .
11

Next observe that CEPB is a cyclic quadrilateral, so we


have MPA = KCB = KAB, which means
MP = MA . Likewise we have MQ = MA and hence
1
1 63
85
AM = PQ = + 2 =
.
22
2
2 11

M
A

E
B
P

15. Note that the requirement of the question is satisfied


if and only if both AP and BP are smaller than 2, or
equivalently, both DP and CP are smaller than 3 .

Hence if DC is part of the number line with D


representing 0 and C representing 2, then P must be
between 2 3 and 3 in order to satisfy the

B
1

condition, and the probability for this to happen is

3 (2 3)
= 3 1 .
2

16. Let d be the H.C.F. of a and c. Then we may write a = dx and c = dy where x, y are integers
with an H.C.F. of 1. Then we have
b=

ac
dxy
=
and d ( x y ) = 101 .
a+c x+ y

Since x and y are relatively prime, the first equation shows that x + y must divide d. Hence
d > 1 and we know from the second equation that d = 101 and x y = 1 . Since 101 is prime,
101 50 51
we must then have x + y = 101 and thus x = 51 , y = 50 and b =
= 2550 .
101

17. The sum of the numbers on all balls is a + b = 20 + 21 + L + 214 = 215 1 = 32767 . Since both a
and b are divisible by d, so is a + b = 32767 . It is easy to see that d cannot be equal to 32767.
Since the prime factorisation of 32767 is 7 31151 , the next largest possible candidate for d
is 31151 = 4681 . This value of d is attainable; to see this, note that the binary representation
of 4681 is 1001001001001, so that if balls numbered 2k are red if k is divisible by 3 and blue
if otherwise, then we will have a = 4681 , b = 4681 6 and d = 4681 .

18. Being a quadratic equation in tan x with discriminant 82 4(3)(3) > 0 , we see that there are
two possible values of tan x (say tan x1 and tan x2 ) and hence four possible values of x in the
8
3
range 0 < x < 2 . Since tan x1 + tan x2 = and tan x1 tan x2 = = 1 , both tan x1 and tan x2
3
3
are negative, so we have two possible values of x in the second quadrant and two in the fourth
quadrant.
Consider x1 , x2 in the second quadrant satisfying the equation. From tan x1 tan x2 = 1 , we get
3
3

x1 . As
x1 is also in the second quadrant, we must have
tan x2 = cot x1 = tan
2

2
3
x2 =
x1 . On the other hand, it is easy to see that the two possible values of x in the fourth
2
quadrant are simply x1 + and x2 + , as tan( x + ) = tan x for all x (except at points where

tan x is undefined). It follows that the answer is


3

x1 +
x1 + ( x1 + ) +
x1 + = 5 .
2

19. In an iterative manner, we can work out the graphs of f 0 , f1 and f 2 (together with the graph
of y = x in dotted line) in the range 0 x 1 , which explain everything:

Based on the pattern, it is easy to see (and work out an inductive proof) that the graph of f10
consist of 210 = 1024 copies of V, and hence 1024 2 = 2048 intersections with the line
y = x . This gives 2048 as the answer.

20. Rewrite the equation as x y + y x + 2007 xy = 2007 x + 2007 y + 2007 , which becomes
xy ( x + y + 2007 ) = 2007 ( x + y + 2007 ) upon factorisation. Since the common

factor

x + y + 2007 is positive, we must have xy = 2007 = 32 223 . As 2007 has

(2 + 1)(1 + 1) = 6 positive factors, the answer is 6.

21. Let n be such an integer. Note that n is divisible by 4950 if and only if it is divisible by each of
50, 9 and 11. As no two digits of n are the same, the last two digits of n must be 50. Note also
that n misses exactly one digit from 0 to 9 with every other digit occurring exactly once. As n
is divisible by 9 and the unit digit of n is 0, it is easy to check that the digit missing must be 9,
and hence the first 7 digits of n must be some permutation of 1, 2, 3, 4, 6, 7, 8.
Let n = ABCDEFG50 . Since n is divisible by 11, we have A + C + E + G B + D + F + 5 (mod
11). Also, the sum of A to G is 1 + 2 + 3 + 4 + 6 + 7 + 8 = 31 . It is easy to check that the only
possibility is A + C + E + G = 18 and B + D + F = 13 .
Now there are 4 ways to choose 3 digits from {1, 2, 3, 4, 6, 7, 8} to make up a sum of 13,
namely, {8, 4, 1}, {8, 3, 2}, {7, 4, 2} and {6, 4, 3}. Each of these gives rise to 3! choices for
the ordered triple (B, D, F), and another 4! choices for the ordered quadruple (A, C, E, G). It
follows that the answer is 4 3! 4! = 576 .

22. Note that S contains 3 4 5 = 60 elements. Divide the points of S into 8 categories according
to the parity of each coordinate. For instance, EEO refers to the points of S whose x- and ycoordinates are even and whose z-coordinate is odd (and similarly for the other combinations
of E and O). There are 2 2 2 = 8 points in the EEO category. In a similar manner we can
work out the size of the other categories:
Category

EEE

EEO

EOE

EOO

OEE

OEO

OOE

OOO

Size

12

12

Note further that two points form a favourable outcome if and only if they are from the same
category. It follows that the required probability is
C212 + C28 + C212 + C28 + C26 + C24 + C26 + C24 23
=
.
177
C260

1
23. Let A, B, P be the points (0, 1), (1, 2) and x, respectively. Then
x
x 4 + x 2 + 2 x + 1 + x 4 2 x3 + 5 x 2 4 x + 1
2 1
4 1
= x2 + 1 + + 2 + x2 2 x + 5 + 2
x
x x
x x
2

1
1

= x + + 1 + ( x 1)2 + 2
x
x

= PA + PB

AB
= (0 1) 2 + (1 2)2
= 10
Equality is possible if A, P, B are collinear, i.e. P is the intersection of the curve xy = 1 with
the straight line AB in the first quadrant. One can find by simple computation that x =

1 + 13
6

in this case. Hence the answer is 10 .

24. (This question was cancelled in the live paper. We present below a solution to find the sum of
all possible values of S counting multiplicities, i.e. if a possible value of S is attained in two
different situations, we shall sum that value twice.)
Lets also take into consideration the case where the number of red balls is a positive odd
number, and in this case let T be the product of the numbers on all red balls. Let G be the sum
of all possible values of S (counting multiplicities) and H be the sum of all possible values of T
(counting multiplicities). Note that
1
3 4
1001
1001
999
1 1
G + H = 1 + 1 + L 1 +
1 =
1 =
1 = L
2 3
1000
2
2
2 3 1000
since each possible value of S or T arises from a choice of whether each ball is red or blue, and
the term 1 in the end eliminates the case where all balls are blue which is not allowed by the
question. In a similar manner, we have
1
1 2
999
1
999
1 1
G H = 1 1 L 1
1 =
1 =
1 = L
2 3
1000
1000
1000
2 3 1000
since each possible value of T (corresponding to a choice of an odd number of red balls) is
evaluated 1 time in the above product while each possible value of S is evaluated +1 time. It
follows that
1 999 999 1 499500 999 498501

G=
.
=
=
2 2 1000 2
1000
2000

25. Let a = sin x and b = cos x . Then a 2 + b 2 = 1 and we want to minimise


a+b+

a b 1 1
1+ a + b
+ + + = a+b+
.
b a b a
ab

( a + b) 2 ( a 2 + b 2 ) c 2 1
If we set c = a + b , then ab =
=
and so the above quantity becomes
2
2
c+

2(1 + c)
2
2
= c+
= (c 1) +
+1 .
2
c 1
c 1
c 1

Let f (c) denote this quantity. As c = 2 sin x + , c may take values between 2 and

4
2

2
2
2 . On the other hand, for positive real number r we have r + = r
+2 2 2 2 ,
r
r
with equality when

r=

2
, i.e. r = 2 . Hence
r

if c > 1 , then f (c) 2 2 + 1 ;

if c < 1 , then f (c) = 1 (1 c) +


1 2 2 = 2 2 1 .
1 c

Equality in the second case holds if 1 c = 2 , i.e. if c = 1 2 , which is possible since

2 < 1 2 < 2 . It follows that the answer is 2 2 1 .

International Mathematical Olympiad


Preliminary Selection Contest Hong Kong 2006

Outline of Solutions

Answers:

6 2
2

1.

666

2.

1475

3.

60

4.

5.

4 3
3

6.

7.

27
23

8.

9.

30000

10. 240

11. 4020

12.

14. 3456

15. 51

16. 30

18. 281

19. 2310

20. 13

13. 1505

17.

16 17
17

32

Solutions:

1.

Let the two numbers be x and y. Then we have x + y = xy 2006 . Upon factorisation, we have
( x 1)( y 1) = 2007 .
Since 2007 = 32 223 , it has 6 positive factors, namely, 1, 3, 9, 223, 669 and 2007. As one of x
and y (say, x) is a perfect square, x 1 must be 1 less than a perfect square. Among 1, 3, 9, 223,
669 and 2007, only 3 has this property ( 3 = 22 1 ). Therefore we should take x 1 = 3 and
y 1 = 669 , i.e. x = 4 and y = 670 . It follows that the answer is 670 4 = 666 .

2.

2006n
20062 + 2006n
20062
20062
=

= 2006
.
Note that
2006 + n
2006 + n
2006 + n
2006 + n
Hence, in order for 2006n to be a multiple of 2006 + n , 2006 + n must be a factor of 20062 .
As 1 n 2005 , we have 2007 2006 + n 4011 . As 20062 = 22 17 2 592 , the only factor of
2006 + n between 2007 and 4011 is 592 = 3481 . Hence we must have 2006 + n = 3481 , or
n = 1475 .

3.

Let x be the normal walking speed of Peter and u be the speed of the escalator. Since speed is
u + x 30
inversely proportional to time taken, we have
=
, which gives u = 2 x . Let the
u + 2 x 40
u
40
40(u + x) 40(2 x + x)
answer be t seconds. Then we have
, so that t =
=
= 60 .
=
u+x t
u
2x

4.

We claim that AC = 1 . If AC > 1 , then by


considering DAC we see that DAC must be
smaller than 60 (recall that in a triangle, the side
opposite a larger angle is longer), while CAB must
be smaller than 75 by considering ABC. It leads to
the contradiction DAB < 135 . In a similar way, we
can get a contradiction when AC < 1 , thereby

135

75
1

establishing the claim. Now CAB is isosceles with


CA = CB = 1 and base angles 75. Therefore

AB = 2 cos 75
= 2 cos(45 + 30)
= 2(cos 45 cos 30 sin 45 sin 30)
=

5.

6 2
2

We claim that X is the circumcentre of ABC.


Assuming the claim, we know from the sine
4
4 3
law that
.
= 2 AX , so that AX =
sin 60
3
It remains to prove the claim. Since X is on the
perpendicular bisector of BC, we have
XB = XC . It therefore suffices to show that
BXC = 2BAC = 120 . But this is clear,
because
BXC = BIC
= 180 IBC ICB
ABC ACB

2
2
180 BAC
= 180
2
= 120
= 180

60
X
B

I
C

6.

Let x = cos where 0 . Then x + 1 x 2 = cos + sin = 2 cos . Since the


4

maximum value of the cosine function is 1, the maximum value of x + 1 x 2 is


equality is attained when =

, i.e. when x =

2 , and

2
.
2

Remark. A solution using calculus obviously exists.

7.

Note that Sn and Tn , both being the sum of the first n terms of an arithmetic sequence, are
both quadratic polynomials with constant term 0. Hence we may assume Sn = An 2 + Bn and

Tn = Cn 2 + Dn . Consequently, an = Sn Sn 1 = 2 An + ( B A) . Similarly bn = 2Cn + ( D C ) .


Plugging these into the given equation, we have
2 An + ( B A) 2n 1
,
=
2Cn + ( D C ) 3n + 1
which gives 6 An 2 + (3B A)n + ( B A) = 4Cn 2 + (2 D 4C )n + (C D) . As this is true for all
3A
5A
n, we equate the coefficients to solve for A, B, C, D. We get B = 0 , C =
and D =
.
2
2
Hence we have
S9
A(9) 2 + (0)(9)
27
=
=
.
T6 3 A 2 5 A
23

(6) +
(6)
2
2

Remark. It is easy to guess the answer if one assumes an = 2n 1 and bn = 3n + 1 .

8.

Note that the point G is not useful except that it


tells us that the chord AB subtends an angle of
48 2 = 96 at the centre of the circle (which
we call O) so that EOF = 96 3 = 32 . We

claim that HA // OE and HB // OF. Assuming


the claim, then AHB = EOF = 32 and
hence the answer would be 32.
O

It remains to prove the claim. To show that HA


// OE, we shall prove that AF is perpendicular
to both HA and OE. That AF OE is clear,
because E is the mid-point of arc AF. Now
produce HA and FE to meet at K. First note that
AE = EF , and AC = CD implies KE = EF . In
other words, the circle with diameter KF passes
through A, so AF HA. This shows HA // OE,
3

A
K

D
B

and by the same argument we have HB // OF,


thereby establishing the claim.

9.

Suppose Tommy invests $a, $b and $c on funds A, B and C respectively. Clearly, as long as a
positive net profit can be guaranteed, we must take a + b + c = 90000 . Then we want to
maximise
n = min{3a, 4b, 6c} 90000

subject to the above conditions.


For maximality, we must have 3a = 4b = 6c . This is because when 3a, 4b and 6c are not all
equal, say, if 3a < 4b and 3a 6c , then Tommy can shift some investments from fund B to
fund A (or some to fund C as well, if necessary). In this way the value of 3a increases while the
values of 4b and 6c will remain greater than or equal to 3a when the shift is sufficiently small.
Therefore n increases, i.e. the value of n cannot be maximum when 3a, 4b and 6c are not all
equal.
1 1 1
Hence, in order to attain maximality, we want a : b : c = : : = 4 : 3 : 2 . In this case, the
3 4 6
maximum value of n is
3a 90000 = 3 90000

4
90000 = 30000 .
4+3+ 2

10. We first choose 4 boxes to place balls A, B, C, D. There are 10 possible combinations, namely,
1234, 1245, 1256, 1267, 2345, 2356, 2367, 3456, 3467 and 4567. Once the 4 boxes are chosen,
there are 4 ways to place ball A, and then 1 way to place ball B (once ball A is placed there is
only one possible position for ball B in order that balls C and D can be placed in boxes with
consecutive numbers), and finally 2 ways to place balls C and D. There are 3 remaining boxes
in which ball E can be placed. Hence the answer is 10 4 2 3 = 240 .

11. Let O be the common centre of the two


circles, and let P' be a point such that
APP' is equilateral and that B and P' are
on different sides of AP. Observe that
PA = PP ' and PB = CP ' (as BAP

A
P'
O

CAP'), so PCP' is the desired triangle.


Let R = 2007 , r = 2006 , POB = x and
POC = y. Then we have x + y = 120
x y
= 60 y . Now
and
2

C
P

[ PCP '] = [ APP '] [ APC ] [ AP ' C ]


= [ APP '] [ APC ] [ APB]
= [ APP '] [ ABPC ]
= [ APP '] [OAB] [OAC ] [OBP] [OCP]
=
=
=
=
=

3 2 2
3 2 2 1
R + r 2 Rr cos(120 + y )
(r + r ) Rr (sin x + sin y )
4
4
2
x+ y
x y
3 2 2
3
1
Rr cos(60 y ) Rr 2sin
(R r ) +
cos
4
2
2
2
2
3
3
3 2 2
Rr cos(60 y )
Rr cos(60 y )
(R r ) +
2
2
4
3 2 2
(R r )
4
4013 3
4

Hence a + b + c = 4013 + 3 + 4 = 4020 .


Remark. It would be easy to guess the answer if one considers the special case where A, O, P

are collinear.

12. Each face of the octahedron is an equilateral triangle of side length 6, the altitude of which is
6sin 60 = 3 3 . Consequently, half of the octahedron (i.e. the square pyramid in which every
edge has length 6) has altitude

(3 3 )

32 = 3 2 , i.e. the volume of the octahedron is

1
2 62 3 2 = 72 2 .
3
On the other hand, if we let r be the radius of the inscribed sphere of the regular octahedron
and A be the surface area of the octahedron, then the volume of the octahedron would also be
1
1
1
equal to Ar . Now A = 8 6 3 3 = 72 3 . Hence we have 72 3 r = 72 2 , which
2
3
3
gives r = 6 . This is also the greatest possible radius of a sphere which can be placed inside
the tetrahedron.

13. Let f (n) =

n2
. For n 1003 , we have
2006
f (n) f (n 1) =

n2
(n 1) 2 2n 1

=
<1
2006 2006
2006

10032
Since f (1) = 0 and f (1003) =
= 501.5 , the sequence contains every integer from 0 to
2006
501 inclusive. On the other hand, when n > 1003 ,
5

f (n) f (n 1) =

2n 1
>1.
2006

Moreover, since
f (1004) =

10042 (1003 + 1) 2 10032 2 1003


1
1
=
=
+
+
= 501.5 + 1 +
> 502 ,
2006
2006
2006
2006
2006
2006

20062
10042 10052
,
,
,
we see that

are all distinct integers, all greater than 501.


2006
2006 2006

Therefore, the number of different integers in the sequence is 502 + 1003 = 1505 .

14. Such positive integers are all divisible by 9 since each has sum of digits 0 + 1 + 2 +

+ 9 = 45 .

Hence we can change the number 11111 in the question to 99999 without affecting the
answer. We denote such a 10-digit positive integer by ABCDEFGHIJ . For this number to be
divisible by 99999, a necessary and sufficient condition is that ABCDE + FGHIJ should be
divisible by 99999 (see Remark). Hence ABCDE + FGHIJ must be exactly 99999. In other
words, each of A + F , B + G , C + H , D + I and E + J must be equal to 9, i.e. we need only
choose A, B, C, D, E. There are 9 choices for A as it cannot be zero (once A is chosen, F is also
fixed). There are 8 choices for B as it cannot be equal to A and F (once B is chosen, G is also
fixed). Similarly, there are 6, 4, 2 choices for C, D, E respectively, and at the same time H, I, J
are fixed. Hence the answer is 9 8 6 4 2 = 3456 .
Remark. The divisibility test for 99999 is to put the digits into groups of 5 from the right, and

then add up the numbers (as positive integers) in each group. The original number is divisible
by 99999 if and only if the sum obtained is divisible by 99999. In the 10-digit case, this can be
explained by the following congruence equation:
ABCDEFGHIJ = 100000 ABCDE + FGHIJ
1 ABCDE + FGHIJ
= ABCDE + FGHIJ

(mod 99999)

15. Since ABCDEF is convex, it must be either C


or F which has y-coordinate 4. Suppose C has
y-coordinate 4. Then, since AB = BC , either C
must be either (2b, 4) or (0, 4), both of which
are not allowed (as the former implies that A, B,
C are collinear while the latter implies that
ABCDEF is concave). Hence F has ycoordinate 4. Furthermore, since CD // AF, the
y-coordinate of D exceeds that of C by 4. It
follows that the y-coordinates of D and C are 10
6

y
D (d, 10)
E (e, 8)
C (c, 6)
F (f, 4)
B (b, 2)
A (0, 0)

and 6 respectively, and thus the y-coordinate of


E is 8.
Let C = (c, 6), D = (d, 10), E = (e, 8) and F = (f, 4). From the fact that BC = CD , we know that
b = d , i.e. BD is vertical. As AB and ED are equal and parallel, ABDE is a parallelogram, and
hence AE is also vertical , so e = 0 . Furthermore, AEF and BCD are congruent by the SSS
condition. Note also that f < 0 by the convexity condition.
1
Now the area of ABCDEF is equal to 2 [ AFE ] + [ ABDE ] = 2 8 ( f ) + 8b = 8(b f ) , so
2
we must find the values of b and f. Let x be the side length of the hexagon. Computing the
lengths of AB and AF respectively, we have
x 2 = b 2 + 4 = f 2 + 16 .
On the other hand, since FAB = 120, applying cosine law in FAB gives
(b f ) 2 + (2 4) 2 = x 2 + x 2 2( x)( x) cos120 = 3 x 2 .
Solving, we get b =

10
8
and f =
. Hence the area of ABCDEF is 8(b f ) = 48 3 , and
3
3

hence the answer is 48 + 3 = 51 .

16. Note that y < 21 , and the triangle inequality in ADE


implies y + y > 21 y , or y > 7 . It follows that y is

between 8 and 20 inclusive.

cos A =

21 y

Now, applying cosine law in ADE and ABC, we get


y 2 + (21 y ) 2 y 2
2 y (21 y )

D
y

E
y

33 y
and
332 + 212 x 2
cos A =
2(33)(21)

respectively. Equating and simplifying, we get


y (2223 x 2 ) = 14553 = 33 7 2 11 .

Since 8 y 20 , the only possible values of y are 9


and 11. These correspond to x 2 = 606 and 900
respectively. As the former is not a perfect square, we
take the latter which corresponds to x = 30 .

17. Suppose that in ABC, BC = 3 , AC = 4 and AB = 5 .


Of course ACB = 90. Without loss of generality,
we fix A at the origin and assume that ABC lies in
the first quadrant, and that the square to enclose
ABC has sides parallel to the coordinate axes and
has A as a vertex. Let ADEF denote the smallest
rectangle with sides parallel to the coordinate axes
that can enclose ABC. Clearly, when ADEF is a
square, that is the smallest square that can enclose
ABC.

5
4
A

C
D

Now suppose ADEF is indeed a square with C on DE and B on EF as shown. Let


CAD = BCE = . We have AD = 4 cos and ED = 3cos + 4sin . Equating, we get
1
4
16 17
cos = 4sin and hence tan = . Thus AD = 4 cos = 4
=
, which is the
4
17
17
desired smallest possible side length.

18. We use (m, n) to denote the H.C.F. of m and n. Using the facts that (m, n) = (m, n + km) for any
integer k and (m, n) = (2m, n) if n is odd, we have
g (n) = ( f (n), f (n + 1)) = (70 + n 2 , 70 + (n + 1) 2 )
= (70 + n 2 , 2n + 1) = (140 + 2n 2 , 2n + 1)
= (140 n, 2n + 1) = (280 2n, 2n + 1)
= (281, 2n + 1)
281

Hence the largest possible value of g (n) is 281. Finally, 281 is indeed attainable because
f (140) = 70 + 1402 = 281 70 while f (141) = 70 + 1412 = 281 71 . It follows that the answer is
281.

M
. Then each mi is a positive integer
ai
and m1 > m2 > > m11 . Furthermore, m1 , m2 , , m11 form an arithmetic sequence and M is
also the L.C.M. of m1 , m2 , , m11 . Write b11 = b and bi bi +1 = d , where b, d are positive

19. Let M be the L.C.M. of a1 , a2 , , a11 , and set mi =

integers. Note that b and d have to be relatively prime by the definition of M. We want to
minimise
a1 =

lcm(b, b + d , , b + 10d )
.
b + 10d

When b = 2 and d = 1 , we find that a1 = 2310 . We prove below that a1 cannot be any smaller,
so that the answer would be 2310. Indeed, if d 2 , then
8

a1

lcm(b + 5d , b + 6d , , b + 10d ) (b + 5d )(b + 6d ) (b + 10d ) 5 6 7 8 9 4


d > 2310 .
=

b + 10d
2 2 2 3 (b + 10d )
2 2 2 23

Hence we may assume d = 1 . In this case


a1

lcm(b + 5, b + 6, , b + 10) (b + 5)(b + 6) (b + 10) (b + 5)(b + 6)


=

2 2 2 2 3 (b + 10)
48
b + 10

(b + 9)

If b 4 , we see that a1 > 2310 . So it remains to check the cases b = 1, 2, 3. Among these, we
find that b = 2 gives the minimum value a1 = 2310 .

20. Let f (n) be the number of the card received by child n. Consider the sequence
1, f (1) , f ( f (1)) , f ( f ( f (1))) ,
Note that the term 2006 must eventually occur and the sequence can no longer be continued as
f (2006) is undefined. Furthermore, if we let S be the set of integers from 1 to 2006 which
have not occurred in the above sequence, then we see that f ( S ) = S . It easily follows that
every child in S must receive the card with the same number as his own.
It therefore remains to count the number of such sequences, starting from 1 and ending in
2006, and each term (except for the first one) is a multiple of the previous term. Note that
2006 = 2 17 59 , and every positive factor of 2006 is of the form 2a17b59c and can be
denoted by the three-tuple (a, b, c) where each of a, b, c is 0 or 1. The problem is thus reduced
to counting the number of sequences from (0,0,0) to (1,1,1) for which each coordinate must be
greater than or equal to the corresponding coordinate in the previous term.
If we increase the three coordinates one by one, we get 3! = 6 such sequences. If we first
increase one of the coordinates and then increase the other two at once, we get C13 = 3 such
sequences. If we first increase two of the coordinates at once and then increase the remaining
one, we get C23 = 3 such sequences. Finally, we get 1 such sequence if we jump from (0,0,0) to
(1,1,1) directly. It follows that the answer is 6 + 3 + 3 + 1 = 13 .

International Mathematical Olympiad


Preliminary Selection Contest 2005 Hong Kong

Outline of Solutions

Answers:
1.

2.

35
12

3.

5050
10101

4.

165

5.

516

6.

957

7.

72

8.

9.

9 3
32

10. 6

11. 340

13. 546

14. 333

15.

17. 4659

18.

5 1
2

19.

14
4

12.

21
3

16. 101

4022030
3

20.

2005 2006
4012

Solutions:

1.

Since 2005 is odd, all ai s must be odd. Since the odd ai s add up to 2005, n must be odd as
2005
well. Consider the case n = 3 with a1 a2 a3 . Then a1
and this forces a1 = 2005 by
3
considering the factors of 2005. Then we must have a2 + a3 = 0 and a2 a3 = 1 , which means
a2 2 + 1 = 0 and hence leads to no solution. Finally, we see that n = 5 is possible since
2005 +1 + ( 1) +1 + ( 1) = 2005 1 ( 1) 1 ( 1) = 2005 .

Hence the answer is 5.

2.

Produce AD to G and BE to H such that AC // BG


and BC // AH . Let BD = 2 x , DC = 3 x , AE = 3 y ,
EC = 4 y . Since ACD ~ GBD ,
we have
AC AD CD 3
14 y
=
=
=
. Hence BG =
and
BG DG DB 2
3
3
AD : DG = 3 : 2 , i.e. AD = AG . On the other hand,
5
1

H
3y
E
F

2x

D 3x

4y
C

since AEF ~ GBF ,

AF FE AE
3y
9
=
=
=
= .
14
y
FG BF BG
14
3

BF 14
9
3
9
24
=
and AF = FG . If follows that FD = AD AF = AG FG =
FG .
FE 9
23
5
23
115
9
FG
AF
15
AF BF 15 14 35
23
Thus
=
=
and hence

= =
.
24
FD
FD FE 8 9 12
FG 8
115

Thus

3.

Observe that 1 + k 2 + k 4 = (1 + k 2 ) 2 k 2 = (1 k + k 2 )(1 + k + k 2 ) . Let


k
1+ k + k
2

Solving, we have A =

A
B
+
.
k ( k 1) + 1 k( k + 1) + 1

1
1
and B = . It follows that
2
2

1
2
3
100
+
+
+L+
2
4
2
4
2
4
1 +1 + 1 1+ 2 + 2 1+ 3 + 3
1 + 1002 + 1004
1 1
1 1
1
1
1

+
+L +

2 0 1+ 1 1 2 + 1 1 2 + 1 2 3 + 1
99 100 + 1 100 101 + 1
1
1
1

2 10101
5050
=
10101
=

4.

Let the length and width of the banner be x m and y m respectively. According to the question,
x, y are positive integers with 330 x + 450 y 10000 . Thus the area of the banner is
(330 x )(450 y )
1
1
330 x + 450 y
10000
xy =

< 169 m2

(330)(450)
(330)(450)
2
(330)(450) 2

by the AM-GM inequality. Note that we must have x 30 and y 22 .


If the area is 168 m2 , then we have the possibilities (x, y) = (12, 14); (14, 12) and (24, 7), yet
none of them satisfies 330 x + 450 y 10000 . Since 167 is prime and the only factors of 166 are
1, 2, 83 and 166, we easily see that the area cannot be 167 m2 nor 166 m2 either.
Finally, we see that x = 15 and y = 11 satisfies all conditions and give an area of 165 m2 . Thus
the maximum area of the banner is 165 m2 .

5.

There are 16 ordered pairs (x, y) of integers satisfying 1 x 4 and 1 y 4 . Thus C316 = 560
triangles can be formed. We must, however, delete those degenerate triangles, i.e. sets of three
2

points which are collinear. There are 4 horizontal lines of 4 points and 4 vertic al lines of 4
points. These together produce (4 + 4) C34 = 32 degenerate triangles. Also, the points (1, 1);
(2, 2); (3, 3) and (4, 4) are collinear, leading to C34 = 4 degenerate triangles, and so are the
points (1, 4); (2, 3); (3, 2) and (4, 1). Finally, we also need to delete the 4 degenerate triangles
{(1, 2); (2, 3); (3, 4)}, {(2, 1); (3, 2); (4, 3)}, {(1, 3); (2, 2); (3, 1)} and {(2, 4); (3, 3); (4, 2)}. It
follows that the answer is 560 32 4 4 4 = 516 .

6.

Note that

m+4
m2 + 7
is
in
lowest
term
if
and
only
of
is in lowest term. Since
m2 + 7
m+4

m2 + 7 m2 16
23
23
=
+
= m4+
,
m+4
m+4
m+ 4
m+4
the fraction is in lowest term except when m + 4 is a multiple of 23. Since m may be equal to
1, 2, , 1000, we shall count the number of multiples of 23 from 5 to 1004. The first one is
23 = 23 1 and the last one is 989 = 23 43 . Hence there are 43 such multiples. It follows that
the answer is 1000 43 = 957 .

7.

As shown in the figure, let the medians AD, BE and


CF of ABC meet at the centroid G. Recall that the
medians divide ABC into 6 smaller triangles of
equal area and that the centroid divides each median
in the ratio 2 : 1.

A
F
B

2
2
Let AD = 9 and BE = 12 . Then AG = 9 = 6 and BG = 12 = 8 . Hence the area of
3
3
ABG is
1
1
AG BG sin AGB = 6 8 sin AGB = 24sin AGB 24 ,
2
2

where equality is possible when AGB = 90 . Since the area of ABG is

ABC , the largest possible area of ABC is 24 3 = 72 .

8.

We have
2 xy 3 x 5 y = k
3
5
k
xy x y =
2
2
2
5
3 k 15

x y = +
2
2 2 4

(2x 5)(2 y 3) = 2k +15


3

1
the area of
3

When k is a positive integer, the number of positive integral solutions to the above equation is
precisely the number of positive divisors of 2 k +15 . For this number to be odd, 2 k +15 has to
be a perfect square. The smallest such k is 5. Indeed, we can check that when k = 5, the original
equation has 3 solutions, namely, (3, 14); (5, 4) and (15, 2).

9.

1
3
, AC =
, QAT = 90, QCP =
2
2
RBP is a straight line. The n

Note that BC =
150

and
3
[ AQT ] =
AT = [ ART ] implies TQ = TR. Thus
4

1
P
[ PRT ] = [ PQT ] = [ PQR ]
2
1
= ([ ABC] + [ ABR ] + [ BCP] +[ ACQ] + [ CPQ] [ AQR] )
2
1 3
3
3 3 3
3
3
=
+
+
+
+

2 8
4 16
16 16
8
9 3
=
32

B
T
A

10. Setting x = y = z = 1 , we have | a + b + c | = 1 .


Setting x = 1 and y = z = 0 , we have | a | + | b | + | c | = 1 .
Setting (x, y, z) = (1, 1, 0), we have | a b | + | b c | + | c a | = 2 .
Since | a + b + c | = | a | + | b | + | c | , a, b, c must be of the same sign (unless some of them is/are
equal to 0). But

2 = | a b | + | b c | + | c a | (| a | + | b | ) + (| b | + | c |) + ( | c | + | a | ) = 2 (| a | + | b | + | c | ) = 2 .
The equality | a b | = | a | + | b | holds only if a and b are of opposite signs (unless some of
them is/are equal to 0). Similarly, b and c have opposite signs, and c and a have opposite signs.
Yet a, b, c are of the same sign. Therefore two of a, b, c must be 0, and the other may be 1 or
1 . Hence there are 6 possibilities for (a, b, c), namely, ( 1 , 0, 0); (0, 1 , 0) and (0, 0, 1 ).

11. When n dice are thrown, the smallest possible sum obtained is n and the greatest possible sum
obtained is 6n. The probabilities of obtaining these sums are symmetric about the middle,
7n
namely,
. In other words, the probabilities of obtaining a sum of S and obtaining a sum of
2
7n S are the same. (To see this, we may consider the symmetry 1 6 , 2 5 , 3 4 .)
Furthermore, the probability for obtaining the different possible sums increases for sums from
4

7n
7n
and decreases for sums from
to 6n. Thus the same probability occurs for at most
2
2
two possible sums. From the above discussions, we know that S is either equal to 2005 or

n to

7 n 2005 . Therefore, to minimise S we should minimise n.


Since 2005 = 6 334 + 1 , n is at least 335. Indeed, when n is 335, S may be equal to
7 n 2005 = 7(335) 2005 = 340 . Since 340 is smaller than 2005, we conclude that the
smallest possible value of S is 340.

12. Let I and J be the centres of ABCD and EFGH


respectively, with the orientation as shown. Let AB
meet EH at N, BC meet GH at M with HM = x and
HN = y . Let also K be a point such that IK // AB and
1 1
JK // EH . Note that IK = + x = 1 x , and
2 2
1
similarly JK = 1 y . Since xy = , we have
16
IJ 2 = IK 2 + JK 2

E
J
B
N
y
H x M

I
D

= (1 x) 2 + (1 y ) 2

G
K

= x 2 + y 2 2( x + y ) + 2
= x 2 + y 2 + 2 xy 2 xy 2( x + y) + 2
1
= ( x + y )2 2 2( x + y ) + 2
16
15
2
= ( x + y ) 2( x + y ) +
8
7
= ( x + y 1)2 +
8

Hence the minimum value of IJ is

7
14
1
=
, which occurs when x + y = 1 and xy = , i.e.
8
4
16

2 + 3 2 3
{x, y} =
,
.
4
4

13. Suppose the ant starts at (0, 0, 0) and stops at (1, 1, 1). In each step, the ant changes exactly
one of the x-, y- or z- coordinate. The change is uniquely determined: either from 0 to 1 or
from 1 to 0. Also, the number of times of changing each coordinate is odd. Hence the number
of changes in the three coordinates may be 5-1-1 or 3-3-1 (up to permutations). It follows that
the answer is 3 C57 + 3 C37 C14 = 546 .

14. We first show that student 333 is a good student. Divide the 997 students (all except students
333, 666 and 999) into the following 498 sets:
{1, 2, 4, }, {3, 6, 12, }, {5, 10, 20, }, , {331, 662}, {335, 670}, , {995}, {997}
Now consider a group of 500 students containing student 333. If either student 666 or 999 is in
the group, then it is a good group. Otherwise, the other 499 students of the group come from
the above 498 sets, and the pigeon-hole principle asserts that two of them come from the same
set. For any two students coming from the same set, one must have a student number which
divides the others. It follows that this must be a good group, and hence student 333 is a
good student.
Finally we show that students 334 to 1000 are not good students. Clearly, students 501 to
1000 form a bad group , and hence students 501 to 1000 are not good students. Now
consider the group formed by the following 500 students:
334, 335, , 667, 669, 671, 673, 997, 999.
This group consists of students 334 to 667 and all odd- numbered students from 669 to 999. It
is easy to see that these students form a bad group, because for 334 n 500 , student n is in
the group but student 2n is not, and 3n already exceeds 1000. For n 501 , 2n already exceeds
1000. Hence we cant find two students for which the student number of one divides that of
another. It follows that this is a bad group, and hence students 334 to 500 are not good
students. Consequently, the answer is 333.

15. Note that ABC and ACD are equilateral. Rotate


ACP clockwise through 60 about C to DCQ .
Note that PCQ is equilateral and hence APQ is a
straight line. Also, we have AQC BPC . Since

APC + ABC = 180 , ABCP is a cyclic


quadrilateral. Hence DQP = APB = ACB = 60 .
Let AP = x and CP = y . Then

A
x

120
B

y
Q
C

3 = BP = AQ = AP + PQ = x + y .

On the other hand, applying cosine law in DQP gives 2 2 = x 2 + y 2 2 xy cos60 . These two
5
equations give x 2 + y 2 + 2 xy = 9 and x 2 + y 2 xy = 4 respectively. Thus xy = and hence
3
7
5 7
3
x 2 + y 2 2 xy = 4 = . Thus ( x y) 2 = , or ( x y) =
, i.e. the difference between the
3
3 3
7
7
21
lengths of AP and CP is
=
.
3
3

16. Since n leaves a remainder of 502 when divided by 802, we may write n = 802k + 502 for
some integer k. Let k = 5h + r for some integers h and r such that 0 r 4 . Then we have
n = 802(5h + r ) + 502 = 2005(2h) + (802r + 502) .

Setting r = 0, 1, 2, 3, 4 respectively, we see that the remainders when n is divided by 2005 are
502, 1304, 101, 903 and 1705 respectively. Finally, it is indeed possible for the remainder to be
101. The reason is as follows. According to the question, we know that n + 300 is divisible by
both 902 and 702, and hence by their L.C.M. (say L). Since L and 2005 are relatively prime,
the Chinese remainder theorem asserts that there exists a positive integer n which is congruent
to 300 modulo L and to 101 modulo 2005. It follows that the answer is 101.

17. Let a n denote the number held by children n. Since the sum of the numbers of any 2005
consecutive children is equal to 2005, we have an = an+ 2005 for all n, where the index is taken
modulo 5555. In particular, a0 = a2005k for all positive integers k. Since (2005,5555) = 5 , there
exists a positive integer k such that 2005k 5 (mod 5555). Hence a0 = a5 , and in general
an = an+ 5 for all positive integers n. Therefore the sum of the numbers of any 5 consecutive
5
children is the same, and should be equal to 2005
= 5 . In particular, since {1, 12, 123,
2005
1234, 5555} form a complete residue system modulo 5, we must have
a1 + a12 + a123 + a1234 + a5555 = 5 , and hence the answer is 5 1 21 321 4321 = 4659 .

18. Note that


r

1 a (b c ) c(b a) ab ac cb + ca b( a c )
=

=
=
= 1 .
r b( c a ) b (c a)
b (c a )
b( c a )

1 1 2 4(1)( 1)
1
Hence r + 1 = 0 , i.e. r 2 + r 1 = 0 . Solving, we get r =
. Since r > 0 , we
r
2
5 1
take the positive root to get r =
.
2
19. Note that Bn = (l1 + l2 + l3 +L + ln ,0) . Set Sn = l 1 +l 2 +L +l n . The equation of Bn 1 An is

y = 3( x S n1 ) . Hence the y-coordinate of An satisfies


yn = 3( y n2 Sn 1) .
Solving under the condition yn > 0 , we get
yn =

1 + 1 + 12S n 1
2 3

Since ln =

yn
, we have
sin60

1 + 1 + 12S n1
3
ln =
,
2
2 3

i.e. (3ln 1) 2 = 1 + 12S n1 . Upon simplification, we have

and hence

3l n2 2l n = 4 S n 1

(1)

3l n+12 2l n+1 = 4S n

(2)

3(l n+12 ln2 ) 2(l n+ 1 ln ) = 4( S n S n1 )

(2) (1):

3(ln +1 + l n )( ln +1 ln ) 2(l n+ 1 ln ) = 4ln


3( ln +1 + l n )(l n+ 1 ln ) 2(l n+1 + ln ) = 0

Since ln l n+1 , we have


ln +1 ln =

2
3

Thus {ln } is an arithmetic sequence with common difference

2
. Furthermore,
3

1 + 1 + 12 S0
3
l1 =
,
2
2 3

where S0 is taken to be zero. This gives l1 =


l1 + l2 +L + l2005 =

2
. It follows that
3

2
2 2005 2006 4022030
(1 + 2 + L 2005) =
=
.
3
3
2
3

Remark. The condition that B1 , B2 , are distinct is missing in the original question. This is
necessary to ensure that ln l n+1 so that we can cancel out the term ln +1 ln .

20. Applying the product-to-sum formula, we have

sin B = 2005cos( A + B)sin A


2005
=
[ sin(2 A + B)sin( B )]
2
2007
2005
sin B =
sin(2 A + B)
2
2
2005
sin B =
sin(2 A + B)
2007
2005
2005
2005
2005 2006
, and so tan B
=
=
. Equality is
2007
4012
2007 2 2005 2 2 2006
possible when sin(2 A + B) = 1 , i.e. when 2 A + B is a right angle.

Hence sin B

International Mathematical Olympiad


Preliminary Selection Contest 2004 Hong Kong

Outline of Solutions

Answers:
1.

2.

10

3.

3
N
2

4.

49894

5.

1
1

2 2004!

6.

4011
4010

7.

63

8.

171

9.

10. 103

11.

3007
2

12. 280616

13. + 2

14. 35

15. 603

16. 2475

17. 7

18. 540

19.

20. 518656

3:2

Solutions:

1.

A total of C38 = 56 triangles can be formed by joining any three vertices of the cuboid. Among
these, if any two vertices of a triangle are adjacent vertices of the cuboid, the triangle is rightangled. Otherwise, it is acute angled. To see this latter statement, note that if the dimensions of
the cuboid are p q r , then we can find from the cosine law that the cosines of the angles of
such a triangle will be equal to

p2
q2
r2
,
and
,
( p 2 + q 2 )( p2 + r 2 ) (q 2 + p 2 )(q 2 + r 2 )
(r 2 + p 2 )(r 2 + q 2 )

which are all positive.


For each fixed vertex (say A), we can form 6 triangles which are right-angled at A (two on each
of the three faces incident to A). Thus the answer is 56 6 8 = 8 .

2.

Let the total weight of the stones be 100. Then the weight of the three heaviest stones is 35 and
5
that of the three lightest stones is (100 35) = 25 . The remaining N 6 stones, of total
13
25
35
35
weight 100 35 25 = 40 , has average weight between
and
. Since 40 > 3 and
3
3
3
1

40

3.

25
< 5 , we must have N 6 = 4 , from which the answer N = 10 follows.
3

From the first equation, we have 2 y = N + 2 [ x] , which is an integer. Hence y is either an


integer or midway between two integers. The same is true for x by looking at the second
1
1
equation. Hence, either [ x] = x or [ x] = x , and either [ y ] = y or [ y ] = y .
2
2
If x and y are both integers, [ x] = x and [ y ] = y . Solving the equations, we get x =
y=N+

4
N and
3

1
which are not consistent with our original assumptions.
3

1
If x is an integer and y is midway between two integers, [ x] = x and [ y ] = y . Solving the
2
5
1
equations, we have x = N and y = N + which are not consistent with our original
3
6
assumptions.

Similarly, if x is midway between two integers and y is an integer, we have [ x] = x


[ y ] = y . Solving the equations, we have x =

1
and
2

7
2
N and y = N + which are again not
6
3

consistent with our original assumptions.


Finally, if x and y are both midway between two integers, we have [ x] = x
Solving the equations, we have x =

4.

1
1
and [ y ] = y .
2
2

3
1
N and y = N + . This gives the correct answer.
2
2

Let the answer be abcba. Note that


abcba = 10001a + 1010b + 100c = 101(99a + 10b + c) + 2a c

For the number to be divisible by 101, we must have 2a c = 0 . For the number to be largest,
we may take a = 4 , c = 8 and b = 9 . This gives the answer is 49894.

5.

Note that
k+2
1
k +1
1
1
=
=
=

.
k ! + (k + 1)! + ( k + 2)! k !(k + 2) (k + 2)! (k + 1)! (k + 2)!

Hence

3
4
2004
+
+L +
1! + 2! + 3! 2! + 3! + 4!
2002! + 2003! + 2004!
1
1 1 1 1
1
= + +L +

2! 3! 3! 4!
2003! 2004!
1
1
=
2 2004!

6.

Let x = a + b , y = b + c and z = c + a . Then


2004 (a b)(b c)(c a ) ( z y )( y x)( x z )
=
=
2005 (a + b)(b + c)(c + a)
xyz
and hence
a
b
c
x y+ z y z+ x z x+ y
+
+
=
+
+
a +b b+ c c+ a
2x
2y
2z
1 y z 1 z x 1 x y
=
+
+

2z
2 2x 2 2 y 2
=

3 1 ( z y )( y x)( x z )

2 2
xyz

3 1 2004

2 2 2005
4011
=
4010
=

7.

Draw E such that ABCE is a parallelogram.


Since AEC = ABC = 55 and ADC =
180 31 24 = 125, we have AEC +
ADC = 180 and thus ADCE is a cyclic
quadrilateral. Now EC = AB = DC, so CDE =
CED = CAD = 31. Considering CDE, we
have ACE = 180 31 31 24 = 94. It
follows that DAB = BAC 31 = ACE
31 = 94 31 = 63.

E
31

24

55

(Alternatively, instead of drawing the point E, one can reflect B across AC and proceed in
essentially the same way.)

8.

We have
999973 = 1000000 27
= 1003 33
= (100 3)(1002 + 100 3 + 32 )
= 97 10309
= 97 13 793
= 97 132 61
and so the answer is 97 + 13 + 61 = 171 .

9.

We first note that the five primes 5, 11, 17, 23, 29 satisfy the conditions. So n is at least 5.
Next we show that n cannot be greater than 5. Suppose there are six primes satisfying the
above conditions. Let a be the smallest of the six primes and let d be the common difference of
the resulting arithmetic sequence. Then d must be even, for if d is odd then exactly three of the
six primes are even, which is not possible. Similarly, d must be divisible by 3, for otherwise
exactly 2 of the six primes are divisible by 3, which is not possible.
Moreover, if d is not divisible by 5, then at least one of the six primes must be divisible by 5.
Therefore 5 is one of the primes picked. But we have shown that d is divisible by 6, so 5 is the
smallest among the six primes. But then the largest of the six primes, 5 + 5d , is also divisible
by 5 and is larger than 5. This is absurd.
Hence d is divisible by 2, 3 and 5, hence divisible by 30. So the largest of the 6 primes, which
is a + 5d , must be larger than 150, a contradiction. It follows that the answer is 5.

10. By setting p (0) = 1 , we may write S = p (000) + p (001) + p (002) + L + p (999) p (000) . Since
we are now computing the product of non-zero digits only, we may change all the 0s to 1s,
i.e. S = p (111) + p (111) + p (112) + L + p (999) p (111) . Each term is the product of three
numbers, and each multiplicand runs through 1, 1, 2, 3, 4, 5, 6, 7, 8, 9 (note that 1 occurs twice
as all 0s have been changed to 1s). Hence we see that

S = (1 + 1 + 2 + 3 + L + 9) (1 + 1+ 2 + 3 + L + 9) (1 + 1 + 2 + 3 + L + 9) 1
= 463 1
= (46 1) (462 + 46 + 1)
= 32 5 2163
= 32 5 3 7 103
It follows that the answer is 103.

11. Using

1
1
1
, we have
=
k (k + 1) k k + 1

1 1 1
1
1
A = 1 + +L +

2 3 4
2003 2004
1 1 1
1
1
= 1 + + L +
2 + +L +

2004 2 4
2004
2
1 1
1
1
= 1 + + L +
1 + + L +

2004 2
1002
2
1
1
1
=
+
+L +
1003 1004
2004
Consequently,
1 1
1
1
1
1
2A =
+
+
+
+
+L +

1003 2004 1004 2003


2004 1003
3007
3007
3007
=
+
+L +
1003 2004 1004 2003
2004 1003
= 3007 B

It follows that

A 3007
=
.
B
2

12. If a b is odd, both digits are odd and we have 5 5 = 25 choices. If it is even, we have
9 9 5 5 = 56 choices (note that both digits cannot be zero). The same is true for the
quantities c d and e f .
Now, for condition (b) to hold, either all three quantities a b , c d and e f are even, or
exactly two of them are odd. Hence the answer is 563 + 3 56 252 = 280616 .

13. Observe that the graph is symmetric about the


x-axis and the y-axis. Hence we need only
consider the first quadrant. In the first quadrant,
the equation of the graph can be written as
2

y
1

1
1
1

x + y = x + y , or x + y = ,
2
2
2

which is a circle passing through (0, 0), (1, 0)


and (0, 1). By symmetry, the whole graph can
be constructed as shown.
Now the area bounded by the curve can be
thought of as a square of side length 2 plus
four semi-circles of diameter 2 . Its area is
5

( 2)

1 2
+ 4
= +2.
2 2

14. Using the identity


a 3 + b3 + c 3 3abc =

1
(a + b + c) (a b) 2 + (b c) 2 + (c a) 2 ,
2

we have
m3 + n3 + 99mn 333 = m3 + n3 + (33)3 3mn(33)
=

1
(m + n 33)[(m n) 2 + (m + 33) 2 + (n + 33) 2 ]
2

For this expression to be equal to 0, we either have m + n = 33 or m = n = 33 . The latter gives


one solution (33, 33) while the former gives the 34 solutions (0, 33), (1, 32), , (33, 0).
Hence the answer is 35.

15. Let f (n) be the number of significant figures when 2 n is written in decimal notation. Then
f (n) = f (n + 1) when n is lucky, and f (n) + 1 = f (n + 1) otherwise. Now f (2) = 2 , and we
want to compute f (2004) . We first note that
22004 =

52004
,
102004

so f (2004) is equal to the number of digits of 52004 . Now


log 52004 = 2004(1 log 2) 2004 (0.7 0.001) = 1402.8 2.004 = 1400.796 ,
so 52004 has 1401 digits. It follows that the number of lucky numbers less than 2004 is equal to
2004 1401 = 603 .

16. Let n be such a number. Since 3n is divisible by 3, the sum of the digits of 3n is divisible by 3.
But the sum of the digits of 3n is the same as the sum of the digits of n, so the sum of digits of
n is divisible by 3, i.e. n is divisible by 3. As a result, 3n is divisible by 9, so the sum of digits
of 3n is divisible by 9. Again, the sum of the digits of 3n is the same as the sum of the digits of
n, so the sum of digits of n is divisible by 9, i.e. n is divisible by 9.
Let n = abcd . Since n is to be divisible by both 9 and 11, a + b + c + d is divisible by 9 and
(a + c) (b + d ) is divisible by 11. Considering the parities of a + c and b + d we see that
a + b + c + d has to be equal to 18 with a + c = b + d = 9 .

The rest is largely trial and error. Noting that a can be no larger than 3, we have the
possibilities (a, c) = (1, 8); (2, 7) or (3, 6). Considering the digits, the only possibilities for n
6

are 1287, 1386, 1485, 2079, 2475, 2574, 3465, 3762 and 3861. Among these, we find that only
n = 2475 works as 3n = 7425 in this case.

b 10 n + 1
=
. Let this be an integer d. Noting that 10 n1 a < 10n
2
a
a
and n > 1 , we must have 1 < d < 11 . Since 10n + 1 is not divisible by 2, 3 and 5, the only
possible value of d is 7. Indeed, when a = 143 , we have b = 143143 and d = 7 .

17. Note that b = a(10n + 1) , so

18. By the AM-GM inequality, 9 tan 2 x + cot 2 x 2

( 9 tan x )( cot x) = 6 . It follows that the


2

minimum value of the right hand side is 1. On the other hand, the maximum value of the left
hand side is 1. For equality to hold, both sides must be equal to 1, and we must have
1
9 tan 2 x = cot 2 x (which implies tan x =
), cos12 x = 1 and sin 3 x = 1 .
3
For tan x =

1
, the solutions are x = 30, 150, 210, 330.
3

For cos12 x = 1 , the solutions are x = 0, 30, 60, , 330, 360.


For sin 3 x = 1 , the solutions are x = 90, 210, 330.
Therefore the equation has solutions x = 210, 330 and so the answer is 210 + 330 = 540 .

19. Let [DEF] = 2.


C

Since CD : DE = 3 : 2 , [FCD] = 3.
Since AB : BC = 1: 2 , [FBD] = 1.

G
B

Since EF = FG, [GFD] = [DEF] = 2.


So [GBF] = 1 = [FBD], i.e. GB = BD.

Together with GF = FE, BF // CE.

Hence AF : FD = AB : BC = 1: 2 .
E

Let GB = BD = x , AF = y, FD = 2y.
Since CAD = EGD, GAFB is a cyclic quadrilateral.
Thus DB DG = DF DA , i.e. x(2 x) = 2 y (3 y ) .
As a result we have x : y = 3 .
It follows that BD : DF = x : 2 y = 3 : 2 .

20. The key observation is that f (n) = n g (n) , where g (n) is the number of 1s in the binary
representation of n. To see this, it suffices to check that for non-negative integers
a1 < a2 < L < ak and n = 2a1 + 2a2 + L + 2ak , we have
n n
n
2 + 22 + L + 2ak

= nk.

Such checking is straightforward.


Next we try to compute g (0) + g (1) + L + g (1023) . Note that the binary representations of 0
and 1023 are respectively 0000000000 and 1111111111, so as n runs through 0 to 1023, g (n)
is equal to 5 on average. Since g (0) = 0 , we have
g (1) + L + g (1023) = 5 1024 = 5120 .

Now we have
f (1) + f (2) + L + f (1023) = (1 + 2 + L + 1023) [ g (1) + g (2) + L + g (1023) ]
1023 1024
5120
2
= 512 (1023 10)
= 518656
=

International Mathematical Olympiad 2004


Hong Kong Preliminary Selection Contest
May 31, 2003
Time allowed: 3 hours
Answer ALL questions.
Put your answers on the answer sheet.
The use of calculator is NOT allowed.
3

1.

2.

3.

4.

5.

6.

7.

8.

1
Let x = 3 4 + 3 2 + 1 . Find the value of 1 + .
x

(1 mark)

15 students join a summer course. Every day, 3 students are on duty after school to
clean the classroom. After the course, it was found that every pair of students have
been on duty together exactly once. How many days does the course last for?

(1 mark)

Find the number of pairs of consecutive integers in the set {1000, 1001, 1002, ,
2000} such that no carrying is required when the two integers are added.

(1 mark)

A positive integer x is called a magic number if, when x is expressed in binary


form, it possesses an even number of 1s. For example, the first five magic
numbers are 3, 5, 6, 9 and 10. Find, in decimal notation, the sum of the first 2003
magic numbers.

(1 mark)

A positive integer n is said to be increasing if, by reversing the digits of n, we get


an integer larger than n. For example, 2003 is increasing because, by reversing the
digits of 2003, we get 3002, which is larger than 2003. How many four-digit
positive integers are increasing?

(1 mark)

The ratio of the sides of a triangle, which is inscribed in a circle of radius 2 3 , is


3 : 5 : 7 . Find the area of the triangle.

(1 mark)

The number of apples produced by a group of farmers is less than 1000. It is


known that they shared the apples in the following way. In turn, each farmer took
from the collection of apples either exactly one-half or exactly one-third of the
apples remaining in the collection. No apples were cut into pieces. After each
farmer had taken his share, the rest was given to charity. Find the greatest number
of farmers that could take part in the apple sharing.

(1 mark)

Let a and b be positive integers such that 90 < a + b < 99 and 0.9 <

(1 mark)

ab.

9.

1
Find an integer x such that 1 +
x

a
< 0.91. Find
b

x +1

= 1 +

2003

Taught By Mr. Sophorn ROEUNG, Dir. READ AND THINK,

2003

(1 mark)

(012-013) 47 36 17 & 099 72 3003

10. Simplify (cos 42 + cos102 + cos114 + cos174) 2 into a rational number.

(1 mark)

11. On a certain planet there are 100 countries. They all agree to form unions, each
with a maximum of 50 countries, and that each country will be joining a number of
unions, so that every two different countries will belong to a same union. At least
how many unions must be formed?

(1 mark)

12. Find the last two digits of 7 19 31 1999 . (Here 7, 19, 31, , 1999 form an
arithmetic sequence of common difference 12.)

(1 mark)

13. ABCDEFGH is a cube in which ABCD is the top face, with vertices H, G, F and E
directly below the vertices A, B, C and D respectively. A real number is assigned to
each vertex. At each vertex, the average of the numbers in the three adjacent
vertices is then computed. The averages obtained at A, B, C, D, E, F, G, H are 1, 2,
3, 4, 5, 6, 7, 8 respectively. Find the number assigned to vertex F.
(2 marks)
14. A regular 201-sided polygon is inscribed inside a circle of center C. Triangles are
drawn by connecting any three of the 201 vertices of the polygon. How many of
these triangles have the point C lying inside the triangle?
(2 marks)
15. Given a rectangle ABCD, X and Y are respectively points on AB and BC. Suppose
the areas of the triangles AXD, BXY and DYC are respectively 5, 4 and 3. Find
the area of DXY.
(2 marks)
16. Let ABC be an acute triangle, BC = 5. E is a point on AC such that BE AC , F
is a point on AB such that AF = BF . Moreover, BE = CF = 4. Find the area of the
triangle.
(2 marks)

17. Given a triangle ABC , ABC = 80 , ACB = 70 and BC = 2. A perpendicular


line is drawn from A to BC, another perpendicular line drawn from B to AC. The
two perpendicular lines meet at H. Find the length of AH.
(2 marks)
18. Let A be a set containing only positive integers, and for any elements x and y in A,
xy
x y . Determine at most how many elements A may contain.
(2 marks)
30
19. A man chooses two positive integers m and n. He then defines a positive integer k
to be good if a triangle with side lengths log m, log n and log k exists. He finds that
there are exactly 100 good numbers. Find the maximum possible value of mn.
(3 marks)
20. The perimeter of triangle ABC, in which AB < AC , is 7 times the side length of BC.
The inscribed circle of the triangle touches BC at E, and the diameter DE of the
DF
circle is drawn, cutting the median from A to BC at F. Find
.
(3 marks)
FE

End of Paper
Taught By Mr. Sophorn ROEUNG, Dir. READ AND THINK,

(012-013) 47 36 17 & 099 72 3003

International Mathematical Olympiad 2004


Hong Kong Preliminary Selection Contest

Outline of Solutions

Answers:
1.

2.

35

3.

156

4.

4015014

5.

4005

6.

135
3
49

7.

12

8.

2346

9.

2004

10.

3
4

11. 6

12. 75

13. 13

14. 338350

15. 2 21

16. 8 3 6

17. 2 3

18. 10

19. 134

20.

5
7

Solutions:

1.


1
y 1
1
3
3
Let y = 2 . Then y = 2 and so 1 + = 1 + 2
= 1 + 3 = (1 + y 1) = y = 2 .
y + y +1
y 1
x

2.

Every day C23 = 3 pairs of students are on duty. Altogether C215 = 105 pairs of students have been on

duty. Therefore the answer is 105 3 = 35 .

3.

Suppose n and n + 1 are to be added, and write n in the form 1ABC. If any one of A, B and C is
equal to 5, 6, 7 or 8, then carrying is needed in the addition. If B is 9 and C is not 9, or if A is 9 and
B and C are not both 9, carrying is also required. Therefore, if no carrying is required, n must be of
one of the following forms:
1ABC

1AB9

1A99

1999

where each of A, B, C is from 0 to 4. It follows that the answer is 53 + 52 + 5 + 1 = 156 .

4.

Let n be a positive integer. Consider the numbers 4n, 4n + 1 , 4n + 2 and 4n + 3 . The last two digits
of their binary representations are 00, 01, 10 and 11 respectively. Among these, either 4n and 4n + 3
are magic numbers, or 4n + 1 and 4n + 2 are magic numbers. In any case, the sum of the two magic
numbers among the four is equal to 8n + 3 . Now the first magic number is 3 = 4(0) + 3 , so the
answer is 3 + 8(1 + 2 + L + 1001) + 3(1001) = 4015014 .

5.

First we count the number of increasing integers of the form ABCA. In this case C must be greater
than B. For each choice of A (there are 9 choices, namely, 1, 2, , 9), there are C210 = 45 choices for
B and C (as we require C to be greater than B and we can choose among 0 to 9). So there are
9 45 = 405 increasing numbers of this form.
Next we count the number of increasing integers of the form ABCD, where A is different from D.
In this case we need only require D to be greater than A, whereas B and C can be arbitrary. Clearly,
A and D have to be non-zero. So there are C29 = 36 ways to choose A and D, and 10 2 = 100 ways to
choose B and C. Thus there are 36 100 = 3600 increasing numbers of this form.
It follows that the answer is 405 + 3600 = 4005 .
Alternative Solution
Clearly, an increasing number cannot end with 0. Consider all four-digit positive integers that do not
end with 0. There are 9 10 10 9 = 8100 such numbers. Among these, 9 10 = 90 of them are
palindromes (as all palindromes are of the form ABBA where A is from 1 to 9 while B is from 0 to
9). So 8100 90 = 8010 of them are not palindromes. By grouping an integer with its reverse (i.e.
ABCD with DCBA), we get 8010 2 = 4005 pairs. In each pair, exactly one integer is increasing. So
the answer is 4005.

6.

Let ABC denote the triangle, and the lengths of the sides opposite A, B, C be 3k, 5k, 7k respectively.
By cosine law, we have

cos C =

(3k ) 2 + (5k ) 2 (7 k ) 2
1
=
2(3k )(5k )
2

Hence C = 120. By sine law,

7k
=2 2 3
sin120

from which we get k =

6
. It follows that the area of ABC is equal to
7

1
15 6
(3k )(5k ) sin120 =
2
2 7

3 135
3.

=
2
49

7.

If there are 729 apples, it is possible that 12 farmers took part in the sharing if the odd-numbered
farmers took one-third of the remaining apples and the even-numbered farmers took one-half of the
remaining apples. In this case one apple is left after the last farmer took his apple and the number of
apples remaining is an integer throughout.
Now suppose p farmers took one-half of the apples and q farmers took one-third. We will show that
p + q 12 , so that 12 is indeed the greatest possible number of farmers. Indeed, we have
q

p q
p q
p +q
3
1000 2 = 2 p q 3q = 4 2 3q 3 2 3q = 3 2 ,
2

from which p + q 12 follows.

8.

We have 0.9b < a < 0.91b , so 1.9b < a + b < 1.91b . Therefore we get the inequalities
1.9b < 99 and 90 < 1.91b .

It follows that 48 b 52 . Putting each value of b into the inequality 0.9b < a < 0.91b , we get two
possibilities (a, b) = (46, 51) or (47, 52). In the latter case, the requirement 90 < a + b < 99 fails.
Thus the former case gives the answer 46 51 = 2346 .

9.

1
1
Setting y = x + 1 , the equation becomes 1 +
= 1 +

y 1 2003

1 +

z 1

= 1 +

2003

2003

z +1

= 1 +

2003

2003

1
z +1

= 1 +

z 2003

2003

2003

. Setting z = y , we get

LL (*)

Hence one possible integer solution is z = 2003 , which corresponds to y = 2003 and x = 2004 .
[From the above solution we also see that the solution for z, and hence the solution for x, is unique.
Indeed, the left hand side of (*) is strictly increasing in z on the natural numbers, so there is no
positive integer solution for z other than 2003. It is clear that z is not equal to 0 or 1. If z is smaller
than 1, then x is a positive integer. Looking at the original equation, we readily see that this is not
possible because on one hand x has to be divisible by 2003, and on the other hand the power x + 1
has to be equal to 2003 by counting the exponent of 2003.]

10. On the unit circle, the points at 114, 42, 30, 102 and 174 form a regular pentagon. So the sum
of the cosines of these angles is 0. It follows that
(cos 42 + cos102 + cos114 + cos174) 2
= [cos(114) + cos(42) + cos102 + cos174]2
= ( cos 30)2
3
=
4
Alternative Solution
We have
(cos 42 + cos102 + cos114 + cos174)2
= (2cos 72 cos 30 + 2 cos144 cos 30)2
= 3(cos 72 + cos144) 2

Now
2sin 36(cos 72 + cos144)
= 2 sin 36 cos 72 + 2 sin 36 cos144
= sin108 sin 36 + sin180 sin108
= sin 36
1
3
so that cos 72 + cos144 = . It follows that the answer is .
2
4

[As one could expect, there are many other ways of obtaining the answer, notably by solving for
cos 36 in surd form. The above solutions are surd-free in the entire process.]

11. First, observe that at least 6 unions must be formed. If a country joins only 2 unions, then it is in a
common union with at most 49 2 = 98 countries. So each country must join at least 3 unions, and
thus the smallest possible number of unions is 100 3 50 = 6 .
Next, 6 unions are enough. We can divide the 100 countries into 4 groups of 25 countries each.
Choose 2 of the 4 groups at a time, and put the 50 countries in the 2 groups into a union. This gives
C24 = 6 unions. It is easy to see that in this way, any two countries belong to some union in common.
It follows that the answer is 6.

12. Note that 7, 19, 31, , 1999 form an arithmetic sequence with 167 terms and common difference 12.
Since 12 and 25 are relatively prime, at least one of the terms is divisible by 25.
It remains to consider the product (mod 4). Note that each term is congruent to 1 (mod 4). As there
are 167 terms, the product is also congruent to 1 (mod 4). Since it is divisible by 25, the answer
must be 75.

13. Let a, b, c, d, e, f, g, h denote the numbers assigned, and A,


B, C, D, E, F, G, H the average obtained, at the vertices A
to H respectively. First we note that
a + b + c + d + e + f + g + h = 1 + 2 + 3 + 4 + 5 + 6 + 7 + 8 = 36 .

A
C

Also,

c + f = 36 (b + d + h) (a + e + g ) = 36 3( A + H ) .

Similarly, we have

e + f = 36 3( A + B ) and f + g = 36 3( A + D ) .

Therefore
3 f = ( c + f ) + (e + f ) + ( f + g ) (c + e + g )

= [36 3(1 + 8)] + [36 3(1 + 2)] + [ 36 3(1 + 4) ] 6 3


= 39

from which we get f = 13.

14. Fix a vertex A and draw a diameter from A. This diameter splits the remaining 200 vertices into two
groups of 100 each. Number them as P1 , P2 , K , P100 and Q1 , Q2 , K , Q100 according to the distance of
each vertex from A. Note that a triangle with A as vertex contains point C if and only if it is of the
form APQ
i j where i + j > 100 . Hence there are 1 + 2 + L + 100 = 5050 triangles with A as a vertex
which contains C. Since A is arbitrary we have 5050 201 = 1015050 such triangles. However, in
this way each triangle is counted three times. So the answer is 1015050 3 = 338350 .

15. Let AX = a and XB = b . Then AD =


Now the area of the rectangle is

10
8
and BY = .
a
b

B
4

10
(a + b) = 10 + 10 R ,
a

b
where R = is to be determined. The area of DXY is then
a
equal to 10 + 10 R 5 4 3 = 10 R 2 .

5
Y
3
D

Consider DYC. We have


8
10 8
6 = DC CY = (a + b) = 2 + 10 R .
R
a b

It follows that 5 R 2 2 R 4 = 0 . This gives R =


R is positive. The answer is 10 R 2 = 2 21 .

2 22 4(5)(4)
. We take the positive root since
2(5)

16. Let D be the foot of the perpendicular from F to AC.

Note that EC = BC 2 BE 2 = 52 42 = 3 . On the


1
other hand, FD = BE = 2 . Therefore
2

DC = CF 2 FD 2 = 42 22 = 2 3 .

It follows that AD = DE = 2 3 3 . Thus


[ ABC ] = [ ABE ] + [ BEC ]

1
1
(4) 2 2 3 3 + (4)(3)

2
2
=8 3 6
=

17. Note that H is the orthocentre of ABC. Let D and E


be the feet of the perpendiculars from A to BC and B
to AC respectively. We have HAE = 20 and HCE
= 60. Thus

HE
sin 20
EC tan 60
=
sin 20
BC cos 70 tan 60
=
sin 20
= BC tan 60

AH =

E
H

=2 3

Alternative Solution 1
Note that H is the orthocentre of ABC. We have
HBC = 20, HCB = 10, BAH = 80 and ABH
= ACH = 60. Applying sine law in BHC, we have

2
BH
=
sin150 sin10
BH = 4 sin10

Applying sine law in ABH, we have


4sin10
AH
=
sin10 sin 60

AH = 2 3

Alternative Solution 2
Note that H is the orthocentre of ABC. Construct the
circumcircle of ABC and the diameter BD. The
diameter of the circle is

A
D

BC
2
=
= 4.
sin A sin 30

Since AH and DC are both perpendicular to BC, they


are parallel. Similarly, AD and HC are parallel. So
AHCD is a parallelogram, and thus

AH = DC = BD 2 BC 2

= 42 22

=2 3

18. We note that x and y cannot be too large. Starting with A = {1} and applying the greedy algorithm,
we try to append the smallest possible element to A each time. By this process get
A = {1, 2, 3, 4, 5, 6, 8, 11, 18, 45} ,

after which we can no longer append more elements to A. So A can have at least 10 elements.
Let A = {a1 , a2 , K , an } . We will show that n 10 , so that the answer is indeed 10. Suppose
a a
1
1
1
1 a1 < a2 < L < an . Then ai +1 ai i +1 i , or

for 1 i n 1 . Summing from i = 5


30
ai ai +1 30
1 1 n5
1 n5
to i = n 1 , we get

. So
>
. Noting that ai i for all i, this implies
30
30
a5 an
a5
30
5 a5 <
, from which we get n 10 as desired.
n5

19. Assume m n . Then k is good if and only if log m log n < log k < log m + log n . This is equivalent
m
to
< k < mn . According to the question, there are 100 possible integral values of k. It is easy to
n
see that n cannot be 1. We claim that mn 134 . Otherwise, we have
mn

m
mn
mn 3mn 3(135)
= mn 2 mn
=

> 101
n
n
4
4
4

so that there would be more than 100 good numbers. Moreover, when mn = 134 with m = 67, n = 2,
k can take one of the 100 values 34, 35, 36, , 133. Hence the maximum value of mn is 134.

20. Let M be the mid-point of BC and K be the foot of the


perpendicular from A to BC. Let also AK = h, s be the semiperimeter and r be the radius of the inscribed circle. With
standard notations, we have

CK = b cos C =

a 2 + b2 c 2
2a

CE = s c

bc
ME = CE CM =
2
b2 c 2
MK = CK CM =
2a
Noting that MEF ~ MKA, we have
On the other hand, we have

FE ME
a
1
1
=
=
= , i.e. FE = h .
AK MK b + c 6
6

ah
= rs since both are equal to the area of ABC.
2

a
2
2 1 1 5
As a result, DE = 2r = h = h . So the answer is : = .
s
7
7 6 6 7

ME K

International Mathematical Olympiad 2003


Hong Kong Preliminary Selection Contest
May 26, 2002
Answer ALL questions
Put your answers on the answer sheet
The use of calculator is NOT allowed

Time allowed: 3 hours

1. (1 mark) Let n be a positive integer such that no matter how 10n is expressed as the
product of two positive integers, at least one of these two integers contains the digit
0. Find the smallest possible value of n.
2. (1 mark) A clock has an hour hand of length 3 and a minute hand of length 4. From
1:00 am to 1:00 pm of the same day, find the number of occurrences when the
distance between the tips of the two hands is an integer.
3. (1 mark) Find the sum of all integers from 1 to 1000 which contain at least one 7 in
their digits.
4. (1 mark) A multiple choice test consists of 100 questions. If a student answers a
question correctly, he will get 4 marks; if he answers a question wrongly, he will get
1 mark. He will get 0 mark for an unanswered question. Determine the number of
different total marks of the test. (A total mark can be negative.)
5. (1 mark) A positive integer is said to be a palindrome if it reads the same from left
to right as from right to left. For example 2002 is a palindrome. Find the sum of all 4digit palindromes.
6. (1 mark) Points A and B lie on a plane. A straight line passing through A will
divide the plane into 2 regions. A further straight line through B will altogether
divide the plane into 4 regions, and so on. If 1002 and 1000 straight lines are drawn
passing through A and B respectively, what is the maximum number of regions
formed?
7. (1 mark) In ABC , X , Y are points on BC such that BX = XY = YC , M , N are
points on AC such that AM = MN = NC. BM and BN intersect AY at S and R
respectively. If the area of ABC is 1, find the area of SMNR.
8. (1 mark) Given that 0.3010 < log 2 < 0.3011 and 0.4771 < log 3 < 0.4772. Find the
leftmost digit of 1237.
9. (1 mark) Let x1 , y1 , x2 , y2 be real numbers satisfying the equations x12 + 5 x22 = 10,
x2 y1 x1 y2 = 5 and x1 y1 + 5 x2 y2 = 105. Find the value of y12 + 5 y22 .
10. (1 mark) How many positive integers less than 500 have exactly 15 positive integer
factors?
11. (1 mark) Find the 2002nd positive integer that is not the difference of two square
integers.
12. (1 mark) In trapezium ABCD, BC AB, BC CD and AC BD. Given AB = 11
and AD = 1001. Find BC.
13. (2 marks) Let ABCD be a square of side 5, E a point on BC such that
BE = 3, EC = 2. Let P be a variable point on the diagonal BD. Determine the length
of PB if PE + PC is smallest.

14. (2 marks) In ABC , ACB = 3BAC , BC = 5, AB = 11. Find AC.


15. (2 marks) In ABC , D, E and F are respectively the midpoints of AB, BC and CA.
Furthermore AB = 10, CD = 9, CD AE. Find BF .
16. (2 marks) Each face and each vertex of a regular tetrahedron is coloured red or blue.
How many different ways of colouring are there? (Two tetrahedrons are said to have
the same colouring if we can rotate them suitably so that corresponding faces and
vertices are of the same colour.
3 an 1 + 1
17. (2 marks) Let a0 = 2 and for n 1, an =
. Find the value of a2002 in the form
3 an 1
p + q 3 where p and q are rational numbers.
18. (2 marks) Let A1 A2 " A2002 be a regular 2002-sided polygon. Each vertex Ai is
associated with a positive integer ai such that the following condition is satisfied: If
j1 , j2 ," , jk are positive integers such that k < 500 and Aj1 Aj2 " Ajk is a regular
k sided polygon, then the values of a j1 , a j2 ," , a jk are all different. Find the
smallest possible value of a1 + a2 + " + a2002 .
19. (3 marks) There are 5 points on the plane. The following steps are used to construct
lines. In step 1, connect all possible pairs of the points; it is found that no two lines
are parallel, nor any two lines perpendicular to each other, also no three lines are
concurrent. In step 2, perpendicular lines are drawn from each of the five given points
to straight lines connecting any two of the other four points. What is the maximum
number of points of intersection formed by the lines drawn in step 2, including the 5
given points?
20. (3 marks) A rectangular piece of paper has integer side lengths. The paper is folded so
that a pair of diagonally opposite vertices coincide, and it is found that the crease is of
length 65. Find a possible value of the perimeter of the paper.
End of Paper

International Mathematical Olympiad 2003


HK Preliminary Selection Contest (May 23, 2002)
Solutions
1.

We only need to consider the case 10 n = 2 n 5 n . (In other cases, there will be a zero as unit
digit for one of the two factors.) For n 7 , 2 n and 5 n do not contain the digit 0 . For
n = 8 , 58 = 390625 . So answer is 8.

2.

Between 1:00 am and 1:00 pm, the hands overlap 11 times. Between two overlappings there are
11 times when the distance is an integer (2 times for distances equal 2, 3, 4, 5, 6, and 1 time for
distance equals 7). After the last overlapping (12:00 noon), and before 1:00 pm, again 11 times
when distances are integers. Thus altogether 11 + 11 10 + 11 = 132 times that distances are
integers.

3.

1
1
799 800 699 700 = 74950 . The
2
2
1
1
sum of the numbers from 70 to 79 is 79 80 69 70 = 745 . Hence all numbers
2
2
that end from 70 to 79 (excluding those starting from 7, as we have already counted those from
700 to 799) is 7 4 5 9 + 1 0 (1 0 0 + 2 0 0 + " + 6 0 0 + 8 0 0 + 9 0 0 ) = 4 4 7 0 5 . The sum of
all
numbers
ending
in
7,
(excluding
the
previous
two
cases),
is
9 ( 7 + 1 7 + " + 6 7 + 8 7 + 9 7 ) + 9 (1 0 0 + 2 0 0 + " + 6 0 0 + 8 0 0 + 9 0 0 ) = 3 8 1 8 7 . So
The sum of the numbers from 700 to 799 is

the total sum of numbers containing a 7 is 7 4 9 5 0 + 4 7 7 0 5 + 3 8 1 8 7 = 1 5 7 8 4 2.


Alternate Solution
Let P and Q be respectively the sum of all positive integers less than 1000 and the sum of all
999 1000
= 499500. On the
positive integers containing no 7s in their digits. Clearly P =
2
other hand, we can easily see Q is the sum of 9 3 = 729 numbers. Considering the unit digits of
729
these numbers, there should be 81(=
) occurrences for each digit except 7. Therefore, the sum
9
of all unit digits equals 81 (0 + 1 + 2 + ...... + 6 + 8 + 9 ) = 81 83 . The same is true for the tenth
and hundredth digits. Hence, Q = 81 83 (1 + 10 + 100 ) = 341658. Finally, the answer required is
therefore P Q = 157842
4.

If a student answers 100 questions correctly, he gets 400 marks. If he answers 99 questions
correctly, depending if he answers or does not answer the remaining question, he gets 396 or 395
marks. If a student answers 98 questions correctly, depending if he answers or does not answer
the remaining 2 questions, he gets 390, 391, or 392 marks. Hence the total marks cannot be 389,
393, 394, 397, 398, 399. If a student answers 97 (or fewer) questions correctly, he get at most
388 marks, by adjusting the questions he answers wrongly, he gets everything possible in
between . Thus the number of different total marks = 100 + 401 6 = 495 .

5.

Each 4-digit palindrome is of the form abba , where a ranges from 1 to 9 and b from 0 to 9.
9

Hence the sum of all 4-digit palindromes is equal to

(1001 m + 110 n )
n = 0 m =1

= (1001 45 + 110n 9) = 1001 45 10 + 45 9 110 =11000 45 = 495000.


n =0

6.

The 1000 lines through B divide the plane into 2000 regions. For the first line drawn through A,
the number of new regions formed is 1001. For each new line drawn through A, the number of
new regions formed is 1002. Therefore the total number of regions formed is
1002 1002 1 + 2000 = 1006003.

7.

Denote by [ BMN ] the area of BMN , then


[BMN ] = 1 . Join YN , then YN // BA and
3
RN YN CY 1
BR 3
=
=
= , hence
= . Draw
BR AB CB 3
BN 4
AD and CD such that AD // BC , CD // AB ;
extend BM and BN to meet AD and CD

at E and F respectively. We have

AE AM 1
=
= , so E is the midpoint of AD and
BC MC 2

2
4
2
BS BY 3 4
4
6
BS
BM = BE . Also
=
= = and so BS = BE . Finally we have
= 7 = . Now
1
2
3
7
3
SE AE
7
BM
2
3
[BSR] = BS BR = 6 3 = 9 .
and
Therefore
[BSR] = 9 1 [ABC ] = 3 ,
[BMN ] BM BN 7 4 14
14 3
14
[MNRS ] = 1 3 [ABC ] = 5 [ABC ] = 5 .
42
42
3 4

Alternate Solution
The problem is a matter of ratios of areas, and we may take any triangle satisfying the prescribed
1 2
conditions. Suppose we take A(0, 1), B(-2,0), C(1,0), then we have Y(0,0), M , and
3 3
2 1
4
1
N , . AY and BM meet at S 0, ; also AY and BN meet at R 0, . The area of the
3 3
7
4

4
7
1
0
4
15
1 2 1
1 4 4 1 1 1 1
quadrilateral SRNM is
= + =
(45) = . Hence, the ratio
2 9 21 6 9 2 126
84
2 3 3
1 2
3 3
4
0
7
15 3 5
of the areas of the quadrilateral SRNM over the triangle is
/ =
84 2 42
Remark: The problem may also be solved using the Menelaus theorem.
0

8.

9.

Check that log12 = 2 log 2 + log 3 , so we have 1.0791 = 2 0.3010 + 0.4771 < log 12 <
2 0.3011 + 0.4772 = 1.0794 , which implies 39 + log 8 = 39 + 3 log 2 < 39.9267 < 37 log 12 <
39.9378 < 39 + 2 log 3 = 39 + log 9 . Hence the leftmost digit of 12 37 is 8.

(x

)(

+ 5 x22 y12 + 5 y 22 = ( x1 y1 + 5 x2 y 2 ) + 5( x2 y1 x1 y 2 ) , thus 10 y12 + 5 y 22 = 105 + 5 5 2 = 230 , or


y + 5 y22 = 23.
2
1
2
1

Alternate Solution
From x1 + 5 x 2 = 10 , we may have let x1 = 10 cos , x2 = 2 sin . Similarly if we let
r
2
2
sin , then y1 + 5 y 2 = r 2 . From x 2 y1 x1 y 2 = 5 and
y1 = r cos and y 2 =
5
x1 y1 + 5 x 2 y 2 = 105 , we have
r

2 sin r cos 10 cos 5 sin = 2r sin( + ) = 5


.

r
10 cos r cos + 5 2 sin
sin = 10r cos( ) = 105
5

5 105
= 23
This implies r =
+
2 10
Alternate Solution 2
2

Apparently any special case should work. Try x1 = 10, x2 = 0. From x2 y1 x1 y2 = 5, get
y2 =

5
105
105 5 25
+
= 23.
. From x1 y1 + 5 x2 y2 = 105, get y1 =
. Hence y12 + 5 y22 =
10
10
10
10

10. Since 15 = 1 15 = 3 15 are the only factorizations of 15, so a positive integer has exactly 15
positive integer factors if and only if it is of the form p 14 for some prime p , or p 2 q 4 for two
distinct primes p and q . Now 214 > 500 , so there are no numbers of the first form between 1
3

and 500. The only one of the second form are 3 2 2 4 = 144 , 5 2 2 4 = 400 , and 2 2 34 = 324 ,
hence 3 is the answer.
11. Note that ( n + 1) 2 n 2 = 2 n + 1 , hence every odd positive integer is the difference of squares of
integers. Also ( n + 2) 2 n 2 = 4 n + 4 = 4( n + 1) , hence any integer divisible by 4 is the difference
of squares of integers. Now for a number of the form 4n + 2 , suppose 4n + 2 = x 2 y 2 , then x
and y must be of the same parity, hence x 2 y 2 is divisible by 4, while 4n + 2 is not. So the
positive integers that cannot be expressed as the difference of two square integers are exactly
those of the
form 4n + 2 . The 2002th member is 4(2001) + 2 = 8006 .
12. Let AC and BD intersect at O . From ABC , get
OA OB
OB OC
=
, from BCD
, get
=
. If OA = a ,
OB OC
OC OD
OB = ar , then OC = ar 2 , OD = ar 3 . By Pythagorean
theorem , AB = a 1 + r 2 , AD = a 1 + r 6 .
1001 AD 2 1 + r 6
=
=
= r 4 r 2 + 1.
Then 91 =
2
2
11
AB
1+ r
4
2
2
Hence r r 90 = r 10 r 2 + 9 = 0 , or r 2 = 10 , r = 10 .

)(

Since 11 = AB = a 1 + r 2 , get a = 1 , and


BC = OB 2 + OC 2 = 110
Alternate Solution
Assign the coordinates A(0, 11), B(0, 0), C (c, 0) and D(c, d ).
11
d
= 1 , or c 2 = 11d .
As AC BD , we have
c c
Hence, AD 2 = c 2 + (d 11) 2
c2
=c +(
11) 2 = 1001.
11
2

Simplify to get c 4 11c 2 10890 = 0.


Let c 2 = s , and observe that 10890 = 99 110 ,
have ( s 110)( s + 99) = 0 , or s = 110 ,
implying c = 110.

13. Apply cosine rule to BCP , we have


BE 2 + PE 2 PB 2
cos PEB =
,
2 BE PE
PE 2 + EC 2 PC 2
cos PEC =
. Using the fact
2 PE EC
cos PEB = cos PEC ,
and
after
EC
simplifications,
get
PE 2 = PB 2
BC
BE
2
+ PC 2
EC BE ,
or
PE 2 = PB 2
BC
5
3
+ PC 2 6 (1) Apply cosine rule to
5
BP 2 + PC 2 BC 2
DP 2 + PC 2 CD 2
BCD , we have cos CPB =
, cos CPD =
. Again by
2 BP PC
2 DP PC
BC = CD = 5 ,
BP = 5 2 ,
after
simplifications,
get
cos CPB = cos CPD ,
2
2
2
PC = BC BP DP = BC BP(BD BP) = 25 BP 5 2 BP (2). Put (2) in (1), get

PE 2 = PB 2 3 2 BP + 9 (3). Hence PE + PC =

PB 2 3 2 BP + 9 +

PB 2 5 2 BP + 25

3
5 2 25

9
+
= PB
2 + + PB
. Take PB = x , then PE + PC corresponds to the

2
2
2

5 2 5 2

2 , 2 . For PE + PC to be smallest

5 2
3
5 2

2
2

=
2
(x, 0) lies on the line formed by these two points. Hence
, solve to
3
5 2
5 2
x
2
2
2
2
15 2
get x =
. (Corresponds to PE + PC = 34 ).
8
3
3

2,
2 , and
sum of distances from (x, 0) to
2
2

Alternate Solution
Clearly PC = PA, hence PE + PC is smallest when PE + PA is smallest,
i.e. when A, P, E are collinear.
Assign the coordinates A(0,5), B(5,5), C(5,0), D(0,0),
3
then E = (5, 2), and the equation of the line AE is y = x + 5.
5
2

25
25
25 25

This means P = , and PB 2 = 5 + 5


8
8
8 8

15
450
2
=
or PB =
64
8

BDC =
14. From C construct ACD = BAC , with D
on
AB .
Then
BCD = 2DAC . Hence BC = BD = 5 , AD = CD = 6 . Apply cosine rule to
52 + 52 6 2 14
=
. Apply cosine rule to ABC , get
BDC , get DBC =
255
50
24 24 5
154 576
=
=
.
AC 2 = 112 + 52 2(5)(11) cos B = 146
, Hence AC =
5
5
5
5

Alternate Solution
11
5
AC
=
=
.
sin 3 x sin x sin(1800 x )
11 sin 3 x
1
2
=
= 3 4 sin 2 x , hence sin x =
From the first equality, we have
, and cos x =
.
5
sin x
5
5
5 sin 4 x
= 5(4 cos x(1 2sin 2 x))
From the second equality, we have AC =
sin x
2 2
24 5
24
1 =
= (5)(4)
=
5
5
5 5
Let CAB = x < 90 D . By the sine rule, we have

(The compound angle formula sin( A + B ) = sin A cos B + cos A sin B is used repeatedly here.)
15. As shown in the figure, AB = 10 , BC = 2 x and CA = 2 y . D , E and F are midpoints of AB ,
BC and CA respectively, G is the centroid, and further CD AE , we have CD = 9 , from the
CG AG
BG
=
=
= 2 , hence CG = 6 , GD = 3 . Let GE = a , then
property of centroid
GD GE
GF
2
2
2
AG = 2a . By Pythagorean theorem, we have a 2 + 36 = x 2 , (2a ) + 36 = (2 y ) , 9 + (2a ) = 25 .
If follows that a = 2 , x = 2 10 and y = 13 . Let = BCA. Apply cosine rule to BCA , get
2
2
100 = (2 y ) + (2 x ) 2(2 x )(2 y ) cos , so xy cos = 14 . Apply cosine rule to BCF , get
2
BF 2 = y 2 + (2 x ) 2 y (2 x ) cos = 13 + 160 4 14 = 117 . Hence BF = 117 = 3 13 .

Alternate Solution
Choose the coordinates of A , B and C as (0,0), (10,0) and ( b, c ) respectively. Then the
10 + b c
b c
coordinates of D, E and F are (5, 0), (
, ) and ( , ) respectively.
2
2
2 2
As CD = 9, hence (b 5) + c 2 = 81 , or b 2 + c 2 =10b + 56.(1)
2

Also CD AE , get

c
c

= -1, or b 2 + c 2 = -5b + 50.(2)


10 + b b 5

(1) and (2) imply b =


2

2
.
5
2

1
1
c b

Finally BF 2 = + 10 = c 2 + (b 20) 2
4
4
2 2

1
1
= (b 2 + c 2 ) 10b + 100 = (52) + 4 + 100 = 117 .
4
4
Hence BF = 117 = 3 13

16. Let x be the number of blue vertices and y be the number of blue faces in a coloring. If x = 0 ,
we have 5 different ways to color the faces, namely y = 0, 1, 2, 3, 4. If x = 1 , there are 2 ways to
color the face opposite to the blue vertex and the remaining three faces may have 0, 1, 2 or 3 blue
faces. Thus there are 2 4 = 8 different ways in this case. If x = 2 , call the edge connecting the
two blue vertices the main edge and the two faces common to this edge the main faces .
There are 3 ways to color the main faces, namely, both red, both blue, and one red and one blue.
In the former two cases there are 3 ways to color the remaining faces, while in the last case there
are four because reversing the color of the two faces does matter. Hence there are 10 ways in this
case. By symmetry, there are 8 ways corresponding to the case x = 3 and 5 ways corresponding
to x = 4 . Hence the answer is 5 + 8 + 10 + 8 + 5 = 36 .
1
3 . Define = tan 1 a , then the above is
17. Rewrite the recurrence relation as an =
n
n
1
1
an1
3
an1 +

6 = tan + . Thus = + (modulo if


equivalent to tan n =
n1
n
n1

6
6

1 tan tan n1
6
necessary)
and
thus
an+6 = tan n+6 = tan ( n + ) = tan n = an .
It
follows
that
tan n1 + tan

a2002

2
2+ 3
2
5 3 8

3
= a4 = tan 4 = tan 0 +
=
=
=
.

2 1 ( 2 ) 2
3
11

1 ( tan 0 ) tan

tan 0 + tan

18. Since 2002 = 2 7 11 13 , we can choose certain vertices among the 2002 vertices to form
regular 7-, 11-, 13-, " polygons (where the number of sides runs through divisors of 2002
greater than 2 and less than 1001). Since 2002 = 7 286 , so there are at least 286 different
positive
integers
among
the
ai ' s .
Hence
the
answer
is
at
least
7 286 287
= 287 1001 = 287287 . Indeed this is possible if we assign
7 (1 + 2 + " + 286 ) =
2
i
the number + 1 to each Ai , for i = 1, 2, ", 2002.
7

19. Let the five given points be A, B, C, D and E. The number of straight lines constructed in step 1
is C25 = 10 . From each of the five pints A, B, C, D and E, a total of C24 = 6 perpendicular lines
can be drawn. It seems that the number of intersection points should be C230 = 435 . However for
any straight line, say AB, the three perpendicular lines drawn from C, D and E are parallel, thus
will not meet. Hence the number of intersection points should be cut down by 10 C23 = 30 . Next
for any of the C 35 = 10 triangle, formed, say ABC, there are three perpendicular lines which are
concurrent. Hence the number of intersection points should be further cut down by
10 C23 1 = 20 . Finally the points A, B, C, D and E, each represents the intersection of six lines
(lines drawn from each of them), hence the number of intersection points should be cut down by
5 C26 1 = 70 . The required number of points of intersection is therefore
435 30 20 70 = 315 .

20. Let ABCD be the piece of paper, with


AB = a b = BC . Let O be the center of the
rectangle, B be folded to coincide with D , and
POQ be the crease, with P on BC and Q on DA .
Then PQ is the perpendicular bisector of BD , and
BPO ~ DBC . Let x be the length of PQ , then
1 2
a + b2
b
a 2
2
= , or x =
a + b 2 . Now
1
b
a
x
2

a 2
a2
a + b 2 = 65 , thus b =
. For
b
652 a 2
b to be an integer, the denominator 65 2 a 2 must be an integer. Hence we try 65 2 as a sum
two
squares.
Using
and
we
have
13 2 = 5 2 + 12 2
5 2 = 32 + 4 2

x=

of

65 2 = (13 3) + (13 4 ) = (5 5 ) + (12 5 ) = (3 12 + 4 5 )2 + (4 12 3 5 )2 = (3 12 4 5 )


2

2
+ (4 12 + 3 5 ) . Consider all these a which may be 16, 25, 33, 39, 52, 56, 60 or 63. Put these to find

b , and bear in mind b is an integer with b a , we get a = 60 , and b = 144 the only possibility. Thus
the perimeter is 2 (60 + 144 ) = 408 .

~ End of Paper ~

2002

2001 5 27
3

1. 1 200120012001
2001729
11 n



n 2001

2. 1 8 11 2
$875 3 2 5 $343 1
1 1
3. 1 n F ( n ) n Tn = F ( n 2 ) F ( n )
T1 + T2 + T3 + + T2002
4. 1 22002 ( 22003 1)
5. 1 xyz
x 2 + y 2 z 2 = 3 xy x 2 y 2 + z 2 = 2 xz

y : z
6. 1 b x 6 2bx 3 + b 2 100 = 0

2 b
7. 1 ABC BAC = 40 ABC = 60D E AC AB

CBD = 40 BCE = 70 BD CE FAF BC G GFC


8. 2 ABCD PQRS ABBCCDDA

Taught By Mr. Sophorn ROEUNG, Dir. READ AND THINK,

(012-013) 47 36 17 & 099 72 3003

AP BQ CR DS
=
=
=
= k < 1
PB QC RD SA

PQRS ABCD 52% k


9. 2 xy x 2 + 4 y 2 = 0 x 2 + 2 xy + 4 y 2 + x + 2 y

Taught By Mr. Sophorn ROEUNG, Dir. READ AND THINK,

(012-013) 47 36 17 & 099 72 3003

10. 2

x y
z
x y + z = 2002
11. 2 ABC M N BC BM < BN BM = NC = 4MN = 3

BAM = MAN = NAC AC


12. 2 10
(a) 1 2
(b) 1
13. 2 x y 5 107 > x > y x 2001 y 2001

d x y x d
14. 2 [x] x x + 1 1 =

x [ x]
x
x 1

15. 2 ABCD 1EFGH ABBCCDDA


20
AE = BF = CG = DH =
AFBGCHDE
21

16. 2 a a ' b b ' c c ' d d ' ( a, b, c, d ) ( a ', b ', c ', d ' )

( a , b, c , d )

abcd

1 a < b < c < d 30 a + d = b + c


1
7 4649
p
1

= 0.00021510002151 p
p
= 0.0002151
4649

17. 2 p

4
10
4
2

3
3
3

18. 3

Taught By Mr. Sophorn ROEUNG, Dir. READ AND THINK,

(012-013) 47 36 17 & 099 72 3003

Taught By Mr. Sophorn ROEUNG, Dir. READ AND THINK,

(012-013) 47 36 17 & 099 72 3003

International Mathematical Olympiad 2002


Hong Kong Preliminary Selection Contest
Outline of solutions
Answers:

1.
5.

8
2 :1

2.

$112

3.

4.

28

6.

6 3

7.

20

8.

1
841

9.

2 (3 + 2 )

10.

30030

11.

12.

880

13.

16026010

14.

15.

2
3

16.

1925

17.

239

18.

4
15

Explanations:
1.

Note that the sum of the odd place digits = 2n + 16 while the sum of the even place digits = n + 2 . The difference is
equal to n + 14 . For the number to be divisible by 11, this difference must be a multiple of 11, and the smallest n having
this property is n = 8 .

2.

Let the cost of each armchair, bookcase and cabinet be $x, $y and $z respectively. Then we have 8 x + 11 y + 2 z = 875 and
3 x + 2 y + 5 z = 343 . Setting u = 2 x + 3 y and v = x + y + z , we get 3u + 2v = 875 and u + 5v = 343 . Solving, we
have u = 217 and v = 112 . So the answer is $112.

3.

4.

Note that T1 + T2 + + T10 = 0 . Similarly, we have T11 + T12 + + T20 = 0 , T21 + T22 + + T30 = 0 , and so on, and
T1991 + T1992 + + T2000 = 0 . Hence the answer is equal to T2001 + T2002 , which is 2.
2
Note that 2 2002 ( 22003 1) = 4 16500 ( 8 16500 1) = 4 8 (16500 ) 16500 7 + 28 = 4 [8 16500 + 7 ] [16500 1] + 28 .

In addition, each of the numbers in the square brackets is divisible by 5, so the first summand is divisible by 100. Hence
the answer is 28.
Alternatively, observing the pattern of the last 2 digits of powers of 2, we see that there is a cycle of period 20 starting
from 2 2 . So the answer is the last two digits of 2 2 ( 23 1) , which is 28.
5.

Let x, y and z be the lengths of the sides of a XYZ with x, y, z be respectively the lengths of the sides opposite X, Y and Z.
Rewriting the given equations as z 2 = x 2 + y 2 2 xy cos 30 and y 2 = x 2 + z 2 2 xz cos 45 , and comparing with the

cosine law, we get Y = 45 and Z = 30 . By sine law, we have y : z = sin Y : sin Z = sin 45 : sin 30 = 2 :1 .
6.

Rewriting the equation as ( x3 b ) = 102 , we see that the two real roots are 3 b + 10 and 3 b 10 . We thus get
3
b + 10 3 b 10 = 2 . Cubing both sides and simplifying, we have 20 3 3 ( b + 10 )( b 10 ) ( 3 b + 10 3 b 10 ) = 8 . Note
2

that

b + 10 3 b 10 = 2 , so we have

( b + 10 )( b 10 ) = 2 , and hence b = 6 3 .

Taught By Mr. Sophorn ROEUNG, Dir. READ AND THINK, 1

(012-013) 47 36 17 & 099 72 3003

7.

Let P, Q be the feet of the perpendiculars from A and F to BC,


respectively. We have BCA = 80 and BP = BA cos 60 .
sin 80
Also, by the sine law, we have BA = BC
= 2 BC cos 40 .
sin 40

Hence BP = BC cos 40 . Note also that BFC = 70 , hence


BC = BF . Thus BP = BF cos 40 = BQ . As a result, G, Q and

P are the same point. Clearly, GFC = 20 .


40

8.

70
G

First, note that the enclosed region is in fact a square. To see this,

first note that EBGD and AHCF are parallelograms, so the region
is also a parallelogram. Next, as BCG CDH , we have
CH BG . By the same reasoning, the enclosed region is a

rectangle. Finally, it can be seen that each side of the region has
1
length equal to
cos x where x = CBG (and hence many
21
other corresponding angles as well). So the enclosed region is a
GC 20
21
, cos x =
and the answer
square, and since tan x =
=
29
BC 21

1
1 21
is equal to =
.
841
21 29

9.

Note that y 2 1 . Therefore we may let y = sin . Then we have x = 2 cos and the given expression is equal to
4 + 4 cos sin + 2 ( cos + sin ) . Since 4 sin cos = 2 ( sin + cos ) 2 , the expression can be further simplified as
2
2 ( sin + cos ) + 2 ( sin + cos ) + 2 . As sin + cos = 2 sin ( + 45 ) 2 , the maximum value of the expression
2

is 2 ( 2 ) + 2 2 + 2 = 2 ( 3 + 2 ) . Indeed, this maximum value is attained when = 45 , i.e. x = 2 , y =


2

2
.
2

10. Let the dimensions of the cuboid be a b c , with a, b, c all distinct. We then have x = ( a 2 )( b 2 )( c 2 ) ,
y = 2 [( a 2 )( b 2 ) + ( b 2 )( c 2 ) + ( c 2 ) ( a 2 )] and z = 4 [( a 2 ) + ( b 2 ) + ( c 2 )] . Set p = a 2 , q = b 2
and r = c 2 . Then 2002 = x y + z = pqr 2 ( pq + qr + rs ) + 4 ( p + q + r ) = ( p 2 )( q 2 ) ( r 2 ) + 8 . Now p, q, r are
distinct integers with

( p 2 )( q 2 ) ( r 2 ) = 1994 , so { p, q, r} = {999, 4, 3} . Hence {a, b, c} = {1001, 6, 5}

and so the

answer is 1001 6 5 = 30030 .


11. By symmetry, AB = AC . (An alternative way to see this is to let
A

the lengths of AB, AM, AN and AC be respectively 4x, 3y, 3x and


4y by the angle bisector theorem. Let D on BC such that AD
x y 4 + MD MD

bisects BAC and MAN. Then 1 = =


,
y x 4 + ND ND
which gives ( MD + ND + 4 )( MD ND ) = 0 . Thus MD = ND
and so x = y .)
Now AB = AC = 4 x and AM = AN = 3x . By cosine law,
( 3 x )2 + ( 3x )2 32
( 3x )2 + ( 4 x )2 42
= cos MAN = cos NAC =
2 ( 3x )( 3 x )
2 ( 3x )( 4 x )

Solving, we get x = 2 and AC = 8 .

Taught By Mr. Sophorn ROEUNG, Dir. READ AND THINK, 2

(012-013) 47 36 17 & 099 72 3003

12. Let an denote the number of n-digit numbers with the two properties. Clearly, a1 = 0 and a2 = 1 . If an n-digit number
( n 3 ) with such properties begins with 2, then there must be two consecutive 1s in the last n 1 digits, so there are
an 1 such numbers. Similarly, there are an 2 n-digit numbers with such properties and begin with 12. Finally, there are
2n 2 n-digit numbers with such properties and begin with 11. It follows that for n 3 , an = an 1 + an 2 + 2n 2 . Using
this recurrence relation, we get a3 = 3 , a4 = 8 , a5 = 19 , a6 = 43 , a7 = 94 , a8 = 201 , a9 = 423 , a10 = 880 .

13. Let y = 2001 + n 2 . Without loss of generality, assume n 0 . Then we have x = 2001 + ( n + 1) . Since d | x and d | y ,
we have d | x y = 2n + 1 . Similarly, we get d | 2 y n ( 2n + 1) = 4002 n and d | 2 ( 4002 n ) + ( 2n + 1) = 8005 . So the
2

x < 5 107 , the only possible value of n is 4002. It follows that

maximum value of d is 8005. Since


x = 2001 + 40032 = 16026010 .

14. Let { x} = x [ x ] . Clearly x 1 . We shall consider the following cases separately.


Case 1: x > 1 . The equation becomes x ( x 1) = { x} . Since x ( x 1) > x 1 { x} , so there is no solution in this case.
Case 2: 1 x < 1 . The equation becomes x (1 x ) = { x} . Since both 1 x and { x} are non-negative, so is x. Then
0 x < 1 , so { x} = x , the equation becomes x (1 x ) = x and thus we get a solution x = 0 .
Case 3: 2 x < 1 . The equation becomes ( 2 + x )( x 1) = { x} . As x 1 < 0 and { x} 0 , we must have x = 2 .
Case 4: 3 x < 2 . Using the fact that { x} = x + 3 , the equation becomes ( x + 2 )( x 1) = x + 3 , so x = 5 .
Case 5: x < 3 . The equation becomes x 2 1 x = x [ x ] . The left hand side is at least 1 4 = 4 , but the right hand
side is less than 1. So there is no solution in this case.
Among the three solutions for x, namely, 0, 2 and 5 , the maximum value of x is
15. Let [XYZ] denote the area of XYZ.
[ SAP ] [ SAP ] [ SAB ] AP SA
k
Note that
=

=
.
[ DAB ] [ SAB ] [ DAB ] AB DA ( k + 1)2
Similarly,

[ PBQ ] = [QCR ] = [ RDS ] =

[ ABC ]

[ BCD ]

[CDA]

( k + 1)2

= [ ABCD ]

( k + 1)

.
S
Q

([ DAB ] + [ ABC ] + [ BCD ] + [CDA])


2

( k + 1)

[ PQRS ] = [ ABCD ] ([ SAP ] + [ PBQ ] + [QCR ] + [ RDS ])


= [ ABCD ]

5.

2
( 2 [ ABCD ]) = k + 12 [ ABCD ]

( k + 1)

k +1
2
= 0.52 , and since k < 1 , we get k = .
2
3
( k + 1)

Consequently,

16. First, we have C330 ways to choose a, b, d for which a < b < d . We want to eliminate the possibility that a + d = 2b . In
that case a and d are either both odd or both even, so among the C330 ways 2 C215 of them have the property that

a + d = 2b , i.e. C330 2 C215 = 3850 of them have the property that a + d b b . Each of these combinations uniquely
determines a value of c. Note that such a value of c obtained must be greater than a and smaller then d. However, it may
be greater than or smaller than b. So each (a, b, c, d) is actually counted twice, and hence the answer is 3850 2 = 1925 .
17. Let

1
= 0.xz where x is the non-recurring part with r digits and z is the recurring part with 7 digits. Then we have
p

10r
z
x= 7
, or (107 1)10r = p [ z + (107 1) x ] . Clearly, p does not divide 2 or 5, so p must divide 107 1 . From
p
10 1
the given example we know that 4649 divides 107 1 . Indeed, 107 1 = 32 239 4649 . Clearly, p 3 . So the only
other possible value of p is 239.

Taught By Mr. Sophorn ROEUNG, Dir. READ AND THINK, 3

(012-013) 47 36 17 & 099 72 3003

18. As shown in the figure, denote the piece of paper as ABC with

a = BC =

A
N

4
10
, b = AC = 2 and c = AB =
. Let D be the
3
3

mid-point of BC, E be the foot of the perpendicular from A to BC


and MN be the line perpendicular to BC along which when folded

along the overlapped area is maximum. Clearly, M must lie


C
between D and E. Let C be the reflection of C across MN, and
D M E
B C
AB meet CN at G. Let also DM = x . Then BC ' = 2 x .
a2 + b2 c2
1
a2 + c2 b2
1
Applying cosine law in ABC, we get cos C =
=
and cos B =
=
. Hence C = 45 ,
2ab
2ac
2
10
1
3
1
and sin B =
. In BCG, sin G = sin ( ABC GC ' B ) = sin ( B C ) = sin B cos C sin C cos B =
.
sin C =
2
10
5
BC 'sin B
1
= 3 2 x . Hence the area of BCG is equal to ( BC ')( GC ' )( sin C ' ) = 3 x 2 .
By sine law, C ' G =
2
sin G
On the other hand, the area of CHN is equal to

1a

+ x
2 2

and so the overlapped region MBGN has area


2

1a
5
a
3 2 3 2 4

2
a = ,
+ x 3x = x + a
2 2

2
10 20
20
15
with equality when x =

a
4
=
. Indeed, we can check that this value of x gives a point M between D and E.
10 30

Taught By Mr. Sophorn ROEUNG, Dir. READ AND THINK, 4

(012-013) 47 36 17 & 099 72 3003

International Mathematical Olympiad 2001


Hong Kong Preliminary Selection Contest
(Sponsored by the Quality Education Fund)
May 28, 2000
Time Allowed: 3 Hours
Answer ALL questions.
Put your answers on the answer sheet.
The use of calculator is NOT allowed.
1.

Find the sum of all real x satisfying (2 x 4)3 + (4 x 2)3 = (4 x + 2 x 6)3 .

2.

In how many ways can 30be expressed as the product of two integers p and q such that

0<

p
< 1 and p, q are relatively prime.
q

(1 mark)

4.

Find the coefficient of x17 in the expansion of (1 + x 5 + x 7 ) 20 .


If [x] represents the greatest integer less than or equal to x, find the sum of

5.

1 1999 2 1999 3 1999


2000 1999
.
2001 + 2001 + 2001 + +
2001
If x, y are nonzero numbers satisfying x 2 + xy + y 2 = 0 . Find the value of

3.

x+ y

6.

2001

y
+

x+ y

(1 mark)

(1 mark)

For how many real numbers a do the quadratic equations x 2 + ax + 8a = 0 have only integral
(1 mark)

Suppose tan and tan are the roots of x 2 + x + 2 = 0 . Evaluate


sin 2 ( + ) + sin( + ) cos( + ) + 2 cos 2 ( + ) .

8.

(1 mark)

2001

roots?
7.

(1 mark)

(1 mark)

2000 lamps are controlled by 2000 switches, numbered 1, 2, 3, , 2000. A click on each
switch will either turn the lamp on or off. In the beginning, all the lamps are off. On the first
day, all the switches are clicked once. On the second day, all the switches numbered 2 or a
multiple of 2 are clicked once. Similarly on the nth day, all the switches numbered n or a
multiple of n are clicked once, and so on. How many lamps will be on after the operation on
the 2000th day?

9.

(1 mark)

Point B is in the exterior of the regular n-sided polygon A1 A2 An and A1 A2 B is an


equilateral triangle. Find the largest value of n such that An , A1 and B are consecutive
vertices of a regular polygon.

(1 mark)

10. There are three parallel lines L1 , L2 and L3 on the plane, with L2 in between. The
distance between L1 and L2 is 4, and the distance between L2 and L3 is 3. A, B and C
Taught By Mr. Sophorn ROEUNG, Dir. READ AND THINK,

(012-013) 47 36 17 & 099 72 3003

are points on L1 , L2 and L3 respectively, such that ABC is an equilateral triangle. Find
the area of the triangle.

(1 mark)

11. A circle is inscribed in ABC . D, E are points on AB and AC respectively, such that DE is
parallel to BC and is tangent to the circle. If the perimeter of ABC is p, find the maximum
length of DE.

(1 mark)

12. In ABC , BC = 5 , AC = 12 , AB = 13 . D, E are points on AB and AC respectively such


that DE divides ABC into two parts of equal area. Find the minimum length of DE.

(1 mark)

13. D is a point inside ABC . PDS, QDT and RDU are lines parallel to BA, CA and CB
respectively such that P, Q lie on BC, R, S lie on CA, and T, U lie on AB. If the areas of
TUD , PQD and RSD are respectively 8, 128 and 32, find the area of ABC .

(1 mark)

14. The numbers x1 , x2 , , x2000 are such that x1 x2 + x2 x3 + + x1999 x2000 = 2000 . Find
the largest value of y1 y2 + y2 y3 + + y1999 y2000 , where
yk =

x1 + x2 + + xk
, for k = 1, 2, , 2000 .
k

(2 marks)

15. There are n distinct points on a plane. Eight different circles C1 , C2 , , C8 are drawn such
that C1 passes through one of the points, C2 passes through two of the points, C3 passes
through three of the points, and so on. Find the minimum value of n.

(2 marks)

16. Let S denotes a finite sequence of the letters a and b, and f denotes a function defined by
f ( S ) = the new sequence formed by changing all as to a, b and all bs to b, a. For
example, f (b, a, a, b) = (b, a, a, b, a, b, b, a ) , and the number of pairs of consecutive bs in
f (b, a, a, b) is 1. If f ( n ) ( S ) denotes f ( f ( ( S ) )) (n times), find the number of pairs of
consecutive bs in f ( n ) ( a ) .

(2 marks)

17. The circumcircle of the isosceles triangle ABC has AB as a diameter. There is a circle
tangent to BC at its midpoint E and tangent to the minor arc BC at F. If AB = 4 , find the
length of the tangent from A to .

(2 marks)

18. In ABC , BC, CA and AB are divided by P, Q and R respectively in the same ratio. AP
intersects BQ at X, BQ intersects CR at Y, and CR intersects AP at Z. Each of the areas of

ARZ , BPX , CQY and XYZ equals 1 cm2. Find the area of the quadrilateral PCYX. (3 marks)
19. B is a point on the line segment AC such that AB = 1 and BC = 3 . Semicircles 1 , 2 , 3
are drawn with diameters AC, AB, BC respectively, and all are on the same side of AC. Let E
be on 1 such that EB AC . Let U on 2 , V on 3 be such that UV is a common
tangent to 2 and 3 . Find the ratio of the area of EUV over the area of EAC .

(3 marks)

20. In each of 12 photographs, there are 3 women; the woman in the middle is the mother of the
person on her left and is a sister of the person on her right. The women in the middle of the 12
photographs are all different persons. Determine the smallest number of different persons in
Taught By Mr. Sophorn ROEUNG, Dir. READ AND THINK,

(012-013) 47 36 17 & 099 72 3003

the photographs.

(3 marks)

End of Paper

Taught By Mr. Sophorn ROEUNG, Dir. READ AND THINK,

(012-013) 47 36 17 & 099 72 3003

International Mathematical Olympiad 2001


Hong Kong Preliminary Selection Contest
(Sponsored by the Quality Education Fund)
Solutions
1.

(1 mark) Find the sum of all real x satisfying (2 x 4)3 + (4 x 2)3 = (4 x + 2 x 6)3 .
Solution: Let a = 2 x 4, and b = 4 x 2, then the equation becomes
a3 + b3 = (a + b) 3 , giving 3ab(a + b) =0
Either a=0, or b = 0, or a + b = 0
i.e., 2x 4 = 0, or 4x 2= 0, or 4x + 2 x 6 = (2 x + 3)( 2x 2) = 0
Get x = 2, 1/2 or 1. The sum is 7/2 .

2.

(1 mark) In how many ways can 30! be expressed as the product of two integers p
p
and q such that 0 < < 1 and p and q are relatively prime.
q
Solution: There are 10 prime factors of 30!, namely 2, 3, 5, 7, 11, 13, 17, 19, 23, 29.
Actually, 30! = 2 x 3 x 57 x 74 x112 x 13 x 17 x 19 x 23 x 29.
Each of these (e.g. 2 and 3), can only appear in one piece in either p or q,
this gives 1024 choices. Half of them (so that p < q) will be 512 choices.
Considering negative integers as well, there are 1024 choices.

3.

(1 mark) Find the coefficient of x17 in the expansion of (1 + x5 + x7 )20 .


Solution: x17 can only be obtained by multiplying two x5 s and one x7 . There are 20
ways to get x7 and 19 C2 = 171 ways to get two x5 s in the remaining 19 factors. So the
answer is 20 X 171 = 3420.

4.

(1 mark) If [x] represents the greatest integer less than or equal to x, find the sum of
11999 2 1999 3 1999
2000 1999
2001 + 2001 + 2001 + L +
.
2001
Solution: Note that if n is an integer and x is not an integer, then
[n + x] = n + [x] and [-x] = -1 [x]
2000 x 1999
1999
1999
Hence [ ---------------] = [1999 - ---------- ] = 1999 1 [-------- ]
2001
2001
2001
Therefore

1 x 1999
2000 x 1999
[------------] + [ -----------------] = 1999 1 = 1998
2001
2001

Similarly

2 x 1999
1999 x 1999
[------------] + [ -----------------] = 1999 1 = 1998
2001
2001
3 x 1999
1998 x 1999
[------------] + [ -----------------] = 1999 1 = 1998
2001
2001

International Mathematical Olympiad 2000


Hong Kong Preliminary Selection Contest
30th May, 1999.
Time allowed:

3 hours

Answer ALL questions.


Put your answers on the answer sheet.
The use of calculators is prohibited.

19k + 99k
for k = 1, 2, 3, Find k such that Ak is greatest.
k!

1.

Let Ak =

2.

Let x, y, z be real numbers satisfying

3.

A solid pyramid VABCD, with a quadrilateral base ABCD, is to be coloured on each of the five faces such

(1 mark)

1
1
1 1
1
1 1
1
1
+
= , +
= , +
= . Find x.
x y+z 2 y z+ x 3 z x+ y 4

(1 mark)

that no two faces with a common edge will have the same colour. If five different colours are available,
what is the number of ways to colour the pyramid?

(1 mark)

4.

Let x, y be integers satisfying y 2 + 3x 2 y 2 = 30 x 2 + 517 . Find 3x 2 y 2 .

(1 mark)

5.

The digits used to number the pages of a book were counted, and the total number of digits used was 2001.
Find the number of pages in the book.

6.

(1 mark)

There exist integers n with the property that n! may be expressed as the product of (n 3) consecutive
integers. For example, 6! = 10 9 8 . Find the largest integer satisfying this property.

7.

(1 mark)

5 + 6 x 15 x 7
Find all possible x that satisfy the equation
= 5 . Here [u] denotes the greatest integer less
8
than or equal to u.

8.

(1 mark)

Determine the number of acute-angled triangles with consecutive integer sides and of perimeter not
exceeding 100.

9.

(1 mark)

ABCD is a convex quadrilateral with AD parallel to BC, AC perpendicular to BD, and AC = 5. CF is the
perpendicular from C to AD and CF = 4. Find the area of ABCD.

(1 mark)

10. A, B, C and D are four vertices of a cyclic quadrilateral, AC and BD meet at E. If BC = CD = 4, AE = 6, and
BE and DE are integers, find BD.

(1 mark)

11. A sequence of integers a1 , a2 , , an , is defined by an = an 1 an 2 for n 3 . The sum of the first


1588 terms of the sequence is 1997, and the sum of the first 1997 terms is 1588. Find the sum of the first
1999 terms of the sequence.

Taught By Mr. Sophorn ROEUNG, Dir. READ AND THINK, 1

(2 marks)

(012-013) 47 36 17 & 099 72 3003

n n
n
12. Find the integer n satisfying + + + = 1999 . Here [x] dented the greatest integer less than
1!
2!

10!
or equal to x.

(2 marks)

13. What is the largest positive integer n for which there is a unique integer k such that

8
n
7
<
< ?
15 n + k 13

(2 marks)

14. The pages of a book are numbered from 1 to n. A boy adds up all the page numbers and gets the sum of
1922. It is known that by mistake he adds a certain number twice. What is that number?

(2 marks)

15. Four consecutive even integers are removed from the sequence of integers 1, 2, , n, and the average of
the remaining numbers is 51.5625. Determine the largest integer removed.

(2 marks)

16. The sequences 1, 3, 4, 9, 10, 12, 13, consists of all those positive integers, listed in increasing order,
which are powers of 3 or sums of distinct powers of 3. Find the 200th term of the sequence.

(2 marks)

17. In ABC, AB = AC = 12, P is a point on BC such that AP = 8. Find PB PC .

(2 marks)

18. Let ABCD be a rhombus with AB = 5. Suppose AC 6 BD , determine the maximum value of AC + BD .

(2 marks)

19. A square ABCD of side 12 is divided into 144 unit squares. A circle of radius 6 is drawn touching all the
four sides of ABCD. Find the number of unit squares which lie completely within the circle.
20. Determine the minimum value of k, so that for any k numbers chosen from 1, 2, , 91, there exists two of
2 p 3
them, p and q, such that .
3 q 2

(2 marks)

(2 marks)

END OF PAPER

Taught By Mr. Sophorn ROEUNG, Dir. READ AND THINK, 2

(012-013) 47 36 17 & 099 72 3003

International Mathematical Olympiad 2000


Hong Kong Preliminary Selection Contest
Outline of solutions
Answers:

1.

98

2.

23
10

3.

420

4.

588

5.

703

6.

23

7.

7 4
,
15 5

8.

29

9.

50
3

10.

11.

1179

12.

1165

13.

112

14.

31

15.

28

16.

2943

17.

80

18.

14

19.

88

20.

10

Explanations:
1.

Note

that

Ak < Ak +1

19 k + 99k 19k +1 + 99k +1


<
( k + 1)(19k + 99k ) < 19k +1 + 99k +1 19k (k 18) + 99k (k 98) < 0 ,
k!
(k + 1)!

which is true for k = 1, 2, , 97 but false for k 98 . Hence Ak is increasing for k = 1, 2, , 98 and then becomes
decreasing. Thus the sequence is greatest when k = 98.
2.

Taking reciprocals, we have

x( y + z )
y ( z + x)
z( x + y)
=2,
= 3 and
= 4 . Summing up and dividing by 2, we have
x+ y+z
x+ y+z
x+ y+z

xy + yz + zx 9
yz
5
zx
3
= . Subtracting the first three equations in turn from the last one, we have
= ,
= ,
2
x+ y+ z
x+ y+z 2 x+ y+z 2
xy
1
5
= . Dividing one of these equations from another, we get y = x , z = 3 y = 5 x . Putting back into the
x+ y+z 2
3
original equation, we get x =
3.

23
.
10

Let the five colours be Ci for i = 1, 2, 3, 4, 5. Start with the base ABCD and the faces VAB, VBC. They can be coloured
in any three different colours (say C1 , C2 , C3 respectively). This gives 5 4 3 = 60 ways. The other two faces VCD
and VDA can be coloured in the following seven ways:
VCD
C2 C2 C2 C4 C4 C5 C5
VDA
C3 C4 C5 C3 C5 C3 C4
Hence the answer is 60 7 = 420 .

4.

Rewrite the equation as ( y 2 10)(3 x 2 + 1) = 517 10 = 507 = 3 132 . Since 3 x 2 + 1 is positive, y 2 10 = 1 , 3, 13, 39,

169 or 507, i.e. y 2 = 11 , 13, 23, 49, 179 or 517. For y to be an integer, we must have y 2 = 49 . This corresponds to
y 2 10 = 39 and 3 x 2 + 1 = 13 . Hence 3 x 2 y 2 = 12 49 = 588 .
5.

For pages 1 to 9, 9 digits were used. For pages 10 to 99, a total of 90 2 = 180 digits were used. This left with
1812
2001 9 180 = 1812 digits, which can be used in
= 604 pages. Thus the answer is 9 + 90 + 604 = 703 .
3

Taught By Mr. Sophorn ROEUNG, Dir. READ AND THINK, 1

(012-013) 47 36 17 & 099 72 3003

6.

Suppose the (n 3) integers begin with k, then n ! = k (n + k 4) implies 2 ( k 1) = (n + 1) (n + k 4) .


Since 2 3 4 = 24, so k = 5, n = 23 is possible. For k = 6, 2 3 4 5 (n + 1)(n + 2) .
For k 7 , n 24 , we have (2 3 4 5) [6 (k 1)] < 25 26 [27 (k + 20)] (n + 1) ( n + k 4) .
So there is no solution for n 24 .

7.

8.

15 x 7 5 + 6 x 15 x 7
41
9
1 15 x 7 13
15 x 7 5 + 6 x
. Since

<
+1
<x <

=
is
5
8
5
90
10
15
5
10
5
8
15 x 7
7
4
an integer,
or .
= 0 or 1, giving x =
5
15
5
The equation implies

Let the sides of the triangle be (n 1) , n and (n + 1) . Then we have (n 1) + n + (n + 1) 100 and (n 1) + n > n + 1 .
These imply 3 n 33 . For the triangle to be acute, n 2 + (n 1) 2 (n + 1)2 > 0 n > 4 . So n = 5, 6, , 33 have the
required properties, and the answer is 29.

9.

Construct CE // BD to meet AD produced at E. Since AC


= 5, CF = 4, we have AF = 3. Note that ACE = 90, so
3 AF AC
5
20
ACF ~ AEC and
.
=
=
=
CE =
4 CF CE CE
3
Hence [ABCD] = [ABC] + [ADC] = [ABC] + [CDE] =
1 20
50
[ACE] = 5 =
.
2 3
3

10. Let EC = x, BE = y and ED = z. Note that EDC = DBC = CAD, so DCE


CD EC
4
x
~ ACD. Hence
, or
=
= . This gives x = 2. By the triangular
CA DC
6+ x 4
inequality, y + z < 4 + 4 = 8 . Since ABCD is a cyclic quadrilateral, by the

power chord theorem, yz = 6 x = 12 . So {y, z} = {3, 4} and BD = x + y = 7.

6
y
B

E
x

C
11. Let a1 = x and a2 = y . Then a3 = y x , a4 = x , a5 = y , a6 = x y , a7 = x and a8 = y . Hence the sequence
is a cyclic period of 6. Note also that the sum of any 6 consecutive terms is zero. Let Sn denote the sum of the first n
terms. Then 1997 = S1588 = S6264 + 4 = S4 = 2 y x and 1588 = S1997 = S6332 + 5 = S5 = y x . Solving, x = 1179 and
y = 409 . Thus S1999 = S6333 +1 = S1 = x = 1179 .
12. Clearly n < 7! . Let n = a 6! + b 5! + c 4! + d 3 + e 2! + f 1! with a 6 , b 5 , c 4 , d 3 , e 2 and f 1 .
Putting this into the original equation, we get 1237 a + 206b + 41c + 10d + 3e + f = 1999 . Solving for integer values of the
variables, we get (a, b, c, d, e, f) = (1, 3, 3, 2, 0, 1). As a result, n = 1165 .
13. Taking reciprocals, we have

13 n + k 15
6 k 7
<
< . Subtracting one from each term, we get
< < . Multiplying each term
7
n
8
7 n 8

by 56n, we have 48n < 56k < 49n . If n > 56 2 = 112 , there are at least two multiples of 56 between 48n and 49n. For
n = 112 , k is unique since 48 112 = 56 96 < 56 97 < 56 98 = 49 112 .
14. Note that 1 + 2 + + 61 = 1891 < 1922 and 1 + 2 + + 62 = 1953 > 1922 . Hence the book consists of 61 pages, and the
page number mistakenly added twice is 1922 1891 = 31 .

Taught By Mr. Sophorn ROEUNG, Dir. READ AND THINK, 2

(012-013) 47 36 17 & 099 72 3003

15. Let S be the sum of the remaining numbers. Then S = (n 4) 51.5625 , or 16 S = 825(n 4) . Hence n 4 (mod 16).

(1 + 2 + + n) n (n 2) (n 4) (n 6)
(1 + 2 + + n) 2 4 6 8
. Solving the inequalities and
51.5625
n4
n4
using the fact that n is an integer, we have 99 n 106 . Thus n = 100. Let x be the largest integer removed. Then
(1 + 2 + + 100) x ( x 2) ( x 4) ( x 6) = 51.5625 (100 4) . This gives x = 28 .

Also,

16. In base 3, the sequence is 1, 10, 11, 100, 101, 110, 111, Since 200 in base 2 is 11001000, the 200th term is 11001000
in base 3, or 2943.
E

17. Produce BA to E and take D on AC such that AE = AD = AP = 8. Note


that APD = ADP, AEP = APE and BAP = 2 AEP. Now
APD = APC CPD = B + 2 AEP CPD, ADP = C +
CPD = B + CPD. Since APD = ADP, we have AEP = CPD.
CP BE
. As a result, we have
Thus CPD ~ BEP and so
=
CD BP
PB PC = CD BE = (12 8)(12 + 8) = 80 .

D
B

18. Let ABD = . Then AC + BD = 10(sin + cos ) = 10 2 sin( + 45) .

3
3
< 90 . Hence 90 < 45 + cos 1 + 45 < 135 .
5
5
Therefore, the maximum value of sin( + 45) is maximum when
Now 45 < cos 1

= cos 1 , and the maximum value of AC + BD is 10 + = 14 .


5
5 5

B
19. By symmetry, we need only consider the northeast quadrant. Suppose that the circle is centred at (0, 0). For the 36 unit
squares, consider the coordinates of their northeast vertices (p, q). The square lies completely inside the circle if and only
if (p, q) lies inside or on the circle, i.e. p 2 + q 2 62 . Now p can take the values 1, 2, , 6 and the same holds for q. We
find that for the following 14 values of (p, q): (6, 6), (6, 5), (6, 4), (6, 3), (6, 2), (6, 1), (5, 6), (5, 5), (5, 4), (4, 6), (4, 5), (3,
6), (2, 6) and (1, 6), the square does not lie completely inside the circle. Hence, 62 14 = 22 unit squares in the northeast
quadrant lie completely inside the circle. Thus the answer is 22 4 = 88 .
20. Partition the first 91 positive integers into the following sets: A1 = {1} , A2 = {2, 3} , A3 = {4, 5, 6} , A4 = {7, 8, 9, 10} ,
A5 = {11, 12, , 16} , A6 = {17, 18, , 25} , A7 = {26, 27, , 39} , A8 = {40, 41, , 60} and A5 = {61, 62, , 91} . If we

3
2
or smaller than . Hence
2
3
2
k > 9 . For k = 10 , by the pigeon-hole principle, two numbers come from the same set and their ratio lies between
3
3
and . Hence the answer is 10.
2
choose one number from each set, then the ratio of any two of them is either greater than

Taught By Mr. Sophorn ROEUNG, Dir. READ AND THINK, 3

(012-013) 47 36 17 & 099 72 3003

International Mathematical Olympiad 1999


Hong Kong Preliminary Selection Contest
23rd May, 1998.
Time allowed:

3 hours

Answer ALL questions.


Put your answers on the answer sheet.
The use of calculators is prohibited.
1.

In a sequence { a1 , a2 , , an }, a1 = 1 , 4an an +1 = ( an + an +1 1) 2 and a n +1 > a n . Find a1998 .

2.

Given that H k =

3.

A book has 30 chapters. The lengths of the chapters are 1, 2, , 30 pages. Chapter One starts from page 1

k (k 1)
k (k 1)
. Find H 19 + H 20 + + H 98 .
cos
2
2

(1 mark)

(1 mark)

of the book, and each chapter starts from a new page. At most how many chapters can start from an oddnumbered page?
4.

(1 mark)

On a square carpet of size 123 123, each unit square is coloured red or blue. Each red square not lying on
the edge of the carpet has exactly five blue squares among its eight neighbours. Each blue square not lying
on the edge of the carpet has exactly four red squares among its eight neighbours. Find the number of red
squares on the carpet.

5.

(1 mark)

In ABC, C = 90, A = 30 and AB = 1. Let ABD, ACE and BCF be equilateral triangles with D, E,
F lying outside ABC. Let DE intersect AB at G. Find the area of DGF.

6.

7.

(1 mark)

ABCDEF is a regular hexagon. M, N are points on the segments AC, CE respectively such that
AM CN
=
= r . If B, M, N are collinear, find r.
AC CE
ABC is an equilateral triangle and P is a variable point on the same plane such that PAB, PBC and
PCA are isosceles triangles. In how many different positions can P lie?

8.

(1 mark)

The lengths of the three medians AD, BE and CF of ABC are 9, 12 and 15 respectively. Find the area of
ABC.

9.

(1 mark)

(1 mark)

Find the 4-digit number such that, when the order of its digits is reversed, the new value is 4 times the
original one.

(1 mark)

10. Determine the number of ordered pairs (x, y), where x and y are integers satisfying the equation
2 xy 5 x + y = 55 .

(1 mark)

11. Given that [x] represents the greatest integer not exceeding x. Find the last three digits of the integer
10 99
33
.
10 + 3
Taught By Mr. Sophorn ROEUNG, Dir. READ AND THINK, 1

(1 mark)

(012-013) 47 36 17 & 099 72 3003

12. Let x 0 = 5 and x n +1 = x n +

1
for n = 0, 1, 2, . Find the integer closest to x1998 .
xn

(2 marks)

13. At least how many of the + signs in the expression +1 + 2 + 3 + + 100 must be replaced by signs so
that its value is 1998?

(2 marks)

14. Except for the first two terms, each term of the sequence 1000, x, 1000 x , is obtained by subtracting
the preceding term from the one before that. The last term is the first negative term encountered. What
positive integer x produces a sequence of maximum length?
15. In ABC, tan A : tan B : tan C = 1 : 2 : 3 . Find

(2 marks)

AC
.
AB

(2 marks)

16. Find the smallest positive integer n such that 1997 n 1 is divisible by 21998 .

(2 marks)

17. In how many ways can 1998 be expressed as the sum of one or more consecutive integers?

(2 marks)

18. What is the largest integer k such that

1001 1002 1998


is an integer?
11k

(2 marks)

19. Find the smallest multiple of 84 whose digits consist entirely of 6s and 7s only.

(2 marks)

20. An m n p rectangular box has half the volume of an (m + 2) (n + 2) ( p + 2) rectangular box, where
m, n and p are integers and m n p . What is the largest possible value of p?

(3 marks)

END OF PAPER

Taught By Mr. Sophorn ROEUNG, Dir. READ AND THINK, 2

(012-013) 47 36 17 & 099 72 3003

International Mathematical Olympiad 1999


Hong Kong Preliminary Selection Contest
Outline of solutions
Answers:

1.

3992004

2.

40

3.

23

4.

6724

5.

9 3
32

6.

3
3

7.

10

8.

72

9.

2178

10.

16

11.

008

12.

63

13.

17

14.

618

15.

2 2
3

16.

21996

17.

16

18.

100

19.

76776

20.

130

Explanations:
1.

Direct computation yields a1 = 1, a2 = 4, a3 = 9, which suggests an = n2. This is true for n = 1. Suppose an = n2, then
2
4n 2 a n +1 = (n 2 1 + a n +1 ) 2 simplifies to 0 = a n +1 ( 2n 2 + 2)a n +1 + (n 2 1) 2 = [ a n +1 (n + 1) 2 ][a n +1 (n 1) 2 ] . Since

a n < a n +1 , a n +1 = (n + 1) 2 . This completes the induction and hence a n = n 2 for all natural numbers n. It follows that

a1998 = 1998 2 = 3992004 .


k (k 1)
(4m 1)(4m 2)
is even if k = 4m or k = 4m + 1 , and is odd if otherwise. Thus H 4 m 1 + H 4 m =
2
2
4m(4m 1)
(4m + 1)(4m) (4m + 2)(4m + 1)
+
= 4m 1 , and H 4 m +1 + H 4 m + 2 =

= 4m 1 . Hence H 4 m 1 + H 4 m
2
2
2
+ H 4 m +1 + H 4 m + 2 = 2 . Therefore H 19 + H 20 + + H 98 = 20(2) = 40 .

2.

Note that

3.

There are 15 chapters with an odd number of pages, and a chapter after this must start from a page of a different parity.
Thus parity changes at least 14 times. Hence there are at least 7 chapters starting from an even-numbered page, so the
answer is 30 7 = 23 .

4.

Divide the carpet into 41 41 = 1681 squares of size 3 3 , and consider the central unit square of each of these 1681
squares. No matter it is red or blue, we can find 4 red and 5 blue unit squares in each 3 3 square. Hence the total
number of red squares = 4 412 = 6724 .

5.

Note that BC =

1
3
, AC =
, EAG = 90, ECF = 150 and DBF
2
2
3
AG = [ ADG] implies GE = GD.
is a straight line. Then [ AEG ] =
4
1
1
Thus
[ DGF ] = [ EGF ] = [ DEF ] = ([ ABC ] + [ ABD] + [ BCF ] +
2
2

1 3
3
3 3 3
3
3 9 3
[ ACE ] + [CEF ] [ ADE ]) =
+
+
+
+

=
2 8
4 16
16
16
8 32

B
F

G
A

E
Taught By Mr. Sophorn ROEUNG, Dir. READ AND THINK, 1

(012-013) 47 36 17 & 099 72 3003

6.

Without loss of generality, assume that AC = CE = 1 . Then AM = NC = r,

3
. Note that
2
NCB = NCA + ACB = 60 + 30 = 90. Since [NCB] = [NCM] +
1 3 1
1 3
[MCB], we have
r
= r (1 r ) sin 60 +
(1 r ) sin 30 .

2 2 2
2 2

MC = NE = 1 r and the side length of the hexagon is

This gives r =

M
F

3
.
3

N
E

7.

There are 10 possibilities for P as indicated by the 10 dots in the figure.

8.

Let G be the centroid and H be the mid-point of CG. Note that GD =


1
1
12
1

AD = 3 , HD = BD = BE = 4 and GH = CF = 5 . Hence
3
2
23
3

GDH is a right-angled triangle and its area is 6. Now the area of this
1
triangle is
that of ABC, so the area of ABC is 72.
12

F
G
5

3
B

9.

4
D

Let the number be abcd . If a 3 , then abcd 4 is a 5-digit number. Thus a = 1 or 2. But dcba is even, so a = 2.
Now 2bcd 4 = dcb2 , so d 8 , i.e. d = 8 or 9. Since d 4 ends in 2, so d = 8. Hence 2bc8 4 = 8cb2 . Then we have
8000 + 40b + 40c + 32 = 8000 + 100c + 10b + 2 390b + 30 = 60c 13b + 1 = 2c b = 1, c = 7 .

10. Note that 2 xy 5 x + y = 55 ( 2 x + 1)(2 y 5) = 105 . Since 105 has 8 positive factors and 8 negative factors, there are
16 ordered pairs satisfying the equation.

10 99 10 99 + 33 33 66
33
33
2
66
33
2
11. Note that 33
=
10

10
+
3

= 10 3 10 + 3 1 . Thus the last three digits,


33
10 33 + 3
10 + 3 10 + 3
from left to right, are 0, 0 and 8.

Taught By Mr. Sophorn ROEUNG, Dir. READ AND THINK, 2

(012-013) 47 36 17 & 099 72 3003

12. Note that xn +12 xn 2 = 2 +

1
xn2

n 1

, so x n 2 x 0 2 = 2n +
k =0

1997

1
xk

. Thus x1998 2 = 4021 +


k =1

1
xk

> 4021 > 63 2 . On the other

1
1
1
1
2
. So x1998 = 4021 + 2 + 2 + +
+
2

2
n
+
25
xn
x1
x 24
x0
1 1
1
1
1
1
++
+
+
++
< 4021 + 25
+ 200
+
2
2
2
2

25
75
x999 x1000
x1001
x1997

hand, note that x n is an increasing sequence and

<

1
1
1 1
1

+
++
+
+
2
2
2
x 2 x 2

x124 x125
x126
26
25
1
1
675
+ 998
4026.58 < (63.5) 2 . It follows that the answer is 63.
475
2025

13. The original expression is equal to 5050. If x is the sum of all the negative terms after changing signs, then we have
(5050 x ) x = 1998 , or x = 1526. Hence we need to find the smallest number of terms which have a sum of 1526. We
check that 84 + 85 + + 100 = 1564 . Thus the answer 17 by changing the signs attached with the numbers 46, 85, 86,
87, , 100.
14. The sequence is 1000, x, 1000 x , 2 x 1000 , 2000 3 x , 5 x 3000 , 5000 8 x , 13 x 8000 , 13000 21x ,
34 x 21000 , 34000 55 x , . If all these terms are to be non-negative, we get the inequalities x 1000 , x 500 ,
x < 666 , x 600 , x 625 , x > 615 , x < 620 , x > 617 , x < 619 , . It follows that x must be equal to 618.

sin A
4S
where S is
= 2
cos A b + c 2 a 2
1 tan A a 2 + c 2 b 2
. Then
=
=
2 tan B b 2 + c 2 a 2

15. Let a, b and c denote the lengths of the sides opposite vertices A, B and C. Then tan A =
the area of ABC. Similarly tan B =
implies 3b 2 c 2 = 3a 2 , and

and so

4S
a + c2 b2

1 tan A a 2 + b 2 c 2
2 2a
3a
implies b 2 2c 2 = 2a 2 . This gives b =
and c =
,
=
=
3 tan C b 2 + c 2 a 2
5
5

AC b 2 2
.
= =
AB c
3

)(

16. Write n = 2 s q where q is odd. Then 1997 n 1 = 1997 2 1 1997 2


factor is odd, so if 2
s

4S
a + b2 c2

and tan C =

1997 2 1 = 1997 2

s 1

1998

| 1997 1 , then 2

)(

1 1997 2

s 2

1998

| 1997

( q 1)

+ 1997 2

( q 2)

+ + 1 . Note that the second

1 . Now for n to be smallest, we take q = 1 , i.e. n = 2 s . Now

1 (1997 + 1)(1997 1) . Note that the last term is divisible by 4, and all the other
k

terms are congruent 2 modulo 4 because 1997 2 + 1 2 (mod 4). So there are altogether ( s + 2) factors of 2 in the
expression. Hence for n to be smallest, s = 1996 and n = 21996 .

n
(2k + n 1) , so n(2k + n 1) = 3996 . Note that the two factors are of
2
different parity, so exactly one of them is odd. The only odd factors of 3996 are 1, 3, 9, 27, 37, 111, 333 and 999. So there
are totally 16 ordered pairs (n, k) by taking the negative of these 8 odd factors into consideration.

17. Let 1998 = k + (k + 1) + + (k + n 1) =

1998 1000
1998 1000
2
18. Note that from 1000 to 1998, there are
= 91 factors of 11,

= 8 factors of 11 and
11
112

1998 1000
3

= 1 factor of 11 . So the answer is 91 + 8 + 1 = 100 .


113

19. Note that 84 = 3 4 7 , so the last two digits must be 76. To make the number divisible by 3, we need at least 2 more 7s.
However, 7776 is not divisible by 7. So we add one more digit, 6. Among 77676, 76776 and 67776, only 76776 is
divisible by 7. So the answer must be 76776.
20. We have 2mnp = (m + 2)(n + 2)( p + 2) , which gives

(n + 2)( p + 2)
2m
=
> 1 , so m 3 . Next rewrite the equation
m+2
np

Taught By Mr. Sophorn ROEUNG, Dir. READ AND THINK, 3

(012-013) 47 36 17 & 099 72 3003

as 2mnp = mnp + 2(mn + np + pm) + 4(m + n + p ) + 8 mnp 2(mn + np + pm) + 4( m + n + p ) 8 = 8(m + n + p) . Thus
(m 2)(n 2)( p 2) = 8(m + n + p ) . Let a = m 2 , b = n 2 and c = p 2 . Then a 1 and
15
c a + b + 6 ab + 1 + 6
[the inequality follows from the fact that
=1+
abc = 8(a + b + c + 6) implies
=

8
ab 8
ab 8
ab 8
(a 1)(b 1) 0 ]. Hence c can be no larger than 8 16 = 128 , and c attains this value if ab = 9 and

a + b = ab + 1 = 10 , i.e. a = 1 and b = 9. Thus (a, b, c) = (1, 9, 128), or equivalently, (m, n, p) = (3, 11, 130) satisfies the
question. It follows that the answer is 130.

Taught By Mr. Sophorn ROEUNG, Dir. READ AND THINK, 4

(012-013) 47 36 17 & 099 72 3003

International Mathematical Olympiad 1998


Hong Kong Preliminary Selection Contest
June 7, 1997.
Time allowed:

3 hours

Answer ALL questions.


Put your answers on the answer sheet.
The use of calculators is prohibited.
1.

Let a, b, c be real numbers such that


Find the value of

2.

1 1 1
1
1
1
+ + = 2 and 2 + 2 + 2 = 1 .
a b c
a
b
c

1
1
1
.
+
+
ab bc ca

(1 mark)

On a Cartesian plane every point with integral coordinates is called a lattice point. Let Pn be the lattice point
(n, n + 3) and f (n) be the number of lattice points (excluding the two endpoints) on the line segment

joining the origin O and the point Pn. Find f (1) + f (2) + + f (1997) .
3.

(1 mark)

A positive integer is said to be good if the digits of the integer can be divided into two groups such that the
sum of the digits in one group is equal to the sum in the other. Find the smallest positive integer n such that n
and n + 1 are good.

4.

(1 mark)

Let x, y, z be real numbers such that x + y + z = 5 and xy + yz + zx = 3 .


Find the maximum value of z.

(1 mark)

5.

Find the number of diagonals that can be drawn in a convex polygon of n ( n 4 ) sides.

(1 mark)

6.

Find the time between 1:00 p.m. and 1:30 p.m., correct to the nearest minute, when the hour hand and minute
hand of a clock form an angle of 100.

(1 mark)

7.

In ABC, AB = 41 cm, AC = 9 cm and BC = 40 cm. Find the radius of the inscribed circle of ABC.

(1 mark)

8.

Two perpendicular chords of a circle are at distances a and b respectively from the centre. These two chords
divide the circle into four pieces. Consider the sum of the areas of the largest and the smallest pieces, and the

9.

sum of the areas of the other two pieces. Find the difference between these two sums.

(1 mark)

How many integers from 1 to 1997 have the sum of their digits divisible by 5?

(1 mark)

10. The faces of a cube are labelled with different positive integers such that the numbers on any two adjacent
faces differ by at least two. Find the minimum value of the sum of all six numbers.

(1 mark)

11. A square whose sides are of integral lengths is cut into 25 smaller squares whose sides are also of integral
lengths. Exactly 24 of these smaller squares are unit squares. Find the area of the original square.

Taught By Mr. Sophorn ROEUNG, Dir. READ AND THINK, 1

(1 mark)

(012-013) 47 36 17 & 099 72 3003

12. Each of the numbers 1, 2, 3, , 25 is written into a 5 5 table, such that the numbers in each row are in
increasing order. Find the maximum value of the sum of the numbers in the third column.
13. Let S = 1 +

1
2

1
3

++

1
80

. Find the integer n such that n 1 < S < n .

(1 mark)
(2 marks)

14. A positive integer N (in base 10) is composed of the digits 0 and 1 only, and is divisible by 2475. Find the
smallest possible number of digits of N.

(2 marks)

15. 20 football teams take part in a tournament. M matches have been played and it is found that
(a) between any two teams at most one match has been played, and
(b) among any three teams at least one match has been played between two of them.
What is the smallest possible value of M?
16. Let an be the integer closest to

n . Find

(2 marks)

1
1
1
1
+
+
++
.
a1 a2 a3
a1997

(2 marks)

17. ABCDEF is a regular hexagon with AB = 1 . P and S are the midpoints of AB and ED respectively. The circle
with PS as diameter cuts the lines PE and PD at Q and R respectively. Find the area of the quadrilateral QRDE.
(2 marks)
18. Points D and E are points inside an equilateral triangle ABC such that DE = 1 , DA = EA = 7 and

DB = EC = 2 . Find the length of AB.

(2 marks)

19. In ABC, E, F, G are points on AB, BC, CA respectively such that AE : EB = BF : FC = CG : GA = 1 : 3 . K, L,


M are the intersection points of the lines AF and CE, BG and AF, CE and BG, respectively. Suppose the area
of ABC is 1; find the area of KLM.

(3 marks)

20. Three lines are drawn through a point in a triangle parallel to its sides. The segments intercepted on these
lines by the triangle turn out to have equal lengths. Given the triangles side lengths a, b, c, find the length of
the segments.

(3 marks)
END OF PAPER

Taught By Mr. Sophorn ROEUNG, Dir. READ AND THINK, 2

(012-013) 47 36 17 & 099 72 3003

International Mathematical Olympiad 1998


Hong Kong Preliminary Selection Contest

Outline of solutions

Answers:

1.

1.5

2.

1330

3.

549

4.

13
3

5.

n(n 3)
2

6.

1:24 p.m.

7.

8.

4ab

9.

399

10.

27

11.

49

12.

85

13.

17

14.

20

15.

90

16.

3977
45

17.

25 3
338

18.

5 + 13
2

19.

4
13

20.

2abc
ab + bc + ca

Explanations:
1.

1 1 1
1
1
1
1
1
1
1
1
1 3
+ )
+
+
= .
( + + ) 2 = 2 + 2 + 2 + 2( +
a b c
a
b
c
ab bc ca
ab bc ca 2

2.

If ( x , y ) is a lattice point on the (open) segment joining the origin and Pn , then

y n+3
=
ny = x ( n + 3) . If n and
x
n
n + 3 have no common factor greater than 1, then n will divide x, contradicting 0 < x < n . If n and n + 3 have a

common factor greater than 1, the factor is 3 and only two lattice points will be on the segment. So
1997
f (1) + f (2) + + f (1997) = 2[
] = 1330 .
3
3.

Observe that the sum of the digits of a good number is even. So n ends in 9. Also n has more than 2 digits because
99 + 1 = 100 is not good. If n = xy9 (bar means base 10 notation), then n + 1 = x ( y + 1) 0 . So x + y = 9 ,

x = y + 1 x = 5, y = 4 n = 549 .
4.

0 ( x y ) 2 = ( x + y ) 2 4 xy = (5 z ) 2 4(3 z (5 z )) = 3z 2 + 10 z + 13 = ( z + 1)(3z 13) . So 1 z

z=

13
. Then max
3

13
1
when x = y = .
3
3

5.

There are n 3 diagonals through each vertex and each diagonal is counted twice. So there are

6.

At that moment, let x be the minute, then 100 = 6 x 6( 5 +


So x = 23

7
. The time to the nearest minute is 1:24 p.m.
11

Taught By Mr. Sophorn ROEUNG, Dir. READ AND THINK, 1

n ( n 3)
diagonals.
2

x
).
12
6(5 +

x
)
12

6x

(012-013) 47 36 17 & 099 72 3003

7.

Since 9 2 + 40 2 = 412 , ACB = 90 . If r is the radius,


then 41 = ( 40 r ) + ( 9 r ) r = 4 .

A
9r
9r
r

40 r

r
r

8.

40 r

Draw also the mirror images of the chords with respect to the centre.
These 4 chords divide the circle into 9 pieces. The difference between
the two sums is the area of the rectangular piece, 4ab.

9.

r
C
r

a
b
b a

From 1 to 9, there is 1 such integer. From 10n to 10n + 9 , where 1 n 198 , there are two such integers. From 1990 to
1997, there are 2 such integers. So the total is 399.

10. There cannot be three consecutive integers n 1, n , n + 1 (otherwise n 1, n + 1 fall on the face opposite n). So the
minimum possible is 1 + 2 + 4 + 5 + 7 + 8 = 27 with 1, 2 on opposite face, 4, 5 on opposite faces and 7, 8 on opposite
faces.
11. Let m, n be the side lengths of the original and the 25th square respectively. Then 24 = m2 n2 = ( m n )( m + n ) . Since
m n , m + n are positive integers (both odd or both even) and n 1 , the only possibility is m = 7 , n = 5 . Then

m2 = 49 .
12. Let a ( i , j ) be the number in the i-th row, j-th column. By permuting the rows, we
may assume a (i , 3) < a ( k , 3) if i < k . Since a (5, 3) < a (5, 4 ) < a (5, 5) 25 ,

11

12

13

a(5, 3)
is
at
most
23.
Since
a ( 4 , 3) < a ( 4 , 4 ) < a ( 4 , 5)
,
a ( 4 , 3) < a ( 5, 3) < a ( 5, 4 ) < a(5, 5) 25 , a( 4 , 3) is at most 20. Similarly

14

15

16

17

18

19

20

21

22

10

23

24

25

a ( 3, 3), a ( 2 , 3), a (1, 3) are at most 17, 14, 11, respectively. Such a table is
possible

as

shown

on

the

right.

So

the

maximum

23 + 20 + 17 + 14 + 11 = 85.

13. Since

8<

k 1 k =

1
k +1 + k

<

1
2 k

<

1
k + k +1

possible

= k k 1 , adding these with k = 1, 2 , , 80 , we have

1
1
1
(1 +
+ +
) < 80 . It follows n = 17 .
2
2
80

14. Since 2475 = 9 11 25, N is divisible by 9, 11 and 25. Hence the last two digits are both 0, sum of all digits is divisible
by 9, sum of digits in the odd and even positions must differ by a multiple of 11. The smallest possible N has 20 digits
with eighteen 1s followed by two 0s.
15. Suppose team A played the least number of matches, say k. Each of these k teams played with A must each played at least
k ( k + 1)
k matches. This makes a total of at least
matches. For the other (19 k ) teams, by (b), every two of them
2
(19 k )(18 k )
must played with each other, contributing
matches.
2
k ( k + 1) (19 k )(18 k )
So M
+
= k 2 18 k + 171 = ( k 9 ) 2 + 90 90 . Thus the smallest M is 90 with k = 9 .
2
2

Taught By Mr. Sophorn ROEUNG, Dir. READ AND THINK, 2

(012-013) 47 36 17 & 099 72 3003

16. Let k be the nearest integer to n , then k

1
1
n < k + k 2 k + 1 n k 2 + k . So an = k from n = k 2 k + 1 to
2
2

n = k 2 + k , a total of 2k terms.
Then

1 1
1
1
1
1
1 1
1
( + ) + + ( + ) + ( + )
+ + +
= 1 1
44 44
45 45
a1 a2
a1997
[ 2 terms]

17.

[88 terms]

= 2 ( 44 ) + 17 (

[1997-1980=17 terms]

1
17
) = 88 .
45
45

1
13
3
, PS = 3 EP =
and area EPD =
.
2
2
2
Now ESP is similar to EQS, so
ES EQ
1
6
.
=
EQ =
PQ = EP EQ =
EP ES
2 13
13
area QPR
PQ 2
12
72 3
Now
=(
) = ( ) 2 . So area QPR =
area EPD
EP
13
169
25 3
.
and area QRDE =
338
ES =

18. Let the angle bisector of BAC intersect DE at F and BC at G.

By symmetry, AG is the perpendicular bisector to DE and BC.


x 3
1 3 3
Now AF = 7 =
. If AB = x, then AG =
. So
2
4
2

3
5 + 13
x 1
.
( x 3)
= AG AF = FG = 2 2
x=
2
2
2

2 D

x 1
B 2
19. Let [XYZ] denote the area of XYZ.
AE 1
= .
Since [ABC] = 1, [AEC] =
AB 4
[ ACK ] FC
[ ABK ] AB
[ ACK ]
Now
=
=3 ,
=
=4
= 12
[ ABK ] BF
[ AEK ] AE
[ AEK ]
12
3
[ ACK ] = [ AEC ] = .
13
13
3
Similarly, [ BLC ] = [CMB ] = .
13
4
Then [ KLM ] = [ ABC ] ([ ACK ] + [ BLC ] + [CMB]) = .
13

x
2

x 1
2

B
F
L

E
K

20. Let the triangle have vertices A, B, C and P be the point inside.
A

Define A1 to be the intersections of lines AP and BC and


similarly define B1 and C1. Suppose the intercepted segments
through P have length r.
r
AP A1 P [ BPC ]
=
=
Then 1 = 1
.
a
AA AA1 [ ABC ]
Since [BPC] + [CPA] + [APB] = [ABC], we have
r
r
r
2abc
.
(1 ) + (1 ) + (1 ) = 1 r =
a
b
c
ab + bc + ca

Taught By Mr. Sophorn ROEUNG, Dir. READ AND THINK, 3

A1

(012-013) 47 36 17 & 099 72 3003

International Mathematical Olympiad 1997


Hong Kong Preliminary Selection Contest
25th May, 1996.
Time allowed:

3 hours

Answer ALL questions.


Put your answers on the answer sheet.
The use of calculator is prohibited.
Part I
1.

If x =
(a)

2.

Multiple Choices

3.

1 + 1996
, then 4 x 3 1999 x 1997 equals
2

(b) 1

By simplifying

10 + 2

(a)

Each question carries 1 mark.

(c)

(d) 2

(e)

8 + 2 10 + 2 5 + 8 2 10 + 2 5 , one obtains
(b)

2 5+2

(c)

(d)

2 52

10 2

(e)

Let an = n 2 + n + 2 for n 1 , then in the sequence a1 , a2 , a3 , ,

(a)

some an are prime numbers, but there are only finitely many of them.

(b) there are infinitely many an which are prime numbers.


(c)

some an are perfect squares, but there are only finitely many of them.

(d) there are infinitely many perfect squares.


(e) all an are not prime numbers nor perfect squares.
4.

For every positive integer n, the number of integral solutions of x 2 + xy + y 2 = n is a multiple of


(a)

5.

(d)

(c)

(d) 7

(e)

7 f (3) 26
28
35

f (3)
3
3

(b)
(e)

1 f (3) 20
8
13
f (3)
3
3

(c)

4 f (3) 15

Let ABCDEFGHI be a regular polygon with 9 sides, a = AB, b = AC and d = AE, then
(a)
(d)

Part II

7.

(b) 5

If f ( x ) = ax 2 c satisfies 4 f (1) 1 and 1 f (2) 5 , then


(a)

6.

d = a2 + b2
a+d
b=
2

(b)

b = ad

(e)

d = a+b

(c)

d = a 2 + ab + b 2

Short Questions

ABC is a triangle with sides 3, 4 and 5 units. A' is the mirror image of the point A across line BC. Similarly
B' and C are mirror images of B and C across lines CA and AB respectively. Find the area of A'B'C'.

(2 marks)

8.

Let f be a function such that f ( x + y 2 ) = f ( x) + 2( f ( y )) 2 and f (1) 0 . Find the value of f (1996) .

(2 marks)

9.

The integer P is the product formed by multiplying together all the terms of the arithmetic progression 4, 8,
12, , 1996. It is found that (in base 10) P ends with exactly n zeros. Find the value of n.

Taught By Mr. Sophorn ROEUNG, Dir. READ AND THINK,

(2 marks)

(012-013) 47 36 17 & 099 72 3003

10.

In ABC, AB 2 + BC 2 + CA 2 is numerically equal to five times the area of ABC. If AB = 1996, find the
value of cot A + cot B + cot C.

11.

(2 marks)

For all integers x, the function f (x ) satisfies f ( x + 1) =

1 + f ( x)
. If f (1) = 2 , find the value of
1 f ( x)

f (1996) .
12.

(2 marks)

A number is constructed by writing down the positive integers in order starting from 1 until a number of
1996 digits is obtained : 1234567891
0111213141
. Find the remainder when this

5161718192
021


1996 digits

number is divided by 9.

(2 marks)

13.

Find the number of positive integers a < 100 such that a 3 + 23 is divisible by 24.

(2 marks)

14.

ABC is a triangle in which 2 CBA = CAB + ACB. P is a point inside ABC such that APB = BPC
= CPA. Find the length of PB if PA= 18 and PC = 8.

15.

(2 marks)

Circles of radii 1 and 2 are externally tangent to each other and are internally tangent to a circle of radius 3.
The circle of radius 3 has a chord that is a common external tangent of the other circles. Find the length of
this chord.

(2 marks)

16.

Find the largest integer n such that n 2 + 1996n is a perfect square.

(2 marks)

17.

Let an =

1023
log10 (n + 1)
1
p
for n = 2, 3, , 1023. Suppose
= , where the positive integers p and
n = 2 log
log10 n
100 q
an

q have no common factor greater than 1. Find the value of p + q .


18.

(2 marks)

Suppose the sum of squares of two numbers is 468 and the sum of their L.C.M. and H.C.F. is 42. Find the
sum of these two numbers.

(2 marks)

19.

Let n = 219396 . Find the number of positive factors of n2 that are less than n and do not divide n.

(2 marks)

20.

ABCDEFGH is an octagon inscribed in a circle of radius R. AB = BC = CD = DE = 1 and EF = FG = GH =


HA = 3. Find the value of R2 in the form a + b , where a and b are rational numbers.

21.

(2 marks)

Suppose the sequence a1 , a2 , a3 , satisfies the relation an + 2 = an+1 an and a2 = 2 . If the sum of the
first 1996 terms is 2000, then find the sum of the first 2000 terms.

22.

23.

(2 marks)

Let [x] represent the greatest integer not exceeding x. Find the non-integral solution(s) of the equation
96
96
x+
= [ x] +
.
x
[ x]
In ABC, E is the mid-point of side BC and D is a point on side AC. If AC = 1, BAC = 60, ACB = 20
and DEC = 80, find the area of ABC plus twice the area of CDE.

24.

(3 marks)

ABC is isosceles with AB = AC and altitude AM = 11. Suppose that there is a point D on AM with AD =

10 and BDC = 3 BAC. Find the perimeter of ABC.


1996

25.

(2 marks)

Let f (n) be the integer closest to

n . Find
k =1

(3 marks)

1
.
f (k )

(4 marks)

END OF PAPER
Taught By Mr. Sophorn ROEUNG, Dir. READ AND THINK,

(012-013) 47 36 17 & 099 72 3003

International Mathematical Olympiad 1997


Hong Kong Preliminary Selection Contest
Outline of solutions
Answers:
1.

(e)

2.

(a)

3.

(c)

4.

(c)

5.

(b)

6.

(e)

7.

18

8.

998

9.

121

10.

5
4

11.

1
3

12.

13.

14.

12

15.

4
14
3

16.

248004

17.

18.

30

19.

1824

20.

5+

21.

1994

22.

9.6

23.

3
8

24.

11 5 + 11

25.

400

3
2
2

1
7

Explanations:

1.

2.

1 + 1996
, then 0 = (2 x 1) 2 1996 = 4 x 2 4 x 1995 .
2
Thus 4 x3 1999 1997 = ( x + 1)(4 x 2 4 x 1995) 2 = 2 .
If x =

Let

x = 8 + 2 10 + 2 5 + 8 2 10 + 2 5 , then

x>0

and

x 2 = 16 + 2 64 4(10 + 2 5 ) = 16 + 4 6 2 5 =

12 + 4 5 = ( 10 + 2 ) 2 . So x = 10 + 2 .
3.

Since an = n(n + 1) + 2 is even, a1 = 4 , n 2 < an = n 2 + n + 2 < ( n + 1) 2 for n > 1 , only (c) is true.

4.

Note that if (x, y) is a solution, then n = x 2 + xy + y 2 = ( x 2 + 2 xy + y 2 ) xy x 2 + x 2 = ( x + y ) 2 + ( x + y )( x) + ( x) 2 . So


( x + y , x) is also a solution. Using this observation repeatedly, we get solutions (y, x y ), (x, y), ( x y , x),
( y, x + y ) and back to (x, y). This cycle of 6 yields distinct solutions when n is positive by simple checking. So the
number of integer solutions of x 2 + xy + y 2 = n is a multiple of 6.

5.

Since

f (1) = a c , f (2) = 4a c , so a =

between

8
5
f (2) f (1)
f (2) 4 f (1)
, c=
. So f (3) = 9a c = f (2) f (1) is
3
3
3
3

8
5
8
5
( 1) + = 1 and (5) + (4) = 20 .
3
3
3
3

Taught By Mr. Sophorn ROEUNG, Dir. READ AND THINK, 1

(012-013) 47 36 17 & 099 72 3003

6.

Let lines AB and ED intersect at X. From ABC = BCD =


X

CDE = 140, we get BAE = DEA = 60, AEX equilateral.

Now AC = b = BD, AE // BD, so d = a + b .


b

60

60

a
A

7.

Let line AA intersect BC at H, BC at H. Then AH BC, AH BC. Also in lengths, BC = BC, AH = 3 AH. So
the area of ABC is 3 times the area of ABC, which is 3 6 =18.

8.

Since f (0) = f (0) + 2 f (0) 2 , so f (0) = 0 . Then f (1) = 0 + 2 f (1) 2 , f (1) 0 implies f (1) =
So f ( x + 1) = f ( x) + 2 f (1) 2 = f ( x) +

x
1
implies f ( x ) = , thus f (1996) = 998 .
2
2

9.

499 499 499


P = 4499 499!= 2 a 3b 5c 7 d . Now a > c, n = c =
+
+
= 121 .
5 25 125

10.

cot A =

11.

1
.
2

cos A AB 2 + AC 2 BC 2 AB 2 + AC 2 BC 2
AB 2 + BC 2 + CA2 5
=
=
. So cot A + cot B + cot C =
= .
4 area of ABC 4
sin A
2( AB)( AC ) sin A
4 area of ABC

1
1
, f ( 4) = , f (5) = 2 = f (1) imply f ( n + 4) = f (n) .
2
3
1
So f (1996) = f (1992) = = f (4) = .
3
f (1) = 2 , f (2) = 3 , f (3) =

12. The number is equal to 1 101995 + 2 101994 + 3 101993 + + 700 10 4 + 701 10 + 7 .


Since a 10n = a (10n 1) + a has the same remainder as a when divided by 9, the remainder we seek is the remainder of

1 + 2 + + 701 + 7 =
13.

701 702
702
+ 7 which is 7, since
= 351 is divisible by 9.
2
2

a 3 + 23 is divisible by 24 if and only if (a 3 + 23) 24 = a 3 1 = (a 1)[a (a + 1) + 1] is divisible by 3 and 8. Since a or


a + 1 is even, a 3 1 is divisible by 8 if and only if a 1 is divisible by 8. Since a 1 or a or a + 1 is divisible by 3,
it follows a 3 1 is divisible by 3 if and only if a 1 is divisible by 3. So the choices for a are 1, 25, 49, 73, 97.

14. 2 ABC = CAB + ACB = 180 ABC gives ABC = 60. PAB = 180 120 PBA = ABC PBA = PBC.
PA PB
It follows that PAB is similar to PBC. So
and PB = PA PC = 12 .
=
PB PC
15. Let C1 , C2 , C3 be the centres of the circles of radius 1, 2, 3, respectively. Then C1 , C2 , C3 are collinear and

C1C3 = 2C2C3 . Next, let D1 , D2 , D3 be the feet of the perpendiculars from C1 , C2 , C3 to the chord. Then
C D + 2C2 D2 5
4
C1 D1 = 1 , C2 D2 = 2 , C3 D3 = 1 1
=
and the chord has length 2 32 (C3 D3 ) 2 =
14 .
3
3
3
16. If n 2 + 1996n = (n + k ) 2 for a positive integer k, then n =
1996 2k divides k 2 , the largest k is 996 and n =

k2
is largest when k is largest. Since 2k < 1996 and
1996 2k

996 2
= 248004 .
4

Taught By Mr. Sophorn ROEUNG, Dir. READ AND THINK, 2

(012-013) 47 36 17 & 099 72 3003

17. We have an = log n (n + 1) . So

1023

log
n=2

1
an 100

1023

= log100 an = log100 (a2 a3 a1023 ) = log100 (log 2 1024) =


n= 2

1 p
= . It follows
2 q

that p + q = 1 + 2 = 3 .
18. Let a, b be the numbers and m, n be their L.C.M. and H.C.F.. Since a 2 + b 2 = 468 = 6 2 13 , n divides 6. Since

m + n = 42 , (m, n) = (41, 1), (40, 2), (39, 3), (36, 6). Now mn = ab, so (a + b) 2 = a 2 + b 2 + mn = 468 + mn is a square,
which implies m = 36, n = 6, a + b = 30 .
19.

n 2 = 2 28 3192 has (38 + 1) (192 + 1) factors. The number of these factors less than n is

39 193 1
= 3763 .
2

n = 219 396 has (19 + 1) (96 + 1) = 1940 factors, each of which is also a factor of n 2 . So the number is 3763 1939 =
1824.
20. Let O be the centre, then BOH = 90, BAH =

1
( 270) = 135 .
2

Applying cosine law to BAH, we get 2 R 2 = 32 + 12 6 cos 135 = 10 + 3 2 .

21. Since an+3 = an+ 2 an +1 = (an +1 an ) an+1 = an , it follows that the sum of any six consecutive terms is 0. Since
a2 = 2

, 2000 = a1 + + a1996 = a1 + a2 + a3 + a4 = a2 + a3 = 2 + a3

so

a3 = 1998

a1 = a2 a3 = 1996

a1 + a2 + + a2000 = a1 + a2 = 1994 .

96
96
96

= ( x [ x])1
and x [x ] implies x[ x] = 96 . Write x = n + d with 0 < d < 1 ,
22. Note 0 = x + [ x] +
x
[ x]
x
[ x]

then n(n + d ) = 96 . If n > 0, then n 2 < 96 < n(n + 1) which is impossible. If n < 0, then n(n + 1) = 96 < n 2 implies

n = 10 , d = 0.4 , x = 9.6 .
23. Let ACF be equilateral with B on AF and let G be the reflection of B with respect to the bisector of ACF.

3
= [ ACF ] = 2[ ABC ] + [ BCG ] = 2([ ABC ] + 2[CDE ]) .
4

Then

24. Let the bisector of AND intersect AD at E. Since BDM = 3


BAD, it follows that BAE = ABE and AE = BE = x.
Then

DE = 10 x ,

BM = BE 2 EM 2 = x 2 (11 x) 2 =

22 x 121 ,

BD = BM 2 + DM 2 = 22 x 120 , AB =
AB AE
BM 2 + AM 2 = 22 x . By angle bisector theorem,
=
,
BD DE

which implies

22 x
22 x 120

x
55
and x =
. Then the
10 x
8

answer is 2( AB + BM ) = 2( 22 x + 22 x 121) = 11 5 + 11 .
x

D
x

1
1
1
25. Let m be a positive integer, then the largest integer n for which f (n) = m isB xm = [(m + )]4 = (m + ) 4
. So for m
M
2
2
16 C
= 1, 2, , the number of (consecutive) positive integers n satisfying f (n) = m is xm xm1 = 4m 3 + m . Since
6 4 < 1996 < 7 4 , so f (1996) = 6 or 7. Since

(4m3 + m) = 1785 ,
m=1

1996

and

k =1

it follows f (1786) = f (1787) = = f (1996) = 7

6
1
4m 3 + m 1996 1785
1
=
+
= 400 .
f (k ) m=1
m
7
7

Taught By Mr. Sophorn ROEUNG, Dir. READ AND THINK, 3

(012-013) 47 36 17 & 099 72 3003

International Mathematical Olympiad 1996


Hong Kong Preliminary Selection Contest
27th May, 1995.
Time allowed:

3 hours

Answer ALL questions.


Put your answers on the answer sheet.
The use of calculators is prohibited.
1.

In a trapezium ABCD, where AB is parallel to DC, area of ABC : area of ACD = 1 : 3. If E and F are the
area of ABEF
mid-points of BC and DA respectively, find the value of the fraction
.
area of EFDC

2.

What is the maximum possible area of a quadrilateral that has sides of lengths 1, 4, 7 and 8?

3.

Suppose there are n numbers x1 , x2 , , xn , each of them being 0, 1 or 2. If x1 + x2 + + xn = 5 and

x1 + x2 + + xn = 19 , then what is the value of x1 + x2 + + xn ?


2

4.

(1 mark)

(1 mark)

(1 mark)

A snail crawls along a plane, making a 90 turn after every metre travelled. What is the maximum possible
distance between its final and initial positions if the snail has crawled 300 metres, having made 99 left turns
and 200 right turns?

(1 mark)

5.

In a quadrilateral ABDC, AB + BC + CD = 4 and area of ABDC = 2. Find AD.

(1 mark)

6.

Let f1 ( x) =

2x 1
. Define f k +1 ( x) = f1 ( f k ( x)) for k = 1, 2, 3, . Given that f 35 ( x) and f 5 ( x) are the
x +1

same, find f 28 ( 2 ) .
7.

(2 marks)

The function f satisfies f ( x) + f ( y ) = f ( x + y ) xy 1 for every real x, y. If f (1) = 1 , find the negative
integer n such that f (n) = n .

8.

(2 marks)

Find the smallest possible value of n that guarantees the following property: For any set of n integers a1 ,

a2 , , an , there exists two distinct members whose sum or difference is divisible by 1995.
9.

(2 marks)

Of all possible ratios of any two sides of a triangle(including degenerate triangles), denote by q the ratio
nearest to the value one. Find the greatest possible value of | q 1 | .

10. Let a, b, c be the distinct roots of the equation x 3 2 x 2 3 x 4 = 0 . Find

(2 marks)

a 5 b 5 b5 c 5 c 5 a 5
+
+
.
a b
bc
ca

(2 marks)

11. Find the greatest natural number possessing the property that each of its digits except the first and the last
one is less than the arithmetic mean of the two neighbouring digits.

(3 marks)

12. Consider a geometric progression of 5 terms, each a positive integer. The sum of the 5 terms is 211. Denote
by S the sum of those terms in the progression which are squares of integers. What is the value of S?

Taught By Mr. Sophorn ROEUNG, Dir. READ AND THINK, 1

(3 marks)

(012-013) 47 36 17 & 099 72 3003

13. Let x, y and z be distinct integers satisfying


( x y )( y z )( z x) = x + y + z .
Find the minimum value of | x + y + z | .

(3 marks)

14. Find the smallest positive integer which can be expressed as the sum of distinct positive integers a, b and c
such that a + b , a + c and b + c are perfect squares.

(3 marks)

15. The area of ABC is 60. Points D and E trisect side AC (D being nearer A). The median to BC intersects BD
at G and BE at H. Find the area of BGH.

(3 marks)

END OF PAPER

Taught By Mr. Sophorn ROEUNG, Dir. READ AND THINK, 2

(012-013) 47 36 17 & 099 72 3003

International Mathematical Olympiad 1996


Hong Kong Preliminary Selection Contest

Outline of solutions

Answers:

1.

3
5

2.

18

3.

125

4.

100 2

5.

2 2

6.

1 2

7.

8.

999

9.

3 5
2

10.

183

11.

96433469

12.

133

13.

54

14.

55

15.

Explanations:

1.

Let h be the height of the trapezium. Since


So

2.

1
2
1
2

AB h 1
1
=
have CD = 3 AB and EF = ( AB + CD ) = 2 AB .
CD h 3
2

[ ABEF ] 12 ( AB + EF ) h2 3
=
= .
[ EFDC ] 12 ( EF + CD ) h2 5

We may assume the sides of length 1 and 8 are adjacent. (If not, then cut the quadrilateral along a diagonal and flip one of
1
1
the triangles without changing the area.) Now the area of a triangle is
ab sin , hence at most
ab . So the area of the
2
2
1
1
quadrilateral is at most
(1 8) + (4 7) = 18 . Since 12 + 82 = 4 2 + 7 2 , we can compose a quadrilateral of area 18 from
2
2
two right triangles by gluing along the hypotenuse.

3.

Since x1 + x2 + + xn = 5 , at least 3 xi s are 2. If exactly 3 xi s are 2, then exactly one xi is 1 and

x1 + x2 + + xn = 13 , a contradiction. Since 5(2) 2 = 20 > x1 + x2 + + xn , exactly 4 xi s are 2 and exactly 3


2

xi s are 1. So x1 + x2 + + xn = 4(2) 5 + 3(1) 5 = 125 .


5

4.

Since the snail made exactly 99 left turns, its path can be divided into 100 parts (each one containing an integer number
of metres) in such a way that the snail made only right turns along each part (if any). Then the distance between the
end-points of each part is at most

2 . By the triangular inequality, the whole distance is at most 100 2 . Now 100 2

is possible is the snail made the following turns RLRLRLRLRRR (R for right turn, L for left turn, 99 left turns and
one metre between turn.)

Taught By Mr. Sophorn ROEUNG, Dir. READ AND THINK, 1

(012-013) 47 36 17 & 099 72 3003

5.

1
(ab sin + b sin ) = 2 . Since sin ,
2

Let AB = a, BC = b, CD = c, ABC = , BCD = , then a + b + c = 4 and

1
1
(ab + bc) 2 . That is,
b(4 b) 2 , which implies b = 2, = 90 = .
2
2

sin 1 , we get

Then AD = b 2 + (a + c) 2 = 2 2 .

6.

Since

y=

g( 2) =

7.

2x 1
x +1
2 +1

2 2

implies

x=

y +1
,
2 y

f1 ( x) has an inverse function

x +1
. Since
2 x

f 30 ( x) x

and

4+3 2
g( 2 ) + 1
, f 28 ( 2 ) = g ( g ( 2 )) =
= 1 2 .
2
2 g( 2)

Let x = 1, then f ( y + 1) f ( y ) = y + 2 . Let y = 0, then f (0) = 1 .


n 1

n2

For n > 0, f (n) f (0) = [ f ( y + 1) f ( y )] = ( y + 2) =


y =0

Similarly, for n < 0, f (n) =

8.

g ( x) =

y =0

n 2 + 3n 2
(n + 1)(n + 2)
1 and f (n) =
.
2
2

n + 3n 2
. Then f (n) = n if and only if n = 1 or 2.
2
2

Consider {0}, {1, 1994}, {2, 1993}, , {997, 998}. If we have 999 integers, then there are two of them when divided by
1995 leave remainders in the same set, hence their sum or difference is divisible by 1995. On the other hand the sequence
0, 1, 2, , 997 does not have the property.

9.

First, observe that if 0 < x 1 , then 0 < 1 x

1
1
1 because x + 2 . So q 1 for all triangles. By scaling, we
x
x

may assume that a triangle has lengths 1, q, r. Let q be the nearest ratio of the sides to the value one, then q 1 . If r 1 ,
1
then 1 q 1 r implies r q . Also, 1 q 1
implies r q 2 . By the triangular inequality, r + q 1 . So
r
5 1
1
1
q 2 + q 1 , which implies q
. If r > 1 , then 1 q 1
implies r . By the triangular inequality,
2
r
q
1 + q r . So 1 + q
is

5 1
1
, which also implies q
. So
2
q

3 5
, attained by the triangle with sides proportional to 1,
2

10. Let S n =

5 1
q 1 and the greatest possible value of | q 1 |
2
5 1 3 5
,
.
2
2

an bn bn cn cn an
+
+
.
ab
bc
ca

Since x n+3 = 2 x n+ 2 + 3x n+1 + 4 x n for x = a, b, c, we have S n+3 = 2 S n+ 2 + 3S n+1 + 4S n .


Since S 0 = 0 , S1 = 3 , S 2 = 4 , we have S3 = 17 , S 4 = 58 , S5 = 183 .
11. Suppose the answer has decimal representation a1a2 an . If ai 1 < ai , there cannot be more then 2 digits after ai .
(a + ai +1 )
Indeed, ai < i 1
implies ai +1 2ai ai 1 + 1 > ai + 1 . So ai +1 ai + 2 . Similarly, ai + 2 2ai +1 ai + 1 ai + 5
2
and ai+3 2ai + 2 ai +1 + 1 2(2ai +1 ai + 1) ai+1 + 1 = 3ai +1 2ai + 3 ai + 9 . Since ai 1 , ai +3 10 , a contradiction.
Similarly, if ai 1 > ai , there cannot be more then 2 digits before ai 1 . These facts imply the answer has at most 8 digits.
In the case of 8 digits, a3 > a4 = a5 < a6 . By the inequalities above, 9 a8 a6 + 5 (so a6 4 , a5 3 ), and

2a7 < a6 + a8 4 + 9 = 13 (so a7 6 ). Similarly, a1 9 , a2 6 , a3 4 , a4 3 . So the number is at most


96433469.

Taught By Mr. Sophorn ROEUNG, Dir. READ AND THINK, 2

(012-013) 47 36 17 & 099 72 3003

m
a a
be the common ratio in lowest form, then 211 = 2 + + a + ar + ar 2 . Note that
n
r
r
2
2
ar
a an
am
is rational. Now 2 = 2 and ar 2 = 2 are integers, so a is divisible by m2 and n2. Then a = km 2 n 2 .
a=
a
r
m
n

12. Let a be the middle term, r =

Since 211 is prime, k = 1. Then 211 = n 4 + n 3 m + n 2 m 2 + nm3 + m 4 . Since 4 4 = 256 > 211 , m < 4, n < 4. Simple checks
2
3
show r =
or
. Then a = 36 and the five terms are 16, 24, 36, 54, 81. So S = 16 + 36 + 81 = 133.
3
2
13. Consider the remainders of x, y, z when divided by 3. If they are all different, then ( x y )( y z )( z x) is not divisible
by 3, but x + y + z is divisibly by 3. So at least two remainders are the same. Then the third remainder must also be the
same and so 27 divides x + y + z . Let y = x + 3a , z = x + 3b , where a and b are distinct and non-zero. Let x + y + z =
27d. Substituting into the equation, we get 27ab(b a ) = 27d , and | d | 2 . Finally, let x = 15, y = 18, z = 21, we get

(15 18)(18 21)(21 15) = 15 + 18 + 21 = 54 .

14. Suppose a < b < c. Solving a + b = x 2 , a + c = y 2 , b + c = z 2 , we get a =

c=

x2 + y2 z2
x2 y2 + z2
, b=
,
2
2

x2 + y2 + z2
. So either all squares are even or two squares are odd and one even. Checking the smallest possible
2

cases, we find x 2 = 25 , y 2 = 36 , z 2 = 49 . So a = 6, b = 19, c = 30 and a + b + c = 55 .

1
1
BD . Also, GD // FE, GD = FE and AG = GF. Let FE = x, then
2
2
x
3x
GH BG 3
BD = 2x, GD =
and BG =
. Since BGH and EFH are similar, we have
=
= . Now AG = GF
2
2
HF EF 2
10
GH
3
implies AF = 2GF = GH . Thus, [ BGH ] =
[ ABF ] = (30) = 9 .
3
AF
10

15. Let F be the mid-point of BC, then FE // BD, FE =

Taught By Mr. Sophorn ROEUNG, Dir. READ AND THINK, 3

(012-013) 47 36 17 & 099 72 3003

International Mathematical Olympiad 1995


Hong Kong Preliminary Selection Contest
28th May, 1994
Time allowed: 3 hours
Answer ALL questions.
Put your answers on the answer sheet.
The use of calculators is prohibited.
1.

In a trapezium ABCD where AB is parallel to DC, the diagonals intersect at E. The area of ABE is 72 and
the area of CDE is 50. What is the area of trapezium ABCD?

2.

(1 mark)

Let P0 ( x ) = x 3 + 9 x 2 + 9 x + 4 . For integers n 1 , define Pn ( x) = Pn 1 ( x n) . What is the coefficient of


x 2 in P100 ( x ) ?

(1 mark)

1+ 1+ 1+ x = x .

3.

Find the real number x satisfying

4.

A point (a, b) on the xy-plane is integral if both a and b are integers. Determine the number of integral

(1 mark)

points enclosed by the square (including the boundary) formed by the points (100, 0), (0, 100), (100, 0)
and (0, 100).
5.

(1 mark)

a, b, c, d, e are real numbers satisfying a + b + c + d + e = 37 and 2 a + 2 b + 2 c + 2 d + 2 e = 1024 . What is


the greatest possible value of a?

6.

(1 mark)

Ten integers, with two of them equal, have the property that when nine of them are added, the distinct
possible sums are 86, 87, 88, 89, 90, 91, 92, 93 and 94. Find the integer that occurs twice.

7.

In the figure, ABC is an equilateral triangle. PE = 1, PQ= 3 and

(1 mark)

AE = CD. Find AD.


E
P

Q
B
8.

In the figure, AC is the diameter of a unit circle ADC with B as

the centre. Another circle BED touches AC at B and the circle


E

ADC at D. A tangent from A to the circle BED meets the circle at

E and BD produced at F. Find AF.


A

C
(1 mark)

Taught By Mr. Sophorn ROEUNG, Dir. READ AND THINK, 1

(012-013) 47 36 17 & 099 72 3003

9.

If f ( x) =

9x
1
2
1994
, find f
+ f
++ f
.
9 +3
1995
1995

1995

(2 marks)

x + y = 13

10. Let x, y, z be positive numbers satisfying the system of equations y 2 + z 2 yz = 25 .


x 2 + z 2 + xz = 144

Find z.

(2 marks)

11. What is x + y + z if x, y, z are integers greater than 1 such that xy 1 is divisible by z, yz 1 is


divisible by x, and zx 1 is divisible by y?

(2 marks)

12. What is the smallest positive integer that can simultaneously be expressed as the sum of 9 consecutive
positive integers, the sum of 10 consecutive positive integers, and the sum of 11 consecutive positive
integers?

(2 marks)

13. How many ordered four-tuples of integers (a, b, c, d) with 0 < a < b < c < d < 100 satisfy a + d = b + c
and bc ad = 94 ?

(2 marks)

14. Find the largest positive integer n satisfying the following conditions:
(a) n is not divisible by 10;
(b) the number formed by deleting the four rightmost digits from n2 is a perfect square.
15. ABCD is a rectangle with AB = 1. Two straight lines, parallel to AB and BC respectively, divide the
rectangle into four smaller triangles such that three of them have areas not less than one and the fourth has
area not less than 2. What is the least possible length of BC?

(2 marks)

16. Consider an equilateral triangle ABC. Let S be the collection of points P inside the triangle such that the
distances from P to the sides of ABC can be the lengths of the three sides of some triangle. Find
area of S
.
area of ABC

(2 marks)

17. Let CH be an altitude of ABC. Let R and S be, respectively, points at which the circles inscribed in
triangles ACH and BCH touch CH. If AB = 1995, AC = 1994 and BC = 1993, find RS.

(3 marks)

18. For positive integers i and j, the function (i, j) is defined by the relations
f (1, 1) = 2 ,

f (i + 1, j ) = 2(i + j ) + f (i, j ) and


f (i, j + 1) = 2(i + j 1) + f (i, j ) .
Find i and j such that f (i, j ) = 1994 .

(3 marks)
END OF PAPER

Taught By Mr. Sophorn ROEUNG, Dir. READ AND THINK, 2

(012-013) 47 36 17 & 099 72 3003

International Mathematical Olympiad 1995


Hong Kong Preliminary Selection Contest

Outline of solutions

Answers:

1.

242

2.

15141

3.

1
(1 + 5 )
2

4.

20201

5.

6.

10

7.

8.

5
3

9.

997

10.

40 3
13

11.

10

12.

495

13.

54

14.

4901

15.

3+ 2 2

16.

1
4

17.

332
665

18.

i = 7, j = 39

Explanations:

1.

Since ABE ~ CDE,

AE
72 6
6
=
=
and [ AED] = 50 = 60 . Similarly, [BCE] = 60. So area of trapezium ABCD
EC
50 5
5

= 50 + 60 + 60 + 72 = 242.
2.

100(x) = 99(x 100 ) = = 0(x 100 99 1 ) = 0(x 5050 ) = ( x 5050) 3 + 9( x 5050) 2 + 9( x 5050) + 4
= x 3 15141x 2 +

3.

Note x = 1 + 1 + 1 + x 0. If x > 1 + x , then x > 1 + 1 + x > 1 + 1 + 1 + x = x , a contradiction. Similarly,


if x < 1 + x , then x < 1 + 1 + x < 1 + 1 + 1 + x = x , a contradiction. So x = 1 + x , which gives the solution

x=

1
(1 + 5 ).
2

4.

Counting horizontally, the total is 1 + 3+199 + 201 + 199 +3 + 1 = 20201 .

5.

Since

x+ y

(2 2 + 2 2 ) 2 0 ,

2 x + 2 y 2( 2

1
(1024 2 a ) , which implies 210 = 1024 2
4

Hence,

37 a
4

=2

b+c+d +e
4

1
2
2

b+c
2

+2

d +e
2

1 b
c
d
e
= [2 + 2 + 2 + 2 ] =
4

3 a + 45

45 a
4

+ 2 a 2( 2

) . This gives a 9. Now a = 9 is possible by taking

b = c = d = e = 7.
6.

Let the numbers be n1 , n2 , , n10 in non-decreasing order and S = n1 + n2 + + n10 . Suppose x is the sum that
occurs twice, then 86 + 87 + + 94 + x = ( S n1 ) + ( S n2 ) + + ( S n10 ) = 9S . So x = 9( S 90) is divisible by 9.
Since 86 x 94 , x = 90, S = 100 and the ni that occurs twice is ni = S x = 10.

Taught By Mr. Sophorn ROEUNG, Dir. READ AND THINK, 1

(012-013) 47 36 17 & 099 72 3003

7.

Since ABE CAD (SAS), AEB = ADC. Then P, D, C, E are concyclic. So BPQ = ECQ = 60, BP =

3 sec 60 = 6 , AD = BE = 6 + 1 = 7.
8.

Let O be the midpoint of BD. Since tan OAB =

1
4
, we have BF = tan FAB = tan (2 OAB) =
.
2
3

4 5
AF = 1 + ) 2 = .
3 3

9.

Since f (1 x) =

10. Using

91 x
9
3
, so f ( x ) + f (1 x) = 1 . Then
=
=
x
9 + 3 9 + 39
3 + 9x
1 x

x = 13 y and rewriting the

z
3z 2
[13 ( y )] 2 +
= 144. Eliminating
2
4

1994

f (1995 ) =
k =1

z
3z 2
(y )2 +
= 25
2
4
z 25
3z 2 3600
40 3
we get y =
. Then
=
and z =
.
2 13
4
169
13

second and the third equations, we

3z 2
4

1994
= 997 .
2

get

and

11. We may assume 2 x y z. Note xy + yz + zx 1 is divisible by x, y, z and x, y, z are pairwise without any common
1 1 1
1
prime divisors. So xy + yz + zx 1 = kxyz for some positive integer k. Then
+ + =
+ k > 1. This has one
x y z xyz
solution (x, y, z) = (2, 3, 5). So x + y + z = 10 .
12. Since the sum of n consecutive integer beginning with a is

n
(2a + n 1). The required number is divisible by 9, 5, 11,
2

hence divisible by 495. Now 495 = 51 + 52 + + 59 = 45 + 46 + + 54 = 40 + 41 + + 50 , so 495 is the smallest such


number.
13. Note that a, b, c, d are of the form a, a + x , a + y , a + x + y with 0 < x < y. Now 94 = bc ad = xy imply (x, y) =
(1, 94) or (2, 47). If (x, y) = (1, 94), then since 0 < a < d < 100, a = 1, 2, 3, 4. If (x, y) = (2, 47), then since 0 < a < d < 100,
a = 1, 2, 3, , 50. So there are 4 + 50 = 54 cases for a, b, c, d.

n = 100a + b , where a, b are positive integers, 0 < b < 100. Then n 2 = 10000a 2 + 200ab + b 2 . If
n2
2
2
200ab + b 2 10000 , then
is a perfect square greater than a , a contradiction. So 200ab + b > 10000 , then
10000

14. Let

a ab <

9999
. The largest a is 49, then b = 1 and n = 4901. We can check 4901 works.
200

15. Suppose the four small rectangles have areas S1 , S 2 , S3 , S 4 with S1 , S 2 , S3 1 , S 4 2 , and S 2 , S3
correspond to rectangles containing B, D respectively. Considering side lengths, we have S 2 S 3 = S1 S 4 2 . So BC =
[ABCD] = S1 + S 2 + S 3 + S 4 1 + 2 S 2 S 3 + 2 3 + 2 2 . Now BC = 3 + 2 2 is possible by cutting AB = ( 2 1) +

(2 2 ) , BC = (1 + 2 ) + (2 + 2 ) .
16. We may let A = (1, 0), B = (1, 0), C = (0, 3 ) and P = (x, y) inside ABC. The distance from P to the sides of ABC are
1
1
( 3 3x y) ,
( 3 + 3 x y ) and y. For these to be the lengths of a triangle, the triangular inequality yields
2
2

3 > 2 y , y > 3 x , y > 3 x . These conditions correspond to the points inside the triangle formed by the midpoints
1
of AB, BC, CA. So the answer is
.
4

Taught By Mr. Sophorn ROEUNG, Dir. READ AND THINK, 2

(012-013) 47 36 17 & 099 72 3003

17. Let x, y be the radii of the inscribed circles of ACH and BCH
respectively and T be the point of tangency to the inscribed circle

of ACH on AH.
First, 1994 = AC = AT + CR = ( AH x ) + (CH x) .
1
So x = ( AH + CH 1994) .
2
1
Similarly, y = BH + CH 1993) .
2
1
Then RS = x y = ( AH BH 1) . By Pythagoras theorem,
2
1994 2 AH 2 = CH 2 = 19932 BH 2 . This implies AH BH =
1 3987 332
1994 2 19932 3987
=
and RS =
1 =
.
AH + BH
1995
2 1995 665

1994

1993

R
x
A

S
H

y
B

18. Since f (k , 1) = 2k + f (k 1, 1) , by induction, 2 + 4 + + 2k = k (k + 1) . Since f (k , h) = f (k , h 1) + 2( k + h 2) , by


induction,

f (k , h) = (k + h) 2 k 3h + 2 . For s > h,

f ( s h, h) = ( s 2 s 2) 2h . In particular,

f (46 h, h) =

2072 2h = 1994 for h = 39. So f (7, 39) = 1994 .

Taught By Mr. Sophorn ROEUNG, Dir. READ AND THINK, 3

(012-013) 47 36 17 & 099 72 3003

International Mathematical Olympiad 1994


Hong Kong Preliminary Selection Contest
5th June 1993
Time allowed: 3 hours
Answer ALL questions. Put your answers in the answer sheet provided.
1.

Let x be a real number and y = x 1 + x 2 + + x 1993 . Determine the smallest possible value of y.
(1 mark)

2.

Find a square number with four digits, such that the first (leftmost) digit is equal to the second digit, and the
third digit is equal to the fourth digit.

3.

(1 mark)

The leftmost digit of a certain positive integer is 4. If this digit is moved to the right of the rightmost digit,
1
of the original number. What is the smallest possible value of
then the new number formed is equal to
4
the original number?

4.

(1 mark)

ABCD is a rectangular court with AB = 30 m and BC = 40 m. Four men stand at different positions in the
court so that the distance between the two men nearest to each other is maximized. What is this distance?

5.

In the diagram, ABCD is a unit square. E, F are mid-points of AB

(1 mark)

and BC. AF meets ED and BD at H and K respectively. Find the


area of the quadrilateral EHKB.

(1 mark)

H
E
K

6.

Find the largest prime number p such that p 3 + p 2 + 11 p + 2 is also prime.

(2 marks)

7.

For which natural number k does

k2
1.001k

(2 marks)

8.

For any sequence A of real numbers a1 , a 2 , a 3 , , define A to be the sequence a 2 a1 , a 3 a 2 ,

attain its maximum value?

a 4 a 3 , , whose nth term is a n +1 a n . Suppose that all of the terms of the sequence (A) are 1, and
that a19 = a 93 = 0 . Find a1 .
9.

(2 marks)

Find the sum of all positive rational numbers that are less than 10 and that have denominator 30 when
written in lowest terms.

10. Given 12 rods of lengths 1, 2, 3, , 12, in how many ways can we choose 3 rods to form a triangle?
Taught By Mr. Sophorn ROEUNG, Dir. READ AND THINK, 1

(2 marks)

(012-013) 47 36 17 & 099 72 3003

11. Let n be a natural number. For 1 x n , find the number of solutions of x 2 [ x 2 ] = ( x [ x ]) 2 , where [x]
denote the greatest integer not exceeding x.

(3 marks)

12. A unit square is completely covered by three identical circles. Find the smallest possible diameter of the
circles.

(3 marks)

13. Triangle ABC is equilateral. O is a point inside the triangle such that AO = 3, BO = 4 and CO = 5. Find the
area of ABC.

(3 marks)

14. In triangle ABC, A', B' and C' are on sides BC, AC and AB respectively. Given that A A', BB' and CC' are
AO BO CO
AO BO CO
concurrent at the point O. If
+
+
= k , find, in terms of k, the value of

.
OA' OB' OC '
OA' OB' OC '
15. Let a, b, c, d, e, be positive integers such that a < b < c < d < e . Find the maximum value of
1
1
1
1
+
+
+
, where [m, n] denotes the L.C.M. of m and n.
[a, b] [b, c] [c, d ] [d , e]

(3 marks)

(3 marks)

END OF PAPER

Taught By Mr. Sophorn ROEUNG, Dir. READ AND THINK, 2

(012-013) 47 36 17 & 099 72 3003

International Mathematical Olympiad 1994


Hong Kong Preliminary Selection Contest

Outline of solutions

Answers:

1.

993012

2.

7744

3.

410256

4.

31.25

5.

7
60

6.

7.

2001

8.

828

9.

400

10.

95

11.

n2 n + 1

12.

13.

9+

14.

k+2

15.

15
16

25 3
4

65
8

Explanations:
1.

The value of x k + x (1994 k ) is smallest in the interval [k, 1994 k ], where 1 k 996 and x 997 is smallest
when x = 997 . At x = 997 , the value of y is smallest and the value = 2(996 + 995 + + 1) = 996 997 = 993012 .

2.

Let x be the first digit, y the last digit and N the number. Then N 2 = 1100 x + 11 y = 11(100 x + y ) . Hence

100 x + y
is a
11

perfect square. Since 100 x + y = 99 x + x + y , 11 is a factor of x + y . The possible values of (x, y) are (2, 9), (3, 6), (4, 7), (5,
6), (6, 5), (7, 4), (8, 3) and (9,2).
By trial, only (7, 4) satisfies the requirements. So the square number is 7744.
3.

Let A be a number with n 1 digits. Then 10 A + 4 =

1
(4 10n + A) implies 13 A = 4 333 332 . So 333 3332 is
4

divisible by 13. By trial, the smallest possible case is 33332. So A = 10256 and the original number is 410256.
4.

To satisfy the given condition, two men will stand at B and D, and

xm P

(40 x) m

the other two men will stand at P and Q so that BPDQ forms a
rhombus. Then

302 + x 2 = 40 x and x = 8.75. The shortest

30 m

distance is (40 8.75) m = 31.25 m.


B

5.

DK
BD
1
. Hence BK =
and the area of ABK =
. Produce AF to meet DC produced
2
3
60
DH
ED
1
1 1
7
at L, we have EH =
. So EH =
and area of AEH =
. It follows that the area of EHKB =
.
=
4
5
20
6 20
60

Since BKF~DKA, we have BK =

Taught By Mr. Sophorn ROEUNG, Dir. READ AND THINK, 1

(012-013) 47 36 17 & 099 72 3003

6.

Let f ( p ) = p 3 + p 2 + 11 p + 2 . Then f (3) = 71 is prime, but f (3k + 1) = 3(9k 3 + 12k 2 + 16k + 5) and f (3k + 2) =
9(3k + 7 k 2 + 9k + 4) are not prime. So the only and therefore the largest such prime is 3.

7.

Set

(k + 1) 2
k2
<
, then k 2 2000k 1000 > 0 , so k (k 2000) > 1000 and k 2001 . To attain the greatest value,
(1.001) k +1 (1.001) k

we have k = 2001.
8.

Suppose the first term in the sequence A is d. Then sequence

A is d, d + 1 , d + 2 , . Sequence A is a1 , a1 + d ,

1
(n 1)(n 2) .
2
1
1
and a19 = a93 = 0 , we have an = ( n 19)(n 93) , so
Since an is a quadratic polynomial, the leading coefficient is
2
2
1
a1 = (1 19)(1 93) = 828 .
2
a1 + d + (d + 1) , a1 + d + ( d + 1) + (d + 2) , , whose nth term is an = a1 + (n 1)d +

9.

The required rational number can be written in the form

30n + r
where n and r are integers satisfying 0 n 9 and
30

0 r 29 . Such a fraction is in lowest form (r, 30) = 1 r 1, 7, 11, 13, 17, 19, 23, 29, so there are 10 choices for
n and 8 choices for r, and no two pairs of choices (n, r) give the same value of
If

30n + r
. Hence there are 8
30

10 = 80 terms.

k
10 k
is one of these fractions then
is as well and their sum is 10. Thus the sum of all such fractions is 40 10 = 400.
30
30

10. Let T (n) be the number of triangles from n rods of lengths 1, 2, , n. If a rod of length n + 1 is added, sums of lengths of
others two sides of new triangles with these sums are 1, 1, 2, 2, 3, 3, ( n 2 terms). So T (n + 1) T (n) =

n(n 2)
if n is even, (n 1) 2 if n is odd. T ( 2n) T (2n 2) = [T (2n) T (2n 1)] +
4
[T ( 2n 1) T (2n 2)] = ( n 1) 2 (n 1)(n 2) = 2n 2 5n + 3 .

1+1+ 2 + 2 + 3 + 3 +=

T ( n ) =

n(n 1)(4n 5)
and so T (12) = 95 .
6

11. Since n 2 [n 2 ] = 0 and (n [ n])2 = 0 , so n is a solution. Let 1 x < n and m = [x]. Then x = m + r with 0 r < 1 . Thus
[ x 2 ] = m 2 + [2mr + r 2 ] and m 2 + 2mr + r 2 m 2 [2m + r 2 ] = r 2 , 2mr = [2mr + r 2 ] , so 2mr is an integer. When r = 0,

1
,
2m

2
2m 1
, or
, 2mr is an integer. So for each m, there are 2m choices for r, i.e. 2m solutions for each m. Hence the
2m
2m
number of solutions is 1 + 2[1 + 2 + + (n 1)] = n 2 n + 1 .
12. Let ABCD be a unit square and M, N the mid-points of DC, PQ

respectively. Suppose the three rectangles are completely covered


by 3 identical circles, then the smallest circles are their
circumcircles.
x2 1 +

Let

the

diameter

be

x,

65
x 1
= AP + PD = 1 and so x =
.
4
8
2

then
P
A

Taught By Mr. Sophorn ROEUNG, Dir. READ AND THINK, 2

N
B

(012-013) 47 36 17 & 099 72 3003

13. Around each vertex of ABC, rotate two smaller triangles as

shown.

1
9 3
16 3
25 3
25 3
Area = 6 +
+6+
+6+
.
=9+
2
4
4
4
4

3
A

3
3
O
4
4
4

AO [ AOB] [COA]
=
=
OA' [ A' OB] [COA' ]

[ AOB] + [COA] [ AOB] + [COA] x + z


=
=
=
, where x = [AOB],
[ A' OB] + [COA' ]
[ BOC ]
y
y = [BOC] and z = [COA].
CO
y+z
BO x + y
Similarly,
=
and
=
.
OB'
z
OC '
x
AO BO CO ( x + z )( x + y )( y + z ) x + z x + y

=
=
+
+
OA' OB' OC '
xyz
y
z

5
4

14. Let [XYZ] be the area of DXYZ. Then

A
B
O
A

y+z
AO BO CO
+2=
+
+
=k +2.
x
OA' OB ' OC '
15. Let S =

1
1
1
1
+
+
+
. Since 1 a < < e , so [ a, b] 2a , [b, c ] 2b , [c, d ] 2c , [ d , e] 2d and
[a, b] [b, c] [c, d ] [d , e]

b 2, c 3.

Case 1: c = 3. If d = 4, then [a, b] = 2, [b, c] = 6, [c, d] = 12, [d , e] 8 , so S


[c, d ] 6 , [ d , e] 10 , so S

1 1 1 1 7
15
+ + + = (<
). If d 5 , then
2 6 12 8 8
16

15
1 1 1 1 14
+ + +
=
(<
).
2 6 6 10 15
16

1 1 1
1
9
15
+ +
+
=
(<
), except when c = 4 and
2 4 20 10 10
16
1 1 1 1 11
15
1 1 1 1 15
d = 6 where S + + +
=
(<
). If d 8 , then S + + +
=
.
2 4 2 12 12
16
2 4 8 16 16
15
When a = 1, b = 2, c = 4, d = 8 and e = 16, S =
which is the maximum value.
16
Case 2: c 4 . If 5 d 7 , then [c, d] = 20, 28, 30, 35 or 42, so S

Taught By Mr. Sophorn ROEUNG, Dir. READ AND THINK, 3

(012-013) 47 36 17 & 099 72 3003

International Mathematical Olympiad 1993


Hong Kong Preliminary Selection Contest
6th June 1992
Time allowed: 3 hours
Answer ALL questions. Put your answers on the answer sheet provided.

a b c
a+b+c
.
= = , find
b c a
a+bc

1.

Given three positive numbers a, b and c such that

2.

1
1 1
Let a, b and c be positive numbers satisfying a + b + c = 1 . Find the least value of 1 1 1 .
a

b c

3.

Let a and b be real numbers satisfying the equation

(1 mark)

(1 mark)

log10 (1 + a 2 ) log10 a 2 log10 2 = 1 log10 (100 + b 2 ) + log10 b .


Find the value of a + b .

(1 mark)

xy
= 1992 .
x+ y

4.

Find the number of positive integral solutions satisfying the equation

5.

Find a positive integer which has exactly 8 factors, and the product of all these 8 factors is 331776.

6.

In the multiplicative magic square, a, b, c, , i are positive integers. The


product of the three numbers in any row, column or diagonal is equal to a
constant k, where k is a number between 10000 and 12000. Find the value of k.

(1 mark)

(1 mark)

7.

Express the number 1992 as the sum of positive integers such that the product of these integers is maximal.

8.

A rhombus PQRS is inscribed in a rectangle ABCD so that the vertices P, Q, R and S lie on the sides AB, BC,

9.

(1 mark)
(2 marks)

CD and DA respectively. Given that PB = 3, BQ = 4, PR = 6 and QS = 8, find the perimeter of ABCD.

(2 marks)

Find the largest value of a such that both roots of the quadratic equation x 2 + ax + 6a = 0 are integers.

(2 marks)

10. Find the least number of integers that must deleted from 1, 2, 3, , 1992 so that none of the remaining
integers is the product of two others.
11. For any two positive numbers x and y, let s be the smallest value of x, y and

(2 marks)

1 1
+ . Find the greatest
x y

possible value of s.

(2 marks)

12. In rectangle ABCD, P and Q are points on AB and BC respectively, such that the triangles APD, PBQ and
AP
QCD have equal area. Find
.
PB

Taught By Mr. Sophorn ROEUNG, Dir. READ AND THINK, 1

(2 marks)

(012-013) 47 36 17 & 099 72 3003

13. A rectangle PQRS is folded along the diagonal QS such that the planes QRS and PQS are perpendicular to
each other. Given that PQ = 4, PS = 3. Find PR.

(2 marks)

14. In ABC, A B = 90 and BC + CA = 2AB. Find cos C.

(2 marks)

15. For any positive integer n, let a1 , a2 , , an be any positive real numbers whose sum is 17. Define Sn to
n

be the minimum value of

(2k 1) 2 + a k 2 . Find the value of n such that S n is an integer.

(3 marks)

k =1

END OF PAPER

Taught By Mr. Sophorn ROEUNG, Dir. READ AND THINK, 2

(012-013) 47 36 17 & 099 72 3003

International Mathematical Olympiad 1993


Hong Kong Preliminary Selection Contest
Outline of solutions
Answers:

1.

2.

3.

11

4.

63

5.

24

6.

10648

7.

3664

8.

672
25

9.

49

10.

43

11.

12.

1+ 5
2

14.

3
4

15.

13.

337
5

2
12

Explanations:

1.

a b c a+b+c
= = =
= 1 as a + b + c 0 .
b c a b+c+a

2.

1
1 1 1 b + c a + c a + b 2 bc 2 ac 2 ab
= 8 . Now 8 is possible by taking a = b = c = .
1 1 1 =

3
abc
a b c
a b c

3.

From the given equation, we obtain (100 + b 2 )a 2 40ab + 100 + b 2 = 0 . For positive a, b, (b 10) 2 (b + 10) 2 0 , which
gives b = 10, a = 1.

4.

Rewrite the given equation as y =

1992 2
+ 1992 , where t = x 1992 > 1992 . As t must be a positive factor of
t

1992 2 = 26 32 832 , the number of solutions is (6 + 1) (2 + 1) (2 + 1) = 63.

5.

Let the 8 factors of the positive integer n be d1, d2, , d8, where d1 = 1, d8 = n and d1d 8 = d 2 d 7 = d 3 d 6 = d 4 d 5 . Hence

(d1d 8 ) 4 = 331776 which gives n = 24.


6.

(aei)(def )( gec)
= k gives e3 = k. Since 10000 k 12000 , k = 10648.
(adg )(cfi)

7.

For any positive integer a 5 , (a 2) + 2 = a, but 2 (a 2) > a. Hence the set of positive integers that the sum up to 1992
cannot contain integers 5 . Now 1992 is the sum of 332 6s. Note that 6 = 3 + 3 = 2 + 4 = 2 + 2 + 2 and the product 3 3
is greatest. So the answer is 3664.

8.

1
ab = (4 + a )(3 + b) which
2
gives 3a + 4b = 2 . But a 2 + b 2 = 25 , so 25a 2 144a + 176 = 0 .
44
117
From this we have a =
and b =
which gives the
25
25
672
perimeter
.
25
Considering the area, 6 6 + 2

Taught By Mr. Sophorn ROEUNG, Dir. READ AND THINK, 1

A
P

4
4

R
C

(012-013) 47 36 17 & 099 72 3003

9.

Let m, n (m > n) be integral roots of the equations. Then a must be an integer and a = (m + n), 6a = mn. Hence
6(m + n) = mn , i.e. (m + 6)(n + 6) = 36. This equation has 10 integral solutions and the largest value of a is 49.

10. Note that 44 45 = 1992 and 45 46 > 1992. So, if 2, 3, , 44 are deleted from the set, none of the remaining integers is the
product of two others. If 2 k 44 , then 45 89 k 87 and 174 k (89 k ) 1980 . So the required number is 43.
11. By definition x s , y s ,

12. Equating areas, we have

1 1
1 1 1 1
+ s . From + + s , we obtain the greatest possible value of s which is
x y
s s x y

1
1
1
( p1 + p2 )q1 = p1 ( q1 + q2 ) = p2 q2 .
2
2
2

p1 q1
q2
=
=
p2 q2 q1 + q2

Thus

From

this

we

C
q1

obtain

Q
q2

p1
p
p
1 + 5
+ 1 1 = 0 , 1 =
.
p
p
p
2
2
2
2
13. By similar triangles,

BR PQ
=
,
QE QS

BR =

12
,
5

2 sin

cos

= 4 cos cos

, cos

+ sin

, 2 cos 2

= 2(cos 2

+
x

x sin

3
sin ) which gives cos C = sin = .
4
2

x cos

x cos
A

15.

14. We have x(1 + cos + sin ) = 4 x cos cos

R
S

p2

193
337
So PB =
, PR =
.
5
5

p1

BS RS
16
9
7
=
, BS =
, BQ = , AB = .
RS QS
5
5
5

2.

2x cos cos

AB = ( ak ) 2 + [(2k 1)]2 = 17 2 + n 4 tk , where

tk = (2k 1) 2 + ak . Equality holds when all tk lie on AB.


2

Thus S n = 17 2 + n 4 . Since n is a positive integer,

17 2 = S n n 4 gives S n + n 2 = 17 2 and S n n 2 = 1 . So n

an

= 12.
tk

2k 1
ak

a2
A

Taught By Mr. Sophorn ROEUNG, Dir. READ AND THINK, 2

a1

(012-013) 47 36 17 & 099 72 3003

International Mathematical Olympiad 1992


Hong Kong Preliminary Selection Contest
6th July 1991
Time allowed: 3 hours
Answer all questions. Put your answers on the answer sheet provided.
1.

ABCD is a cyclic quadrilateral with AB = 25, BC = 39, CD = 52, and DA = 60. Find the length of the
diagonal BD.

(1 mark)

2.

Determine the unit digit of 19971991.

(1 mark)

3.

Let 2, 3, 5, 6, 7, 10, 11, be the increasing sequence of positive integers that are neither the square nor the
cube of an integer. Find the 1991st term of this sequence.

(1 mark)

4.

x y 99
Find the number of integral solutions (x, y) satisfying
.
x + y > 99

(1 mark)

5.

Determine the number of integers from 1 to 1991, both inclusive, which are not divisible by 2, 3 or 5.

(1 mark)

6.

The diagonals of the trapezium ABCD (with AB // DC) meet at E. The areas of ABE and CDE are 4 and 9
respectively. Find the area of ABCD.

(1 mark)

7.

Find the sum of the digits of all the numbers in the sequence 1, 2, 3, , 1991.

(1 mark)

8.

A boy wrote down this fathers age after his own. To this four-digit number he added 16 times the
difference between their ages and obtained 1991. Find the sum of their ages.

(1 mark)

9.

In the diagram, the diameter AD of the semicircle has length 3. B


and C are points on the semicircle such that AB = BC = 1. Find

the length of the chord CD.

1
A

D
(1 mark)

10. Let [x] denote the greatest integer less than or equal to x, and { x} = x [ x ] . Find the nonzero value of x
such that x, [x] and {x} are in geometric progression.

(1 mark)

11. Let N be a positive integer satisfying the following properties:


(i)
(ii)

N is a perfect cube.
None of the last 3 digits of N is zero.

(iii)
If the last 3 digits of N are removed, the number so formed is also a perfect cube.
Determine the maximum possible value of N.
12. A function f, denoted for all ordered pairs of positive integers, satisfies the following properties:
f ( x, x ) = x + 2 , f ( x, y ) = f ( y, x ) and ( x + y ) f ( x, y ) = y f ( x, x + y ) . Calculate f (9,7) .
13. For any positive integer n, let U n+1 =

n(n + 1)
. Suppose U 1 = a . Find U n in terms of a and n.
2n U n

Taught By Mr. Sophorn ROEUNG, Dir. READ AND THINK, 1

(1 mark)

(1 mark)
(1 mark)

(012-013) 47 36 17 & 099 72 3003

14. A flight of stairs consists of ten steps. A boy can climb one or two steps at each time. In how many ways can
the boy climb from the bottom to the top of the stairs?

(2 marks)

15. Let n be the smallest positive integer that is a multiple of 1991 and that has exactly 1991 positive integral
divisors; including 1 and itself. Express n as a product of prime factors.

(2 marks)

16. Let a, b and c be the lengths of the sides of a triangle and let p, q, r be respectively the distances from the
a2 + b2 + c2
centroid to the corresponding vertices. Find the ratio 2
.
p + q2 + r 2

(2 marks)

17. Let a be the sum of the digits of 310000, when it is written in decimal form, b be the sum of the digits of a and
c be the sum of the digits of b. Find c.

(2 marks)

18. Find the real solutions of the equation 3 x 2 [ x ] = 3 , where [x] denotes the greatest integer less than or

equal to x.

(2 marks)

19. What is the minimum value obtained when an arbitrary number of three different non-zero digits is divided
by the sum of its digits?
20. An equilateral triangle ABP with side AB of length 3 is placed
inside a square AXYZ with side AX of length 6. The triangle is

rotated clockwise about B until BP falls on BX; it is then rotated


about P until PA falls on XY, then about A, and so on along the
sides of the square until P, A and B all return to their original
positions. Find in terms of , the length of the path traversed by
the vertex P.

(2 marks)

(2 marks)

21. On a plane, there are 10 circles. Any pair of them intersect at exactly 2 points but no 3 of them intersect at
one common point. Into how many parts do these circles divide the plane?

(2 marks)

22. The integers 1, 2, , n are arranged in such a way that if an integer k ( 1 k n ) is not the first term, then
one of the integers k + 1 or k 1 appears among the terms on the left of k. How many different
arrangements are there?

(2 marks)

23. E, F, G are points on the sides AB, BC, CA respectively of ABC such that AE : EB = BF : FC = CG : GA
= 1 : 3. Let K, L, M be the intersection points of lines AF and CE, BG and AF, and CE and BG respectively.
area of KLM
Find the ratio
.
area of ABC
24. Let [x] denote the greatest integer less than or equal to x. How many of the first 100 positive integers can be
expressed in the form [2x] + [4x] + [6x] + [8x] where x is a real number?

(3 marks)

(3 marks)

25. A collection of k distinct integers is chosen from the integers 1, 2, , 1991. It is required that the difference
of any two integers from the collection does not divide the sum of the corresponding integers. Determine the
maximum possible value of k.

(3 marks)
END OF PAPER

Taught By Mr. Sophorn ROEUNG, Dir. READ AND THINK, 2

(012-013) 47 36 17 & 099 72 3003

International Mathematical Olympiad 1992


Hong Kong Preliminary Selection Contest
Outline of Solutions
Answers:
1.

65

2.

3.

2045

4.

2500

5.

531

6.

25

7.

27804

8.

58

9.

7
3

10.

1+ 5
2

11.

1728

12.

189

13.

a 1
n 1 +

n (n 1)a

14.

89

15.

11180 18011

16.

17.

18.

19.

10.5

20.

20

21.

92

22.

2n 1

23.

4
13

24.

60

25.

664

4
3

Explanations:

252 + 60 2 = 65 = 39 2 + 52 2 A = 90o =C and BD = 65.

1.

Since

2.

72 = 49, 73 = 343, 74 = 2401, so 71991 = 74(497) + 3 has the same unit digit as 73, which is 1.

3.

Between 1 and 1991, there are [ 1991 ] = 44 perfect squares, [3 1991] = 12 perfect cubes and [6 1991] = 3 perfect 6th
power. Beyond 1991, the next number to be skipped is 452 = 2025, so the 1991st term is 1991 + (44 + 12 3) + 1 = 2045.

4.

By graph, (x, y) lies inside or on ABC, where A = (0, 99), B = (44.5, 44.5), C = (99, 99), and not the edge of AB.
Counting horizontally, the answer is 1 + 3 + 5 + + 99 = 2500.

5.

1991 1991 1991 1991 1991 1991 1991


1991

+
+
+

= 531 .
2 3 5 2 3 3 5 5 2 2 3 5

6.

Since ABE and CDE are similar,

AE
4 2
3
3
=
= , [ BCE ] = 4 = 6 . Similarly [ AED] = 4 = 6 . So the area of
EC
9 3
2

2

ABCD = 4 + 9 + 6 + 6 = 25.
7.

1000
The sum of digits in 1, 2, , 999 is (0 + 1 + + 9)
3 = 13500 . So the sum of digits in 1, 2, , 1999 is 2(13500) +
10
1000 = 28000. Subtracting the contribution of 1992, , 1999, the answer is 27804.

Taught By Mr. Sophorn ROEUNG, Dir. READ AND THINK, 1

(012-013) 47 36 17 & 099 72 3003

8.

Let b be boys age and f be fathers age, then 100b + f + 16( f 6) = 1991 , so f = 117 5b +

b+2
. Since 17 divides
17

b + 2 , 117 > 5b, so b = 15, f = 43.


9.

Let O be the center and AC intersect BO at E. Note BEC and ABD are similar, since BCE = BDA. Now BD =

2 8
8
2 8
1
.
, CD = 3 2
, AC =
32 1 = 8 , EC = 8 =
3
3
3 3

10. [ x]2 = x{x}2 = x( x [ x]) implies

x 2 [ x]x [ x]2 = 0 , so

x=

[ x ] [ x ]2 + 4[ x ]2 1 + 5
=
[ x ] . Now 0 x [ x ] =
2
2

5 1

[ x] < 1 , so 0 [ x] < 5 + 1 . Then [x] = 1 and x = 1 + 5 .


2
2
2

N
3
3
3
2
2
11. Let N = n3 and
= k . Since 0 < n 1000k < 1000 , it follows 0 < n 10k , n + 10nk + 100k < 1000 . Then
1000
10k < n and (10k ) 2 + 10(10k )k + 100k 2 = 300k 2 < 1000 , yielding k = 1 and 1000 N 1999 . So N = 123 = 1728.

z
f ( x, z x ) .
zx
9
9 7
9 7 5 3 2
Then f (7,9) = f (7,2) = f (5,2) = = f (1,1) = 189 .
2
2 5
2 5 3 2 1

12. Setting z = x + y in the third equation, we get f ( x, z ) =

13. U 2 =

2
,
2a

U3 =

3( 2 a )
a 1
4(3 2a )
a 1

= 31 +
. U 4 = 4 3a = 4 1 + 4 3a ,
3 2a
3 2a

14. Let an be the number of way of climbing a stair of n steps. For n > 2, we have an = an 1 + an 2 . The sequence an is 1, 2, 3,
5, 8, 13, 21, 34, 55, 89,
15.

n = 11a 181b , where (a + 1)(b + 1) = 1991 , a 1 , b 1 . So (a, b) = (10, 180) or (180, 10). Then n = 11180 18110 .
2

9q 2 + b 2
9r 2 + c 2
3p
a 9 p2 + a2
, c2 + a2 =
, a2 + b2 =
. Adding these
16. By Apollonius formula, b 2 + c 2 = 2 + 2 =
2
2
2
2
2
equation, we have

17.

a2 + b + c2
=3.
p2 + q2 + r 2

log10 310000 = 10000 log10 3 5000 implies a 5000 9 = 45000 , b 3 + 9 + 9 + 9 + 9 = 39 , c 3 + 9 = 12 . Now 310000 ,
a, b, c have the same remainder when divided by 9, so c = 9.

18.

3 x 3 [ x] = 3 implies 2 < 3x 3 x 3 . By graphing y = 3x 3 x , we get 1 < x < 2, so 3 x 3 1 = 3 which implies


x=3

4
.
3

19. For digits a, b, c,

10 +

100a + 10b + c
99a + 9b
99a + 9b
90b + 991
90b + 991
9
=1 +
1+
= 100
100
b + 10 = 10 +
a+b+c
a+b+c
a+b+9
a+b+9
b + 10

9
= 10.5 . Equality holds for a = 1, b = 8, c = 9.
8 + 10

Taught By Mr. Sophorn ROEUNG, Dir. READ AND THINK, 2

(012-013) 47 36 17 & 099 72 3003

20. Three times around the inside of the square is needed. The total angle P rotated about A or B in 3 rounds is 420 + 450 +
20
20
330 = 1200 or
. So the distance P traversed is 3
= 20 .
3
3
21. Let an be the number of parts n circles partitioned the plane. Then a1 = 2 and an = an 1 + 2( n 1) because the first n
1 circles cut the nth circle into 2(n 1) arcs, each arc creates a new region.
10

10

n=2

n=2

We have a10 a1 = (an an 1 ) = 2(n 1) = 90 .

22. For such arrangement, each number after the first is either one greater than the greatest of its predecessors or one less than
the least of its predecessors. In particular, it ends in 1 or n. Discarding the last number, we are left with such kind of
arrangements for 1, 2, , n, then A1 = 2 and An = 2 An 1 . So An = 2 n 1 .

BF 1
12
12 1
3
= , [ FCK ] = 3[ FBK ] , [ ACK ] = 3[ ABK ] = 12[ AEK ] = [ AEC ] = [ ABC ] = [ ABC ] . By symmetry,
FC 3
13
13 4
13
[ KLM ]
3 4
= 1 3 = .
[ ABC ]
13 13

23. Since

24. Let f ( x ) = [2 x ] + [ x] + [6 x] + [8 x] . Note f ( x + n) = f ( x ) + 20n for integer n.


In [0, 1], such changes occurs at

1 1 1 1 3 1 5 2 3 5 7
, , , , , , , , , , , 1 . So 12 of the first 20 positive integers can be
8 6 4 3 8 2 8 3 4 6 8

expressed in desired form, and hence 60 of the first 100 integers can be expressed in desired form.
25. The 664 numbers 1, 4, 7, , 1990 satisfy the condition. If more than 664 numbers are chosen, then there will be two from
one of the following 664 groups {1, 2, 3}, {4, 5, 6}, , {1987, 1988, 1989}, {1990, 1991} and they will fail the
condition.

Taught By Mr. Sophorn ROEUNG, Dir. READ AND THINK, 3

(012-013) 47 36 17 & 099 72 3003

You might also like